ecuaciones diferenciales mediante simetrías · vi contents...

144
Ecuaciones Diferenciales Mediante Simetrías por José Antonio Belinchón Primer Borrador Junio 2005. Segundo Borrador Septiembre 2006 Último Borrador Mayo 2009

Upload: dangcong

Post on 10-Oct-2018

222 views

Category:

Documents


0 download

TRANSCRIPT

Page 1: Ecuaciones Diferenciales Mediante Simetrías · vi CONTENTS Lieperomuchísimomássencillodeaplicaryaqueunonodebeaplicarningúnalgoritmoparaobtener las simetrías que inducen dichos

Ecuaciones DiferencialesMediante Simetrías

porJosé Antonio Belinchón

Primer Borrador Junio 2005.Segundo Borrador Septiembre 2006

Último Borrador Mayo 2009

Page 2: Ecuaciones Diferenciales Mediante Simetrías · vi CONTENTS Lieperomuchísimomássencillodeaplicaryaqueunonodebeaplicarningúnalgoritmoparaobtener las simetrías que inducen dichos

ii

Page 3: Ecuaciones Diferenciales Mediante Simetrías · vi CONTENTS Lieperomuchísimomássencillodeaplicaryaqueunonodebeaplicarningúnalgoritmoparaobtener las simetrías que inducen dichos

Contents

Prólogo v

1 Dimensional Analysis 1

1.1 Assumptions in DA. . . . . . . . . . . . . . . . . . . . . . . . . . . . . . . . . . . . . . . . . . . . . . 1

1.2 The Recipe. . . . . . . . . . . . . . . . . . . . . . . . . . . . . . . . . . . . . . . . . . . . . . . . . . . . 2

1.3 The Pi-Theorem. . . . . . . . . . . . . . . . . . . . . . . . . . . . . . . . . . . . . . . . . . . . . . . . . 3

1.4 Examples. . . . . . . . . . . . . . . . . . . . . . . . . . . . . . . . . . . . . . . . . . . . . . . . . . . . 5

1.5 Applications of dimensional analysis to ODEs and PDEs . . . . . . . . . . . . . . . . . . . . . . . . 8

1.5.1 Differential equations. . . . . . . . . . . . . . . . . . . . . . . . . . . . . . . . . . . . . . . . . 8

1.5.2 Partial differential equations. . . . . . . . . . . . . . . . . . . . . . . . . . . . . . . . . . . . . 11

2 Introduction to the symmetry methods. 15

3 Ecuaciones de Primer Orden 25

3.1 Introducción . . . . . . . . . . . . . . . . . . . . . . . . . . . . . . . . . . . . . . . . . . . . . . . . . . 25

3.2 The method . . . . . . . . . . . . . . . . . . . . . . . . . . . . . . . . . . . . . . . . . . . . . . . . . . 26

3.3 Cuadraturas. . . . . . . . . . . . . . . . . . . . . . . . . . . . . . . . . . . . . . . . . . . . . . . . . . . 31

3.3.1 Simetrías. . . . . . . . . . . . . . . . . . . . . . . . . . . . . . . . . . . . . . . . . . . . . . . . 32

3.3.2 Ejemplos . . . . . . . . . . . . . . . . . . . . . . . . . . . . . . . . . . . . . . . . . . . . . . . . 32

3.4 Ecuaciones en variables separables y reducibles a ellas. . . . . . . . . . . . . . . . . . . . . . . . . . 33

3.4.1 Método tradicional. . . . . . . . . . . . . . . . . . . . . . . . . . . . . . . . . . . . . . . . . . . 34

3.4.2 Método dimensional. . . . . . . . . . . . . . . . . . . . . . . . . . . . . . . . . . . . . . . . . . 34

3.4.3 Método de Lie. . . . . . . . . . . . . . . . . . . . . . . . . . . . . . . . . . . . . . . . . . . . . 34

3.4.4 Ejemplo. . . . . . . . . . . . . . . . . . . . . . . . . . . . . . . . . . . . . . . . . . . . . . . . . 34

3.5 Ecuación homogénea . . . . . . . . . . . . . . . . . . . . . . . . . . . . . . . . . . . . . . . . . . . . . 36

3.5.1 Método tradicional. . . . . . . . . . . . . . . . . . . . . . . . . . . . . . . . . . . . . . . . . . . 37

i

Page 4: Ecuaciones Diferenciales Mediante Simetrías · vi CONTENTS Lieperomuchísimomássencillodeaplicaryaqueunonodebeaplicarningúnalgoritmoparaobtener las simetrías que inducen dichos

ii CONTENTS

3.5.2 Método dimensional. . . . . . . . . . . . . . . . . . . . . . . . . . . . . . . . . . . . . . . . . . 38

3.5.3 Método de Lie. . . . . . . . . . . . . . . . . . . . . . . . . . . . . . . . . . . . . . . . . . . . . 38

3.5.4 Ejemplos. . . . . . . . . . . . . . . . . . . . . . . . . . . . . . . . . . . . . . . . . . . . . . . . 40

3.6 Ecuación lineal de primer orden . . . . . . . . . . . . . . . . . . . . . . . . . . . . . . . . . . . . . . . 46

3.6.1 Método tradicional. . . . . . . . . . . . . . . . . . . . . . . . . . . . . . . . . . . . . . . . . . . 47

3.6.2 Método dimensional. . . . . . . . . . . . . . . . . . . . . . . . . . . . . . . . . . . . . . . . . . 47

3.6.3 Método de Lie. . . . . . . . . . . . . . . . . . . . . . . . . . . . . . . . . . . . . . . . . . . . . 47

3.6.4 Ejemplos. . . . . . . . . . . . . . . . . . . . . . . . . . . . . . . . . . . . . . . . . . . . . . . . 48

3.7 Ecuaciones exactas. . . . . . . . . . . . . . . . . . . . . . . . . . . . . . . . . . . . . . . . . . . . . . . 52

3.8 Ecuación de Bernoulli . . . . . . . . . . . . . . . . . . . . . . . . . . . . . . . . . . . . . . . . . . . . 55

3.8.1 Método tradicional. . . . . . . . . . . . . . . . . . . . . . . . . . . . . . . . . . . . . . . . . . . 55

3.8.2 Método dimensional. . . . . . . . . . . . . . . . . . . . . . . . . . . . . . . . . . . . . . . . . . 56

3.8.3 Método de Lie. . . . . . . . . . . . . . . . . . . . . . . . . . . . . . . . . . . . . . . . . . . . . 56

3.8.4 Ejemplos. . . . . . . . . . . . . . . . . . . . . . . . . . . . . . . . . . . . . . . . . . . . . . . . 57

3.9 Ecuaciones tipo Riccati. . . . . . . . . . . . . . . . . . . . . . . . . . . . . . . . . . . . . . . . . . . . . 62

3.9.1 Ejemplos. . . . . . . . . . . . . . . . . . . . . . . . . . . . . . . . . . . . . . . . . . . . . . . . 63

3.10 Ecuaciones tipo Abel y Chini. . . . . . . . . . . . . . . . . . . . . . . . . . . . . . . . . . . . . . . . . 66

3.10.1 Método tradicional . . . . . . . . . . . . . . . . . . . . . . . . . . . . . . . . . . . . . . . . . . 66

3.10.2 Ejemplos . . . . . . . . . . . . . . . . . . . . . . . . . . . . . . . . . . . . . . . . . . . . . . . . 69

3.10.3 Pathological cases. . . . . . . . . . . . . . . . . . . . . . . . . . . . . . . . . . . . . . . . . . . 73

3.11 Ecuaciones de D’Alambert-Lagrange y de Clairaut . . . . . . . . . . . . . . . . . . . . . . . . . . . . 76

3.11.1 Método tradicional. . . . . . . . . . . . . . . . . . . . . . . . . . . . . . . . . . . . . . . . . . . 76

3.11.2 Método dimensional. . . . . . . . . . . . . . . . . . . . . . . . . . . . . . . . . . . . . . . . . . 76

3.11.3 Ejemplos . . . . . . . . . . . . . . . . . . . . . . . . . . . . . . . . . . . . . . . . . . . . . . . . 77

3.12 Conclusions and discussion. . . . . . . . . . . . . . . . . . . . . . . . . . . . . . . . . . . . . . . . . . 79

4 Ecuaciones de segundo orden. 83

4.1 Introducción . . . . . . . . . . . . . . . . . . . . . . . . . . . . . . . . . . . . . . . . . . . . . . . . . . 83

4.2 Distintos tipos de ecuaciones. . . . . . . . . . . . . . . . . . . . . . . . . . . . . . . . . . . . . . . . . 85

4.2.1 Lineales con coeficiente constantes. . . . . . . . . . . . . . . . . . . . . . . . . . . . . . . . . 85

4.2.2 Ecuación de Bessel. . . . . . . . . . . . . . . . . . . . . . . . . . . . . . . . . . . . . . . . . . . 89

4.2.3 Ecuación de Duffing. . . . . . . . . . . . . . . . . . . . . . . . . . . . . . . . . . . . . . . . . . 89

4.2.4 Ecuación de Emden y Emden-Fowler. . . . . . . . . . . . . . . . . . . . . . . . . . . . . . . . 90

Page 5: Ecuaciones Diferenciales Mediante Simetrías · vi CONTENTS Lieperomuchísimomássencillodeaplicaryaqueunonodebeaplicarningúnalgoritmoparaobtener las simetrías que inducen dichos

CONTENTS iii

4.2.5 Ecuación lineal exacta. . . . . . . . . . . . . . . . . . . . . . . . . . . . . . . . . . . . . . . . . 91

4.2.6 Ecuación no lineal exacta. . . . . . . . . . . . . . . . . . . . . . . . . . . . . . . . . . . . . . . 91

4.2.7 La ecuación lineal. . . . . . . . . . . . . . . . . . . . . . . . . . . . . . . . . . . . . . . . . . . 92

4.2.8 Ecuación de Largerstrom. . . . . . . . . . . . . . . . . . . . . . . . . . . . . . . . . . . . . . . 93

4.2.9 Ecaución de Lienard. . . . . . . . . . . . . . . . . . . . . . . . . . . . . . . . . . . . . . . . . . 93

4.2.10 La ecuación de Liouville. . . . . . . . . . . . . . . . . . . . . . . . . . . . . . . . . . . . . . . 94

4.2.11 La ecuación de Painleve. . . . . . . . . . . . . . . . . . . . . . . . . . . . . . . . . . . . . . . . 94

4.2.12 La ecuación de Titchmarch. . . . . . . . . . . . . . . . . . . . . . . . . . . . . . . . . . . . . . 95

4.2.13 La ecuación de Van der Pol. . . . . . . . . . . . . . . . . . . . . . . . . . . . . . . . . . . . . . 95

4.3 Ejemplos . . . . . . . . . . . . . . . . . . . . . . . . . . . . . . . . . . . . . . . . . . . . . . . . . . . . 96

5 Ecuaciones de tercer orden. 113

5.1 Ejemplos . . . . . . . . . . . . . . . . . . . . . . . . . . . . . . . . . . . . . . . . . . . . . . . . . . . . 113

6 Aplicaciones en problemas cosmológicos. 121

6.1 Distintas Aplicaciones . . . . . . . . . . . . . . . . . . . . . . . . . . . . . . . . . . . . . . . . . . . . 121

6.1.1 Lie method for the H-equation. . . . . . . . . . . . . . . . . . . . . . . . . . . . . . . . . . . . 121

6.1.2 Fluidos perfectos con constantes variables . . . . . . . . . . . . . . . . . . . . . . . . . . . . 124

6.1.3 Fluidos viscosos . . . . . . . . . . . . . . . . . . . . . . . . . . . . . . . . . . . . . . . . . . . . 129

Page 6: Ecuaciones Diferenciales Mediante Simetrías · vi CONTENTS Lieperomuchísimomássencillodeaplicaryaqueunonodebeaplicarningúnalgoritmoparaobtener las simetrías que inducen dichos

iv CONTENTS

Page 7: Ecuaciones Diferenciales Mediante Simetrías · vi CONTENTS Lieperomuchísimomássencillodeaplicaryaqueunonodebeaplicarningúnalgoritmoparaobtener las simetrías que inducen dichos

Prólogo

Estas notas sobre ecuaciones diferenciales ordinarias (odes) análisis dimensional (AD) y grupos de Lie (GL) estándivididas en varios capítulos (no terminados) como sigue:

1. Cap.1. En este capítulo (no terminado) dedicado al AD intento resaltar la estructura de grupo (grupo deLie, grupo generado por la simetría de escala) que tiene, con esto quiero decir que la solución que unoobtiene aplicando el teorema Pi (a una ode dimensionalmente homogénea) es la misma que la generada(obtenida) como solución invariante por la simetría de escala. Esto es debido a la estructura de grupo detiene el AD. Para intentar hacer ver como funciona el método expongo un ejemplo muy concreto de unproblema cosmológico. Tal y como está redactado este capítulo, pienso que uno debería intentar fijarsemás en el método y no en el ejemplo.

Observación 0.0.1 El AD que sigo es básicamente el AD de J. Palacios aunque he tomado (adoptado) todas lasaportaciones y correcciones que M. Castañs ha llevado a cabo para superar los postulados y métodos de Palacios.

2. Cap. 2. “Estará” dedicado a exponer (sin demostrar) el método de los grupos de Lie a las odes. Esperoterminar de redactar estos dos primero capítulos durante el otoño de 2004. Todas las referencias de estecapítulo también son válidas para los capítulos 3 y 4.

Creo y “espero” que los capítulos interesantes (en su forma actual) son los capítulos 3 y 4.

3. Cap. 3. En este capítulo expongo algunas de las odes de primer orden (mediante ejemplos) y comparo suresolución por tres métodos:

El tradicional, en el que uno debe clasificar (saber de que tipo se trata, identificación previa) y luegoaplicar el cambio de variable pertinente (apropiado) i.e. un rollo pues por ejemplo con odes tipoRiccati además debemos buscar soluciones particulares, en fin lo de siempre.

El dimensional. Para aplicar este método lo primero que tenemos que hacer (comprobar) es que la odebajo estudio verifique el principio de homogeneidad dimensional i.e. de forma pedestre se puededecir que: peras = peras pero nunca peras = manzanas, para que se verifique la última igual deber-emos introducir ciertas constantes dimensionales que hagan que dicha ode verifique el principio dehomogeneidad i.e.

y′ = x 7−→ y′ = ax / [a] = x−2y

(ecuaciones con sentido físico).He seguido dos tácticas dimensionales:D1. (Método pedestre) Consiste en aplicar el AD (Teorema Pi) a la ode bajo estudio y obtener (intentarobtener) una solución. Esta solución por lo general será una solución particular de la ode dada peroademás será una solución invariante y atractiva desde el punto de vista de los sistemas dinámicosy con sentido físico (esto si es bastante importante). Esta táctica no siempre funciona y es bastantelimitada.D2. Consiste en obtener variables nuevas mediante el teorema Pi y reformular (reescribir) la ode dadaen estas nuevas variables, la ode resultante es trivialmente integrable, este es en esencia el método de

v

Page 8: Ecuaciones Diferenciales Mediante Simetrías · vi CONTENTS Lieperomuchísimomássencillodeaplicaryaqueunonodebeaplicarningúnalgoritmoparaobtener las simetrías que inducen dichos

vi CONTENTS

Lie pero muchísimo más sencillo de aplicar ya que uno no debe aplicar ningún algoritmo para obtenerlas simetrías que inducen dichos cambios de variable. Esta táctica siempre funciona.Con estas dos tácticas uno puede olvidarse de clasificaciones y del correspondiente cambio de variablequ eno es poca cosa.

El método de Lie. Consiste en buscar mediante un algoritmo (algoritmo de Lie) las posibles simetrías dela ode. Estas simetrías inducen cambios de variables que nos permiten reescribir la ode bajo estudioobteniendo así una ode que admite una integración trivial. El estudiar, aplicar, las odes de primerorden mediante este método es ir a matar moscas a cañonazos, es casi más complicado aplicar elmétodo (resolver la edp resultante al aplicar el algoritmo de Lie) que acordarse del pertinente cambiode variable, pero no en todos los casos, por ejemplo con las odes tipo Riccati nos evita el tener quebuscar una solución particular y en los casos más favorables nos permite encontrar una solución a lasodes tipo Abel o a las de tipo Chini.He intentado resaltar que la solución invariante que induce la simetría de escala coincide con laobtenida al aplicar el teorema Pi (nuestro método pedestre D1).

4. Cap. 4. En este capítulo repaso algunas de las odes de segundo orden, (tampoco está del todo terminado).Mediante ejemplos intento hacer ver como funcionan tanto el método de Lie (básicamente) como el di-mensional. Es aquí donde parece tener más utilidad el método de Lie. En el quinto capítulo estudio sóloalgunas ODEs de tercer orden.

En el último capítulo expongo unos ejemplos de aplicación del método de Lie a ecuacines de segundoorden que me han surgido en el estudio de modelos cosmológicos. Aquí la idea es la utilizar los GL yel AD no sólo para obtener soluciones a la ode dada sino para estudiar condiciones que deben cumplirlas funciones involucradas para que dicha ode sea integrable, por ejemplo expongo una ode de segundoorden en la que están involucradas 3 funciones, entonces el método de Lie nos dice que condiciones debencumplir para que dicha ode sea integrable. De igual forma nos ayuda a establecer ecuaciones de estado(casi nada).

Lamento de antemano todas las erratas, faltas de ortografía etc... y espero que estas notas puedan ser útiles paraalguien.

Page 9: Ecuaciones Diferenciales Mediante Simetrías · vi CONTENTS Lieperomuchísimomássencillodeaplicaryaqueunonodebeaplicarningúnalgoritmoparaobtener las simetrías que inducen dichos

CONTENTS vii

Page 10: Ecuaciones Diferenciales Mediante Simetrías · vi CONTENTS Lieperomuchísimomássencillodeaplicaryaqueunonodebeaplicarningúnalgoritmoparaobtener las simetrías que inducen dichos

viii CONTENTS

Page 11: Ecuaciones Diferenciales Mediante Simetrías · vi CONTENTS Lieperomuchísimomássencillodeaplicaryaqueunonodebeaplicarningúnalgoritmoparaobtener las simetrías que inducen dichos

Chapter 1

Dimensional Analysis

1.1 Assumptions in DA.

DA consider the following concepts:

(i) A quantity u is to be determined in terms of n measurable quantities (variables and constants, characteris-tic and universal) (Wi)

ni=1 ;

u = f (W1, ..., Wn) , (1.1)

where f is a unknown function of (Wi)ni=1 .

(ii) The quantities[u; (Wi)

ni=1

]are measured in term of m fundamental dimensions labeled as dimensional

base B = (

Lj

)m

j=1. We will put special emphasis in calculating the multiplicity of this dimensional basei.e. the number of fundamental dimensions. For example in the case in which we may take into accountthe spatial discrimination, see the section of examples.

(iii) With respect to this dimensional base a quantity Z has the following dimensional equation, i.e. has thefollowing dimensions with respect to this base, denoted as:

[Z] = Lα11 .....Lαm

m , (1.2)

where the numbers (αi)ni=1 , with αi ∈ R ∀i = 1, ..m, are the dimensional exponents of Z.

As we will see through the examples, the choice of the dimensional base will be essential in order to obtaingood results applying DA to diverse physical or mathematical problems.

(iv) We say that a quantity Z is dimensionless (with respect to B) written

[Z] = 1, (1.3)

iff (αi)ni=1 = 0, for all αi ∈ R ∀i = 1, ..m.

(v) Dimensional homogeneity of the equations under study. One we have fixed our dimensional base weneed to check if our equations verify the principle of dimensional homogeneity with respect to this di-mensional base, otherwise we will need to introduce the appropriate dimensional constants that makehomogeneous our equations under study.

(vi) For any set of fundamental dimensions, one can choose a system of units for measuring the value of anyquantity Z, for example the usual International System of units. A change from one system of units toanother involves a positive scaling of each fundamental dimension that in turn induces a scaling of eachquantity Z. Under a change of system of units, the value of a dimensionless quantity is unchanged i.e. itsvalue is invariant under an arbitrary scaling of any fundamental dimension. Therefore, it is meaningful

1

Page 12: Ecuaciones Diferenciales Mediante Simetrías · vi CONTENTS Lieperomuchísimomássencillodeaplicaryaqueunonodebeaplicarningúnalgoritmoparaobtener las simetrías que inducen dichos

2 CHAPTER 1. DIMENSIONAL ANALYSIS

to deem dimensionless quantities as large or small. The last assumption of D.A. is that formula (1.1) actsas a dimensionless equation in the sense that (1.1) is invariant under arbitrary scaling of any fundamentaldimension i.e. it is independent of the choice of a system of units.

Next, we see a particular example in order to clarify all these abstract ideas. For this purpose we may considerthe well known law of Newton

F = ma = md2x

dt2 , (1.4)

where we are interested in calculating the velocity, v, of some body, so v is the quantity to calculate. If we rewritethe above equation (1.4) as

Fm−1x−1t2 = 1, (1.5)

we may choose as dimensional base the following one B = x, m, t that we rewrite as

B = L, M, T , (1.6)

where L stands for length, M for mass and T for time, where the elements belonging to the dimensional baseare usually written in capital letters instead of small letters. Note that we have only one equation and fourquantities, hence the multiplicity of the dimensional base is: (number of quantities)−(number of equations), in factis the rank of the dimensional matrix (see below, the recipe, for clarifying this calculation). In the next chapterwe will clarify this point.

In this way we say that the dimensional equation of the force F is

[F] = LMT−2, (1.7)

while the dimensional equation of the quantity v is

[v] = LT−1. (1.8)

Other bases can be obtained, but I am not interested in exploring this fact. We only stress that it is very importantcalculate the appropriate base for each problem as we will see in the following sections (see section of examples).

Now we may check if our equation verifies the principle of dimensional homogeneity, in such a way that

[F] = [m] [a] , LMT−2 = M · LT−2, (1.9)

since [a] = LT−2.

This example is trivial, but we are only trying to clarify these new concepts.

1.2 The Recipe.

The above concepts are the main assumptions of the Pi-Theorem. Pi-Theorem allows us to calculate the relation-ship between the quantity u and the set of quantities (Wi)

ni=1, the set of variables and constants, characteristic

and universal. These assumptions bring us to the following conclusions:

1. Formula (1.1) can be expressed in term of dimensionless quantities.

2. The number of dimensionless quantities is

k + 1 = n + 1 − rg(B) (1.10)

where rg(B) is the rank of the dimensional matrix. This matrix is formed by the column vectors

bi =

b1i

b2i...

bmi

=⇒ B =

b11 b12 · · · b1n

b21 b22 · · · b2n...

......

...bm1 bm2 · · · bmn

(1.11)

Page 13: Ecuaciones Diferenciales Mediante Simetrías · vi CONTENTS Lieperomuchísimomássencillodeaplicaryaqueunonodebeaplicarningúnalgoritmoparaobtener las simetrías que inducen dichos

1.3. THE PI-THEOREM. 3

where bi is the dimension vector of Wi, i = 1, ..., n.

Precisely k of these dimensionless quantities depend on the measurable quantities (Wi)ni=1.

3. Let

x(i) =

x1i

x2i...

xmi

, (1.12)

with i = 1, ..., k, represent the k = n − rg(B) linearly independent solutions x of the system Bx = 0. Let

a =

a1a2...

am

, (1.13)

be the dimension vector of the quantity u, and let

y =

y1y2...

yn

, (1.14)

represent a solution of the system By = −a. Then formula (1.1) simplifies to

π = g (π1, ...., πk) , (1.15)

where(

π, (πi)ki=1

)are dimensionless quantities given by

π = uWy11 W

y22 ....Wyn

n , πi = Wx1i1 W

x2i2 ....Wxni

n , (1.16)

with i = 1, ...k, and g is some unknown function of its arguments. In particular eq. (1.1) becomes

u = W−y11 W

−y22 ....W−yn

n g (π1, ...., πk) . (1.17)

1.3 The Pi-Theorem.

Now we will give a brief sketch of the proof of this famous theorem. First of all

[u] = La11 .....Lam

m , [Wi] = Lb1i1 .....Lbmi

m , (1.18)

with i = 1, ...n. Next, we use assumption (vi) and consider the invariance of eq. (1.1) under arbitrary scaling ofthe fundamental dimensions by taking each fundamental dimensions in turn. We first scale L1 by letting

L∗1 = eεL1, ε ∈ R, (1.19)

in turn this induces the following scalings of the measurable quantities

u∗ = eεa1 u, W∗i = eεb1i Wi, i = 1, ...n. (1.20)

Equations (1.20) define a one-parameter (ε) Lie group of scaling transformations of the n + 1 quantities (u, W1, W2, ..., Wn)with ε = 0 corresponding to the identity transformation. This group is induced by the one-parameter group ofscalings (1.19) of the fundamental dimension L1.

Page 14: Ecuaciones Diferenciales Mediante Simetrías · vi CONTENTS Lieperomuchísimomássencillodeaplicaryaqueunonodebeaplicarningúnalgoritmoparaobtener las simetrías que inducen dichos

4 CHAPTER 1. DIMENSIONAL ANALYSIS

From the assumption (vi), eq. (1.1) holds iff

u∗ = f (W∗1 W∗

2 .....W∗n ) , (1.21)

i.e.eεa1 u = f

((eεb1i Wi

)n

i=1

), ∀ε ∈ R, (1.22)

then two cases need to be distinguished:

Case I. b11 = .... = b1n = 0 = a1. Here L1, is not a fundamental dimension of the problem, i.e. eq. (1.1) isdimensionless with respect to L1.

Case II. b11 = .... = b1n = 0, and a1 6= 0. Hence u = 0 is a trivial solution.

Therefore it follows that ∃ b1i 6= 0 for some i = 1, ...n. Without loss of generality we assume b11 6= 0. We definenew measurable quantities

Xi−1 = WiW−b1i/b111 , i = 2, 3, ...n, (1.23)

and letXn = W1. (1.24)

We choose as the new unknownv = uW−a1/b11

1 , (1.25)

the transformation given by eqs. (1.23)-(1.25) defines a one-to-one mapping of the quantities (Wi)ni=1 7−→

(Xi)ni=1 , and a one-to-one mapping of the quantities

[u, (Wi)

ni=1

]7−→

[v, (Xi)

ni=1

], in this way eq. (1.1) is equiv-

alent tov = F

((Xi)

ni=1

), (1.26)

where F is a unknown function of (Xi)ni=1 . Thus the group of transformations (1.20) becomes

v∗ = v, (1.27)

X∗i = Xi, i = 1, 2, ....n − 1, (1.28)

X∗n = eεb11 Xn, (1.29)

therefore[v, (Xi)

n−1i=1

]are invariants of (1.20). Moreover, the quantities

[v, (Xi)

ni=1

]satisfy assumptions (iii), and

eq. (1.26) satisfies assumption (vi). Therefore

v = F((Xi)

n−1i=1 , eεb11 Xn

), ∀ε ∈ R. (1.30)

where F is independent of the quantity Xn. Moreover, the measurable quantities (Xi)n−1i=1 are products of powers

of (Wi)ni=1 and v is a product of u and powers of (Wi)

ni=1 . Thus eq. (1.1) reduces to

v = G((Xi)

n−1i=1

), (1.31)

where[v; (Xi)

n−1i=1

]are dimensionless with respect to L1 and G is some function of (n − 1) arguments.

If we repeat this tactic with the (m − 1) fundamental dimensions, we reduce eq. (1.1) to a dimensionless formula

π = g((πi)

ki=1

), (1.32)

where [π] = [πi] = 1, with i = 1, ..., k, and g is some function of (πi)ki=1 , being

π = un

∏i=1

Wyii , πi =

k

∏j=1

Wxmjm , (1.33)

with m = 1, ..., n.

Page 15: Ecuaciones Diferenciales Mediante Simetrías · vi CONTENTS Lieperomuchísimomássencillodeaplicaryaqueunonodebeaplicarningúnalgoritmoparaobtener las simetrías que inducen dichos

1.4. EXAMPLES. 5

Note that this theorem makes no assumptions about the continuity of the function f and hence of g with respectof any of their arguments.

It is shown immediately that the number of measurable dimensionless quantities is k = n − rg(B). This followssince [

Wx11 Wx2

2 ....Wxnn

]= 1, (1.34)

iff

x =

x1x2...

xn

, (1.35)

satisfies Bx = 0. This equation has k = n − rg(B) linearly independent solutions x(i) given by (1.12). The realnumbers (1.14) follow from setting [

uWy11 W

y22 ....Wyn

n

]= 1, (1.36)

leading to y satisfying By = −a.

1.4 Examples.

In this section we will try to explain how works DA through different examples and how we can avoid the caseswhich we obtain a result with an unknown function g. For this purpose we will follow two different tactics,the first of them will consist in making a mathematical hypothesis on the behaviour of this unknown functionand we will justify (as far as possible) such assumption. In the second of our tactics we will use the so calleddimensional discrimination. Such tactic allows us to obtain a complete solution for our problems and it is notneeded to make any previous assumption.

Ejemplo 1.4.1 The famous atomic explosion of 1945.

Solution. We are interested in knowing the radius R of the shock wave that is produced an atomic explosion.For this purpose we treat R as the unknown (u = R with the previous notation) and assume that

R = f (W1, W2, W3, W4), (1.37)

where W1 = E is the energy released by the explosion, W2 = ρ0 is the ambient air density, W3 = P0 is the ambientair pressure, W4 = t is the elapsed time after the explosion takes place. For this problem we use B = L, M, Tas dimensional base. therefore the corresponding dimensional matrix is given by

E t P0 ρ0L 2 0 −1 −3M 1 0 1 1T −2 1 −2 0

, / B =

2 0 −1 −31 0 1 1−2 1 −2 0

(1.38)

since for example [E] = L2MT−2, etc... and as we can see the rg(B) = 3 therefore k = n − rg(B) = 4 − 3 = 1 isthe number of π − monomia.

Following the above explained procedure we find that Bx = 0 i.e.

2 0 −1 −31 0 1 1−2 1 −2 0

x1x2x3x4

= 0, =⇒

x1 = − 25 x4

x2 = 65 x4

x3 = − 35 x4

, (1.39)

Page 16: Ecuaciones Diferenciales Mediante Simetrías · vi CONTENTS Lieperomuchísimomássencillodeaplicaryaqueunonodebeaplicarningúnalgoritmoparaobtener las simetrías que inducen dichos

6 CHAPTER 1. DIMENSIONAL ANALYSIS

where x4 is arbitrary. Setting x4 = 1 we obtain the measurable dimensionless quantity

π1 = P0

(t6

E2ρ30

)1/5

. (1.40)

Now the dimension vector of R is

a =

100

, [R] = L, (1.41)

the general solution of By = −a is

y =15

−1−210

+ x, (1.42)

where x is the general solution of Bx = 0. Setting x = 0, we obtain the dimensionless unknown

π = R

(Et2

ρ0

)−1/5

, (1.43)

therefore the solution that DA gives us is the following one

R =

(Et2

ρ0

)1/5

g(π1), (1.44)

where g is some function of π1.

How to solve the big drawback of having an unknown function g?

We may make the following assumption (called the power law hypothesis, from now on)

π = (π1)n, n ∈ R, (1.45)

i.e. we are supposing that the general solution follows a power law and we fix the value of the numericalconstant n through experiment. Therefore the solution that we suggest take the following form

R =

(Et2

ρ0

)1/5P0

(t6

E2ρ30

)1/5

n

= E1−2n

5 t2+6n

5 ρ3n−1

50 Pn

0 , (1.46)

basically we are assuming that

R ≈ Ct2+6n

5 near t = 0. (1.47)

where C is a constant.

Numerical values from the experiments (i.e. after many nuclear explosions) bring us to obtain n = 0, so thesolution is

R =

(E

ρ0

) 15

t25 =⇒ R = At2/5. (1.48)

Observación 1.4.1 The short cut. The problem is easily reduced to:

R = f (E, t, P0, ρ0) , B = L, M, T (1.49)

where the dimensional matrix is given by

R E t P0 ρ0L 1 2 0 −1 −3M 0 1 0 1 1T 0 −2 1 −2 0

. (1.50)

Page 17: Ecuaciones Diferenciales Mediante Simetrías · vi CONTENTS Lieperomuchísimomássencillodeaplicaryaqueunonodebeaplicarningúnalgoritmoparaobtener las simetrías que inducen dichos

1.4. EXAMPLES. 7

Since the number of quantities is 5 and the multiplicity of the dimensional base is 3 therefore we will obtain 2 π −monomials, i.e. a solution like π = g(π1) where π = f (R; E, t, ρ0) and π1 = f1 (E, t, P0, ρ0).

From the dimensional matrix (1.50) we see that

− 1 + 2αE − αP0 − 3αρ0= 0, (1.51)

αE + αP0 + αρ0= 0, (1.52)

−2αE + αt − 2αP0 = 0 (1.53)

therefore the solution under the assumption π = (π1)n is

R = EαE tαt ραρ00 P

αP00 , i.e. R = E

1−2n5 t

2+6n5 ρ

3n−15

0 Pn0 (1.54)

as we can deduce in a trivial way from the above system of linear equations (1.51)-(1.52).

We need to avoid this kind of assumptions, although they work well, or at least to try to justify mathematically why we mayexpect this class of solutions i.e. power law solutions.

Ejemplo 1.4.2 The range of a projectile.

A projectile of mass m weight W and with initial velocity v, is launched in horizontal direction from a height h. We areinterested in knowing the range x of the projectile.

Solution. In this case we will explore two possibilities, in the first of them the application of DA brings us toobtain an unsatisfactory solution since this depends on an unknown function. To solve this drawback we exploreanother possibility which consists in enlarge the dimensional base taking into account the spacial discrimination.

1. Briefly the problem is reduced to:x = f (m, W, v, h) , (1.55)

that with respect to the dimensional base B = L, M, T it is obtained the following dimensional matrix

x m W v hL 1 0 1 1 1M 0 1 1 0 0T 0 0 −2 −1 0

, =⇒ π =x

h, π1 =

mv2

hW(1.56)

therefore the solution will be

π = ϕ(π2), i.e. x = hϕ

(mv2

hW

). (1.57)

As we have already commented this solution is very poor for a very simple problem, for this reason wewill try another tactic.

2. As we can see there are two privileged spatial directions in this problem(

Lx, Ly

). For this reason, under

this new consideration, the problem is reduced to (as above):

x = f (m, W, v, h) , (1.58)

but with respect to the dimensional base B =

Lx, Ly, M, T

it is obtained the following dimensionalmatrix

x m W v hLx 1 0 0 1 0Ly 0 0 1 0 1M 0 1 1 0 0T 0 0 −2 −1 0

, (1.59)

Page 18: Ecuaciones Diferenciales Mediante Simetrías · vi CONTENTS Lieperomuchísimomássencillodeaplicaryaqueunonodebeaplicarningúnalgoritmoparaobtener las simetrías que inducen dichos

8 CHAPTER 1. DIMENSIONAL ANALYSIS

which bring us to obtain only one π − monomia. Therefore the solution is in this case

x = v

√hm

W= v

√h

g, (1.60)

as it is expected.

With this example we have tried to show another tactic to solve physical problem and without the necessity ofdoing assumptions on the behaviour of the solution as in the previous example.

Ejemplo 1.4.3 The golfer and the moral. We are interested in knowing how long, t, must train a golfer of mass m andweight W, in order to hit a golf-ball to a distance l.

Solution. As in the above example we may see that the set of governing parameters is

P = P (t; m, W, l) , (1.61)

that with respect to the dimensional base B = L, M, T, we obtain the following the dimensional matrix

t m W lL 0 0 1 1M 0 1 1 0T 1 0 −2 0

, =⇒ t = C

√l

g, (1.62)

from which we obtain a single monomial, as it is observed in a trivial way. C ∈ R.

Observación 1.4.2 Obviously this solution is a complete nonsense since, from our point of view, it is necessary that theproblem must be considered within some physical theory (and this is not the case). DA is subordinate to one physicaltheory and not the other way round as we may deduce from this simple example although some times one may found someinteresting relationship between the quantities i.e. with physical meaning.

1.5 Applications of dimensional analysis to ODEs and PDEs

In this section we will try to show how to apply D.A. to solve differential equations and partial differentialequations.

1.5.1 Differential equations.

The method of the Lie groups it has showed as a very useful tool in order to solve nonlinear equations (nl-ODE)as well as PDE. Nevertheless, when one is studying ode of first order its application becomes very complicated(very tedious) if one has not a computer algebra package since if one decides to look for the possible symmetriesof an ODE with pencil and paper this task may be turned very exhausting. However, if we know that an ODEadmits a concrete symmetry, then it is a trivial issue to find new variables which allows us to rewrite the ODE inquadratures or as we will see in this paper to obtain an ode with separating variables.

Page 19: Ecuaciones Diferenciales Mediante Simetrías · vi CONTENTS Lieperomuchísimomássencillodeaplicaryaqueunonodebeaplicarningúnalgoritmoparaobtener las simetrías que inducen dichos

1.5. APPLICATIONS OF DIMENSIONAL ANALYSIS TO ODES AND PDES 9

Our purpose in this work is to explain, through an example, how D.A. works in order to find these changesof variables (c.v.) in a trivial way, i.e. without the knowledge of the symmetries of the ODE under study. Theidea is as follows. When we are studying an ODE from the dimensional point of view, we must require thatsuch ODE verifies the principle of dimensional homogeneity (pdh) i.e. that each term within the equation havethe same dimensions, for example, speed, or energy density. To clarify this concept we consider the following

ode, y′ =y

x+ x, where each term must have dimensions of y′ i.e. [y′] = yx−1, where [·] stands for dimensional

equation of the quantity ·. As we can see, this ODE does not verify the pdh, since the term x has dimensions ofx, i.e. [x] = x. In order to do that this equation verifies the pdh we need to introduce dimensional constants,in such a way that after rewrite the ODE with these constants the ODE now verifies such principle i.e. in thisexample and as we can see easily if we consider only one constant a, such that [a] = yx−2, we make that the

ODE y′ =y

x+ ax, verifies the pdh. We would like to emphasize that this situation does not arise when one is

studying physical or engineering problems since, as it is supposed, such problems (equations) verify the pdhand we do not need to introduce new dimensional constants, that must have physical meaning, for example, theviscosity coefficient, etc....

Precisely this dimensional constant suggests us the c.v. (x, y(x)) 7−→ (t, u(t)) where(t = x, u = yx−2) in such

a way that rewriting the original ODE in these new variables yields a new ODE with separating variables:u + u′t = 1. The reason is the following. We know from the Lie group theory that if it is known a symmetryof an ODE then this symmetry bring us through a c.v. to obtain a simpler ODE (a quadrature or a ODE withseparating variables) and therefore the solution is found in a closed form. To find this c.v. one uses the invariantsthat generate each symmetry, i.e. the first principle is that it is useful to pass to new coordinates such that oneof the coordinate functions is an invariant of the group. After such transformation it often (but not always)happens that the variables separate and the equation can be solved in closed form. We must stress that takingthe new dependent variables to be an invariant of the group does not guarantee the separation of variables. Thechoice of the independent variable is also very important.

One must note that D.A. gives us this invariant (or at least a particular solution). It is observed in our examplethat the solution y = x2, (suggested by D.A.) is a particular solution for this ODE, but if we study this ode formthe Lie theory point of view it is obtained that such ODE has the scaling symmetry X = x∂x + 2y∂y and thereforethe solution y = x2, is furthermore an invariant solution (generated by X), for this reason such c.v. works well.As we will show in the following example, the c.v. that D.A. induces is the same as the one generated by thesymmetries of the ODE. In order to make see this fact, we will solve the example by D.A. as well as by the Liegroup technique, calculating the symmetries and their corresponding invariants and c.v..

Ejemplo 1.5.1 Solve the Abel ODE.

y′ = Cx3y3 + Bxy2 − Ay

x, (1.63)

Solution. If we rewrite eq. (3.256) introducing the following dimensional constants,

y′ = a2Cx3y3 + aBxy2 − Ay

x, (1.64)

where A, B, C ∈ R, and [a] = X−2Y−1. As we can see this ODE is scale invariant since we have needed tointroduce only one constant. DA suggests us the following c.v.

(t = x, u(t) = ax2y

)=⇒

(x = t, y =

u

at2

), (1.65)

in such a way that eq. (3.256) is written in the following form:

tu′ = u(

u2 + u + 1)

, (1.66)

and its solution is:

ln t +12

ln(

u2 + u + 1)

+

√3

3arctan

((32

u +13

)√3)− ln u + C1 = 0, (1.67)

Page 20: Ecuaciones Diferenciales Mediante Simetrías · vi CONTENTS Lieperomuchísimomássencillodeaplicaryaqueunonodebeaplicarningúnalgoritmoparaobtener las simetrías que inducen dichos

10 CHAPTER 1. DIMENSIONAL ANALYSIS

therefore in the original variables it yields:

ln x +12

ln((

ax2y)2

+ ax2y + 1)

+

√3

3arctan

((32

(ax2y

)+

13

)√3)− ln

(ax2y

)+ C1 = 0. (1.68)

In second place, we study eq. (3.256)

y′ = Cx3y3 + Bxy2 − Ay

x, (1.69)

with respect to the dimensional base B = T . This ODE verifies the principle of dimensional homogeneity with

respect to this dimensional base. Note that [y] =[

1H′

]= T2, and [x] = [H] = T−1 hence [y′] = T3. Therefore

rewriting the equation in a dimensionless way we find that y ∝ x−2. This ODE has been obtained studyinga cosmological model and therefore H is the usual Hubble parameter (see the next chapter to understand themeaning of this quantity) and therefore [H] = T−1 as it is usual and T stands for dimensions of time.

But if we study this equation with respect to the dimensional base B = X, Y , we need to introduce newdimensional constants that make the equation verify the principle of dimensional homogeneity

y′ = αCx3y3 + βBxy2 − Ay

x, (1.70)

where[α1/2

]= [β] = X−2Y−1, hence

y β xX 0 −2 1Y 1 −1 0

=⇒ y ∝1

βx2 , (1.71)

which is a particular solution of the original ode.

Lie Method. In order to find the symmetry generator, X = ξ(x, y)∂x + η(x, y)∂y, of a ODE

y′ = f (x, y) , (1.72)

we need to solve the following PDE

ηx + (ηy − ξx) f − ξy f 2 − ξ(x, y) fx − η(x, y) fy = 0, (1.73)

where

fx =d f

dx, fy =

d f

dy. (1.74)

see the appendix in order to clarify notation and concepts.

In this case we have to solve

ηx + (ηy − ξx)(

Cx3y3 + Bxy2 − Ay

x

)− ξy

(Cx3y3 + Bxy2 − A

y

x

)2−

− ξ(x, y)(

3Cx2y3 + By2 + Ay

x2

)− η(x, y)

(3Cx3y2 + Bxy − A

x

)= 0, (1.75)

This ODE admits the following symmetryX = x∂x − 2y∂y, (1.76)

which is a scaling symmetry and it induces the following change of variables,

Xr = 0, Xs = 1, (1.77)

r = x2y, s(r) = ln(x), =⇒ x = es(r), y =r

e2s(r), (1.78)

Page 21: Ecuaciones Diferenciales Mediante Simetrías · vi CONTENTS Lieperomuchísimomássencillodeaplicaryaqueunonodebeaplicarningúnalgoritmoparaobtener las simetrías que inducen dichos

1.5. APPLICATIONS OF DIMENSIONAL ANALYSIS TO ODES AND PDES 11

which brings us to obtain the next ode in quadratures

s′ =1

r (Cr2 + Br + 2 − A), (1.79)

and whose solution is:

s(r) = − ln r

A − 2+

12

ln(Cr2 + Br + 2 − A

)

A − 2−

Barctanh(

2Cr+B√B2+4C(A−2)

)

(A − 2)√

B2 + 4C(A − 2)+ C1, (1.80)

and hence in the original variables (x, y):

ln x = − ln(

x2y)

A − 2+

12

ln(Cx4y2 + Bx2y + 2 − A

)

A − 2−

Barctanh(

2Cx2y+B√B2+4C(A−2)

)

(A − 2)√

B2 + 4C(A − 2)+ C1, (1.81)

which is the most general solution for this ODE.

Now if we consider the invariants I that induce the symmetry X i.e.

dx

ξ=

dy

η, 7−→ y′ :=

dy

dx=

η

ξ, (1.82)

Hence, for example the symmetry X = x∂x − 2y∂y generates the following invariant:

dx

ξ=

dy

η=⇒ dx

x= −dy

2y=⇒ ln x = −1

2ln y =⇒ I = x2y 7−→ y = ax−2, (1.83)

this would be the solution that suggests us precisely the direct use of the Pi theorem (if the ODE is scale invariant,as in this case, the solution obtained applying by the Pi theorem coincides with the invariant solution thatgenerates the scale symmetry, in this case X). We see that we only obtain a particular solution, but that this isinvariant, in fact if we think about the ODE as a dynamical system we see that the fixed point of such equationwould be precisely the solution y = Kx−2.

Observación 1.5.1 Usually the most general solution of this kind of ODEs is unphysical i.e. it lacks of physical meaning.So, why the invariant solution has physical meaning and is stable from the dynamical point of view?

1.5.2 Partial differential equations.

Using the notation of the section 2, now we consider a boundary value problem for a partial differential equation(PDE)

(u, (Wi)

ni=1

)where u is the unknown function (the dependent variable of the PDE) and (Wi)

ni=1 are the

independent variables and constants. From the Pi-theorem it follows that the PDE may be re-expressed in

dimensionless form(

π, (πi)ki=1

), being π the dimensionless dependent variable and (πi)

ki=1 the dimensionless

independent variables and dimensionless constants.

Considering the set of quantities (Wi)ni=1 we distinguish between (Wi)

li=1 variables and (Wi)

ni=l+1 constants. Let

B be the dimensional matrix such thatB = (B1 | B2) , (1.84)

of all (Wi)ni=1 and B1 = M((Wi)

li=1) the dimensional matrix of the quantities and B2 = M((Wi)

ni=l+1) the

dimensional matrix of the constants.

Page 22: Ecuaciones Diferenciales Mediante Simetrías · vi CONTENTS Lieperomuchísimomássencillodeaplicaryaqueunonodebeaplicarningúnalgoritmoparaobtener las simetrías que inducen dichos

12 CHAPTER 1. DIMENSIONAL ANALYSIS

An important objective in applying D.A. to PDE is to reduce the number of independent variables. The rankof B2 i.e. r(B2) represents the reduction in the number of constants through DA. Consequently the reduction inthe number of independent variables is ρ = r(B) − r(B2). In particular the number of dimensionless measur-able quantities is k = n − r(B) = [(l − ρ) + (n − l − r(B2))] , where (l − ρ) of the quantities are dimensionlessindependent variables and (n − l − r(B2)) are dimensionless constants.

If ρ = r(B) − r(B2) = 0, then DA reduces the given boundary value problem to a dimensionless boundaryproblem with (n − l − r(B2)) dimensionless constants. In this case the number of independent variables is notreduced. If l ≥ 2, (l − ρ) = 1, then the resulting solution of the boundary value problem is called self-similar.

Ejemplo 1.5.2 Stokes first problem. Let us consider a motionless fluid in the proximity of a plane wall. If the wall startsmoving abruptly along its own plane (starting current) with a constant speed U0, the Navier-Stokes equation reduce to:

∂u

∂t= ν

∂2u

∂y2 . (1.85)

Find the velocity distribution of the fluid as function of y, the distance to the plane as well as function of, t :i.e.u = u (y, t)

Solution. Let u be the fluid velocity in the direction of U0 (x−direction), in this case the Navier-Stokes equationsreduce to the viscous diffusion equation

∂u

∂t= ν

∂2u

∂y2 , (1.86)

with the boundary conditions:

u(y, 0) = 0, ∀y > 0, (1.87)

u(∞, t) = 0, and u(0, t) = U0, t > 0 (1.88)

where ν is the “kinematic viscosity”.

We are going to explore two dimensional tactics, the first of them is the usual one while the second will consistin the spatial discrimination.

1. In this case the dimensional base isB = L, T and therefore we obtain the following dimensional matrix:

u U0 y t νL 1 1 1 0 2T −1 −1 0 1 −1

(1.89)

obtaining in this way 3 monomials and therefore we do not reduce the number of variables in the problem.We have insisted in outlining this tactic in order to show the useful of the spatial discrimination as follows.

2. In this case the dimensional base is B =

Lx, Ly, T

and therefore the new dimensional equations of eachquantity are:

[ν] = L2yT−1, [u] = [U0] = LxT−1, [y] = Ly, [t] = T (1.90)

The most general solution is:f (y, t, u, U0, ν) = 0, (1.91)

we have the following dimensional matrix:

u U0 y t νLx 1 1 0 0 0Ly 0 0 1 0 2T 0 0 0 1 −1

, (1.92)

Page 23: Ecuaciones Diferenciales Mediante Simetrías · vi CONTENTS Lieperomuchísimomássencillodeaplicaryaqueunonodebeaplicarningúnalgoritmoparaobtener las simetrías que inducen dichos

1.5. APPLICATIONS OF DIMENSIONAL ANALYSIS TO ODES AND PDES 13

obtaining two π − monomia :

π =u

U0, π1 =

y√ν t

, (1.93)

then, new variables η and U are defined as

η =y√ν t

, U (η) =u

U0, i.e. u = U0 · U (η) , (1.94)

Now if we calculate ∂ u∂ t and ∂2u

∂ y2 as functions of U and η, we obtain

d2U

dη2 +η

2dU

dη= 0 (1.95)

with the boundary conditions:U(0) = 1, U(∞) = 0, (1.96)

where U = 0 for t = 0, y > 0 (as, η → ∞) and U = 1 for t → ∞, y ≥ 0 (as, η = 0). Therefore the solution is:

U = erf(η

2

). (1.97)

For example, the change of variablesU′ = z, η = s, (1.98)

brings us to obtain the following ode

z′ = −12

sz =⇒ z = C1e−s2/4, (1.99)

and making the inverse change of variables

U′ = C1e−η2/4, =⇒ U = C1

∫e−η2/4dx = erf

2

), (1.100)

it is obtained the solution (1.97).

In this way we can see how works the dimensional discrimination, showing us that is it a powerful tool to solvepartial differential equations.

Page 24: Ecuaciones Diferenciales Mediante Simetrías · vi CONTENTS Lieperomuchísimomássencillodeaplicaryaqueunonodebeaplicarningúnalgoritmoparaobtener las simetrías que inducen dichos

14 CHAPTER 1. DIMENSIONAL ANALYSIS

Page 25: Ecuaciones Diferenciales Mediante Simetrías · vi CONTENTS Lieperomuchísimomássencillodeaplicaryaqueunonodebeaplicarningúnalgoritmoparaobtener las simetrías que inducen dichos

Chapter 2

Introduction to the symmetry methods.

What are the symmetries of an ODE?. Consider the simplest example

y′ =dy

dx= 0, (2.1)

whose solution is obviously the set of straight lines y = c, with c ∈ R. The ODE (2.1) is represented geometricallyby the set of all solutions and so any symmetry of the ODE must necessarily map the solution set into it self.Formally, the condition that any symmetry maps the object into itself requires that the set of solutions curves inthe (x, y)−plane must be indistinguishable from the image in the (x, y)−plane, so

y′ = 0 when y′ = 0. (2.2)

A smooth transformation of the plane is invertible if its Jacobian is nonzero so we impose the further condition

xx yy − xyyx 6= 0. (2.3)

A particular solution curve will be mapped to a (possibly different) solution curve, and so

y (x, c) = c(c), ∀c ∈ R. (2.4)

Here x is regarded as a function of x and c that is obtained by inverting x = x(x, c). ODE (2.1) is very simple andso all of its symmetries can be found. Differentiating (2.4) with respect to x, we obtain yx(x, c) = 0, ∀c ∈ R,therefore taking (2.3) into account, the symmetries of (2.1) are of the form

(x, y) = ( f (x, y), g(y)) , fx 6= 0, gy 6= 0. (2.5)

where f and g are assumed to be smooth functions of their arguments. The ODE has very large family ofsymmetries. We were able to use the know general solution of (2.1) to derive (2.2) which led to (2.5). However,we could also have obtained this result directly from (2.2) as follows.

On the solution curves, y is a function of x, and hence x = (x, y) and y = (x, y) may be regarded as a function ofx. Then by the chain rule (2.2) can be written as

dy

dx=

Dx y

Dx x= 0, when y′ = 0, (2.6)

where Dx denotes the total derivative with respect to x, i.e. Dx = ∂x + y′∂y + y′′∂y′ + ..., therefore (2.2) amountsto

yx + y′yy

xx + y′ xy= 0, when y′ = 0, (2.7)

and therefore (2.5) holds.

15

Page 26: Ecuaciones Diferenciales Mediante Simetrías · vi CONTENTS Lieperomuchísimomássencillodeaplicaryaqueunonodebeaplicarningúnalgoritmoparaobtener las simetrías que inducen dichos

16 CHAPTER 2. INTRODUCTION TO THE SYMMETRY METHODS.

The advantage of using the symmetry condition in the form (2.2) is that one can obtain information about thesymmetries without having to know the solution of the differential equation in advance.

For an ODE

y′ = ω(x, y), (2.8)

the symmetry condition readsy′ = ω(x, y), when y′ = ω(x, y). (2.9)

As before, we regard y as function of x on the solution curves, therefore (2.9) yields

Dx y

Dx x=

yx + y′yy

xx + y′ xy= ω(x, y), when y′ = ω(x, y), (2.10)

therefore the symmetry condition for (2.8) is equivalent to the constraint

yx + ω(x, y)yy

xx + ω(x, y)xy= ω(x, y), (2.11)

together with the requirement that the mapping should be a diffeomorphism.

Now, suppose that we are able to find a nontrivial one-parameter Lie group (LG) of symmetries of the ODE (2.8)then the Lie group can be used to determine the general solution of the ODE. “Suppose” that the symmetry of(2.8) include the Lie group of transformations in the y-direction i.e. (0, 1)

(x, y) = (x, y + ε) , (2.12)

then, the symmetry condition (2.11) reduces to

ω(x, y) = ω (x, y + ε) , (2.13)

for a very small values of ε. Differentiating (2.13) with respect to ε at ε = 0, ωy(x, y) = 0, and therefore the mostgeneral ODE whose symmetries include the Lie group (2.12) is of the form

y′ = ω(x), =⇒ y =∫

ω(x)dx + c, (2.14)

the articular solution corresponding to c = 0 is mapped by the translation to the solutions

y =∫

ω(x)dx + ε =∫

ω(x)dx + ε, (2.15)

which is the solution corresponding to c = ε. So by using the one-parameter LG we obtain the general solutionfrom one particular solution.

Is the same true for ODEs with other one-parameter LG?. Yes, the same idea works for all one-parameter LG.In a suitable coordinate system, the symmetries parametrized by ε sufficiently close to zero are equivalent totranslations.

How can find symmetries of an ODE?. One method is to use the symmetry condition (2.11). In general this is acomplicated nonlinear PDE in (x, y) . However, Lie symmetries can be derived from a much simpler conditionon the tangent vector field. By definition, the Lie symmetries of an ODE are of the form

x = x + εξ + O(ε2), y = y + εη + O(ε2), (2.16)

if we substitute (2.16) into (2.11) we obtain

ω(x, y) + ε(

ηx + ω(x, y)ηy

)+ O(ε2)

1 + ε(

ξx + ω(x, y)ξy

)+ O(ε2)

= ω(

x + εξ + O(ε2), y + εη + O(ε2))

, (2.17)

Page 27: Ecuaciones Diferenciales Mediante Simetrías · vi CONTENTS Lieperomuchísimomássencillodeaplicaryaqueunonodebeaplicarningúnalgoritmoparaobtener las simetrías que inducen dichos

17

we now expand each side of (2.17) as a Taylor series about ε = 0, assuming that each series converges

ω + ε(

ηx + ω(ηy − ξx

)− ξyω2) + O(ε2) = ω + ε

(ξωx + ηωy

)+ O(ε2), (2.18)

where ω is shorthand for ω(x, y). Equating the O(ε) terms gives the linearized symmetry condition

ηx + ω(ηy − ξx) − ξyω2 = ξωx + ηωy. (2.19)

Canonical Coordinates. We know that every ODE whose symmetries include the translations (2.12) may beintegrated directly. How can these coordinates be found?. All orbits of the symmetry (2.12) have the sametangent vector at every point (ξ, η) = (0, 1) . Given any one-parameter LG, we aim to introduce coordinatessuch that (r, s) = (r(x, y), s(x, y)) , where (r, s) = (r, s + ε). If this is possible then, in the new coordinates, thetangent vector at the point (r, s) is (0, 1) i.e.

dr

dε |ε=0= 0,

ds

dε |ε=0= 1, (2.20)

using the chain rule and (2.16) we get

ξrx + ηry = 0, ξsx + ηsy = 1, (2.21)

i.e. Xr = 0, Xs = 1. Since the change of coordinates should be invertible in some neighborhood of (x, y) then weimpose the condition (2.3).

Suppose that we have been able to find a nontrivial Lie symmetry (LS) of a given ODE (2.8) then this ODE canbe reduce to a quadrature by rewriting it in terms of canonical coordinates as follows:

ds

dr=

sx + ω (x, y) sy

rx + ω (x, y) ry, (2.22)

the right-hand side of (2.22) can be written as a function of r and s. For a general change of variables (x, y) 7→(r, s) the transformed ODE would be of the form

ds

dr= Ω (r, s) , (2.23)

for some function Ω. However, (r, s) are canonical coordinates and so the ODE is invariant under the group thetranslations in the s-direction: (r, s) = (r, s + ε) . Therefore, as we already know, ODE (2.23) is of the from

ds

dr= Ω (r) , (2.24)

the problem is now reduced to a quadrature. The general solution of (2.24) is

s −∫

Ω (r) dr = c, c ∈ R, (2.25)

and hence the general solution of (2.8) is

s(x, y) −∫ r(x,y)

Ω (r) dr = c. (2.26)

In general, for an ODE

y(n) = ω(x, y, y′, ....., y(n−1)), y(k) =dky

dxk, (2.27)

where ω is a smooth function of its arguments. We begin imposing the symmetry condition. A symmetry of(2.27) is a diffeomorphism. that maps the set of solutions of the ODE to itself. Any diffeomorphism. Γ : (x, y) 7→(x, y) maps smooth planar curves to smooth planar curves. This action of Γ on the plane induces an action on the

Page 28: Ecuaciones Diferenciales Mediante Simetrías · vi CONTENTS Lieperomuchísimomássencillodeaplicaryaqueunonodebeaplicarningúnalgoritmoparaobtener las simetrías que inducen dichos

18 CHAPTER 2. INTRODUCTION TO THE SYMMETRY METHODS.

derivatives y(k) which is the mapping Γ : ω(x, y, y′, ....., y(n−1)) 7→ ω(x, y, y′, ....., y(n−1)). This mapping is calledthe nth prolongation of Γ. The functions y(k) are calculated recursively as follows:

y(k) =dy(k−1)

dx=

Dx y(k−1)

Dx x, y(0) ≡ y, (2.28)

where Dx is the total derivative. The symmetry condition for the ODE (2.27) is

y(n) = ω(x, y, y′, ....., y(n−1)), when (2.27) holds. (2.29)

The LS are obtained linearizing (2.29) about ε = 0. The linearized symmetry condition is derived as above. For εsufficiently close to zero, the prolonged LS are of the form:

x = x + εξ + O(ε2), y = y + εη + O(ε2), y(k) = y(k) + εη(k) + O(ε2), (2.30)

if we substitute (2.30) into the symmetry condition (2.29) the O(ε) terms yield the linearized symmetry condition

η(k) = ξωx + ηωy + η(1)ωy′ + ..... + η(n−1)ωy(n−1) , when (2.27) holds. (2.31)

The functions η(k) are calculated recursively from (2.28) as follows. For k = 1,

y(1) =Dx y

Dx x=

y′ + εDxη + O(ε2)

1 + εDxξ + O(ε2)= y′ + ε

(Dxη − y′Dxξ

)+ O(ε2), (2.32)

therefore from (2.30)η(1) = Dxη − y′Dxξ. (2.33)

Similarly

y(k) =y(k) + εDxη(k−1) + O(ε2)

1 + εDxξ + O(ε2), (2.34)

and henceη(k) = Dxη(k−1) − y(k)Dxξ. (2.35)

To find the Lie point symmetries of an ODE (2.27) we must first calculate η(k), k = 1, 2, ..., n. The functions ξ andη depend upon x and y only and therefore (2.33) and (2.35) give the following results:

η(1) = ηx + (ηy − ξx)y′ − ξyy′2, (2.36)

η(2) = ηxx + (2ηxy − ξxx)y′ + (ηyy − 2ξxy)y′2 − ξyyy′3 + (ηy − 2ξx − 3ξyy′)y′′, (2.37)

η(3) = ηxxx + (3ηxxy − ξxxx)y′ + 3(ηxyy − ξxxy)y′2 +(

ηyyy − ξyy

)y′3 − ξyyyy′4 + 3(ηxy − ξxx+

(ηyy − 3ξxy

)y′ − 2ξyyy′2)y′′ − 3ξyy′′2 +

(ηy − 3ξx − 4ξyy′

)y′′′, (2.38)

etc....

For example, for an ODE y′′ = ω(x, y, y′) the linearized symmetry condition is obtained substituting (2.36) and(2.37) into (2.31) and then replacing y′′ by ω(x, y, y′). This gives

η(2) = ξωx + ηωy + η(1)ωy′ ,

i.e.

ηxx + (2ηxy − ξxx)y′ + (ηyy − 2ξxy)y′2 − ξyyy′3 + (ηy − 2ξx − 3ξyy′)ω

= ξωx + ηωy +(

ηx + (ηy − ξx)y′ − ξyy′2)

ωy′ . (2.39)

Page 29: Ecuaciones Diferenciales Mediante Simetrías · vi CONTENTS Lieperomuchísimomássencillodeaplicaryaqueunonodebeaplicarningúnalgoritmoparaobtener las simetrías que inducen dichos

19

For a first-order ODEs, the right-hand side of the linearized symmetry condition (2.31) is Xω where X is theinfinitesimal generator i.e. X = ξ∂x + η∂y. This infinitesimal generator is associated with the tangent vector tothe orbit passing through (x, y) , namely

(ξ, η) =

(dx

dε,

dy

)

|ε=0. (2.40)

To deal with the action of LS on derivatives of order n or smaller we introduce the prolonged infinitesimal generator

X(n) = ξ∂x + η∂y + η(1)∂y′ + ..... + η(n−1)∂y(n−1) , (2.41)

the coefficient of ∂y(k) and so X(n) is associated with the tangent vector in the space variables (x, y, y′, ....., y(n−1)).We can use the prolonged infinitesimal generator to write the linearized symmetry condition (2.31) in a compactform:

X(n)(y(n) − ω(x, y, y′, ....., y(n−1))) = 0, when (2.27) holds. (2.42)

Ejemplo 2.0.3 Consider a third-order ODEy′′′ = N(x, y, y′, y′′). (2.43)

A vector field S = ξ(x, y) ∂∂x + η(x, y) ∂

∂y , is called a symmetry of (2.43) if and only if

S[3](y′′′ − N(x, y, y′, y′′))|(2.43) = 0, (2.44)

where S[3] is the third prolongation of S defined by: S[3] = S + ∑3i=1 η(i)∂y(i) , where

η(1) = ηx + (ηy − ξx)y′ − ξyy′2,

η(2) = ηxx + (2ηxy − ξxx)y′ + (ηyy − 2ξxy)y′2 − ξyyy′3 + (ηy − 2ξx − 3ξyy′)y′′,

η(3) = ηxxx + (3ηxxy − ξxxx)y′ + 3(ηxyy − ξxxy)y′2 +(

ηyyy − ξyy

)y′3 − ξyyyy′4 + 3(ηxy − ξxx+

(ηyy − 3ξxy

)y′ − 2ξyyy′2)y′′ − 3ξyy′′2 +

(ηy − 3ξx − 4ξyy′

)y′′′,

and |(2.43) means evaluated on (2.43). Straightforward calculations indicate that (2.44) is equivalent to

η(3) = ξNx + ηNy + η(1)Ny′ + η(2)Ny′′ . (2.45)

Since η and ξ are independent of y′, Equation (2.45) can be separated with respect to powers of y′ to give an overdeterminedsystem of linear partial differential equations for η and ξ. This system is easier to solve than the original equation (2.43):this is the main stength of the Lie method.

Reduction of order by using canonical coordinates. Suppose that X is an infinitesimal generator of a oneparamenter LG of symmetries of the ODE

y(n) = ω(

x, y′, y′′, ...., y(n−1))

, n ≥ 2. (2.46)

Let (r, s) be canonical coordinates for the group generated by X, so that

X = ∂s. (2.47)

If the ODE (2.46) is written in terms of canonical coordinates, it is of the form

s(n) = Ω(

r, s, s, ..., s(n−1))

, s(k) =dks

drk, (2.48)

Page 30: Ecuaciones Diferenciales Mediante Simetrías · vi CONTENTS Lieperomuchísimomássencillodeaplicaryaqueunonodebeaplicarningúnalgoritmoparaobtener las simetrías que inducen dichos

20 CHAPTER 2. INTRODUCTION TO THE SYMMETRY METHODS.

for some Ω, and where s = dsdr . However, the ODE (2.48) is invariant under the LG of translations in s, so the

symmetry condition gives, Ωs = 0. Therefore

s(n) = Ω(

r, s, ..., s(n−1))

. (2.49)

By writing the ODE (2.46) in terms of the canonical coordinates, we have reduced it to an ODE of order n − 1,for v = s :

v(n−1) = Ω(

r, v, ..., v(n−2))

, v(k) =dk+1s

drk+1 . (2.50)

Suppose that the reduced ODE has the general solution

v = f (r; c1, c2, ...., cn−1) , (2.51)

then the general solution of the original ODE is

s (x, y) =∫ r(x,y)

f (r; c1, c2, ...., cn−1) dr + cn. (2.52)

More generally, if v is any function of s and r such that vs (r, s) 6= 0, the ODE (2.49) reduces to an ODE of theform

v(n−1) = Ω(

r, v, ..., v(n−2))

, v(k) =dkv

drk. (2.53)

Once the general solution of (2.53) has been found, the relationship s = s (r, v) , gives the general solution of(2.46):

s (x, y) =∫ r(x,y)

s (r; v (r; c1, c2, ...., cn−1)) dr + cn. (2.54)

Ejemplo 2.0.4 Consider the linear ODE

y′′ =

(3x− 2x

)y′ + 4y (2.55)

where by using all the above theoretical results we get

ξyy = 0,

−2(

3x− 2x

)ξy + ηyy − 2ξxy = 0,

−12yξy − ξxx +

(3x2 + 2x

)ξ −

(3x− 2x

)ξx + 2ηxy = 0,

−8yξx − 4η + 4yηy −(

3x− 2x

)ηx + ηxx = 0,

and therefore we obtaing the following symmetries:

X1 = y∂y, X2 =(

x2 − 1)

∂y, X3 = e−x2∂y, X4 =

y

x3 ∂x −2y2

x2 ∂y,

X5 =x2 − 1

x3 ∂x +2y

x2 ∂y, X6 =e−x2

x3 ∂x −2ye−x2

x2 ∂y,

X7 =y(x2 − 1)ex2

x3 ∂x −2y2e−x2

x2 ∂y, X8 =(x − 1)2 (x + 1)2 ex2

x3 ∂x −2y (x − 1) (x + 1) ex2

x2 ∂y.

Page 31: Ecuaciones Diferenciales Mediante Simetrías · vi CONTENTS Lieperomuchísimomássencillodeaplicaryaqueunonodebeaplicarningúnalgoritmoparaobtener las simetrías que inducen dichos

21

We consider the the first of them, namely X1 = y∂y, where the simplest canonical coordinates are

r = x, s = ln y,

which prolong to

v =ds

dr=

y′

y,

d2s

dr2 =y′′

y−

(y′

y

)2

,

hence our ODE reduces to the Riccati equation

v′ =

(3r− 2r

)v + 4 − v2,

where its solution is

v = −2r

(C1e−r2 − 1

)

C1e−r2 − 1 + r2,

all that remains it to carry out the c.v.

y′

y= −

2x(

C1e−x2 − 1)

C1e−x2 − 1 + x2,

so the solution for our original ODE yields

y = C2

(C1e−x2 − 1 + x2

).

Ejemplo 2.0.5 For the ODE

y′′ =y′2

y+

(y − 1

y

)y′

we find the following symmetryX = ∂x

which leads us to get the following c.v.r = y, s(r) = x,

thus (this is a central step in all the work)

v =ds

dr=

1y′

, v′ = − y′′

(y′)3 ,

and therefore we get:

v′ = −v

r−

(r +

1r

)v2.

Page 32: Ecuaciones Diferenciales Mediante Simetrías · vi CONTENTS Lieperomuchísimomássencillodeaplicaryaqueunonodebeaplicarningúnalgoritmoparaobtener las simetrías que inducen dichos

22 CHAPTER 2. INTRODUCTION TO THE SYMMETRY METHODS.

Page 33: Ecuaciones Diferenciales Mediante Simetrías · vi CONTENTS Lieperomuchísimomássencillodeaplicaryaqueunonodebeaplicarningúnalgoritmoparaobtener las simetrías que inducen dichos

Bibliography

[1] G. W. Bluman and J. D. Cole. “Similarity Methods for Differentials Equations”. (1974) Appl. Math. Sci. N13Springer-Verlang New York. G. W. Blumann and S.W. Kumei “Symmetries and Differential Equations”.Springer Verlang (1989). G. W. Blumann and S.C. Anco. “Symmetries and Integration Methods for Differen-tial Equations". Springer Verlang (2002).

[2] L. V. Ovsiannikov, Group Analysis of Differential Equations (Academic Press 1982).

[3] N. H. Ibragimov. “Transformation Groups Applied to Mathematical Physics”. D. Reidel (1985). N. H. Ibragimov.“Elementary Lie Group Analysis and Ordinary Differential Equations” (John Wiley & Sons, 1999).

[4] R. Seshadri and T. Y. Na. “Group Invariance in Engineering Boundary” Value Problems. Springer-Verlang. NY,1985.

[5] H. Stephani, “Differential Equations: Their Solutions Using Symmetries” (Cambridge University Press 1989).

[6] P. T. Olver, “Applications of Lie Groups to Differential Equations” (Springer-Verlang, 1993).

[7] L. Dresner, “Applications of Lie´s Theory of Ordinry and Partial Differential Equations”. IOP 1999

[8] G. Bauman, “Symmetry Analysis of Differential Equations”. Springer Verlang (2000).

[9] P. E. Hydon, “Symmetry Methods for Differential Equations” (Cambridge University Press, 2000).

[10] B. J. Cantwell, “Introduction to Symmetry Analysis” (Cambridge University Press, Cambridge, 2002).

23

Page 34: Ecuaciones Diferenciales Mediante Simetrías · vi CONTENTS Lieperomuchísimomássencillodeaplicaryaqueunonodebeaplicarningúnalgoritmoparaobtener las simetrías que inducen dichos

24 BIBLIOGRAPHY

Page 35: Ecuaciones Diferenciales Mediante Simetrías · vi CONTENTS Lieperomuchísimomássencillodeaplicaryaqueunonodebeaplicarningúnalgoritmoparaobtener las simetrías que inducen dichos

Chapter 3

Ecuaciones de Primer Orden

3.1 Introducción

D.A. has usually been employed in different areas (fields) such as engineering problems, fluid mechanics etc...and these problems are always described by partial differential equations (pde) (see [1]-[10]). This method (“tac-tic") helps us to reduce the number of quantities that appear into an equation and to obtain ordinary differentialequations (ode). We would like to point out (emphasize) that this tool is more effective if one practices the spatialdiscrimination, such tactic allows us to obtain better results than with the standard application of D.A. (see [11]).Knowing that D.A. works well in pde we would like to extend this method to the study of ode (the first orderin this case) in a systematic way. There are in the literature previous work in this direction for ode of first order(see [12]-[13].

The method of the Lie groups it has showed as a very useful tool in order to solve nonlinear equations (nl-ode)as well as pde (see [14]-[27] ). Nevertheless, when one is studying ode of first order its application results verycomplicated (very tedious) if one has not a computer algebra package since if one decides to look for the possiblesymmetries of an ode with pencil and paper this task may be turned very exhausting. However, if we knowthat an ode admits a concrete symmetry, then it is a trivial issue to find new variables which allows us to rewritethe ode in quadratures or as we will see in this paper to obtain an ode with separating variables.

Our purpose in this work is to explain, through examples, how D.A. works in order to find these changes ofvariables (c.v.) in a trivial way, i.e. without the knowledge of the symmetries of the ode under study. The ideais as follows. When we are studying an ode form the dimensional point of view, we must require that suchode verifies the principle of dimensional homogeneity (pdh) i.e. that each term within the equation have thesame dimensions, for example, speed, or energy density. To clarify this concept we consider the following ode,

y′ =y

x+ x, where each term must have dimensions of y′ i.e. [y′] = yx−1, where [·] stands for dimensional

equation of the quantity ·. As we can see, this ode does not verify the pdh, since the term x has dimensions of x,i.e. [x] = x. In order to do that this equation verifies the pdh we need to introduce dimensional constants, in sucha way that after rewrite the ode with these constants the ode now verifies such principle i.e. in this example and

as we can see easily if we consider only one constant a, such that [a] = yx−2, we make that the ode y′ =y

x+ ax,

verifies the pdh. We would like to emphasize that this situation does not appear (arise) when one is studyingphysical or engineering problems since (as it is supposed) that such problems (equations) verify the pdh andwe do not need to introduce new dimensional constants, that must have physical meaning, for example, theviscosity coefficient, etc....

Precisely this dimensional constant suggests us the c.v. (x, y(x)) 7−→ (t, u(t)) where(t = x, u = yx−2) in such

a way that rewriting the original ode in these new variables it is obtained a new ode with separating variables:u + u′t = 1. The reason is the following. We know from the Lie group theory that if it is known a symmetry ofan ode then this symmetry bring us through a c.v. to obtain a simpler ode (a quadrature or a ode with separating

25

Page 36: Ecuaciones Diferenciales Mediante Simetrías · vi CONTENTS Lieperomuchísimomássencillodeaplicaryaqueunonodebeaplicarningúnalgoritmoparaobtener las simetrías que inducen dichos

26 CHAPTER 3. ECUACIONES DE PRIMER ORDEN

variables) and therefore the solution is found in a closed form. To find this c.v. it is used the invariants thatgenerate each symmetry, i.e. the first principle is that it is useful to pass to new coordinates such that one of thecoordinate functions is an invariant of the group. After such transformation it often (but not always) happensthat the variables separate and the equation can be solved in closed form. We must stress that taking the newdependent variables to be an invariant of the group does not guarantee the separation of variables. The choiceof the independent variable is also very important.

One must note that D.A. gives us this invariant (or at least a particular solution). It is observed in our examplethat the solution y = x2, (suggested by D.A.) is a particular solution for this ode, but if we study this ode formthe Lie theory point of view it is obtained that such ode has the scaling symmetry X = x∂x + 2y∂y and thereforethe solution y = x2, is furthermore an invariant solution (generated by X), for this reason such c.v. works well.As we will show in the paper, the c.v. that D.A. induces is the same than the generated one by the symmetries ofthe ode. In order to make see this fact, we will solve each example by D.A. as well as by the Lie group technique(abusing of the trivial calculations), calculating the symmetries and their corresponding invariants and c.v..

However D.A. has a little (or rather big) drawback, it only gives us relationship of type power i.e. y = xn, withn ∈ R, and we cannot obtain relationship as y = (x + 1)n. Nevertheless, and as we will see in the examples,although D.A. does not give us an invariant solution it will be sufficient that it provides us a particular solutionin order to obtain a c.v. which brings us to a simpler ode than the original one. This task will be very useful inthe case of studying Riccati and Abel odes. Furthermore, this “tactic" is valid even in the case in which the odehas no symmetries but unfortunately one always founds examples in which any tactic does not work.

The paper is organized as follows: In section 2 we describe (with all the tedious and superfluous calculations),through two examples, how the proposed method works by comparing it with the Lie method in order to showwhich are the symmetries and their corresponding invariants. In section 3 we shall show several kind of ODEspaying special emphasis in for example Bernoulli, Riccati and Abel ODEs (see [28]-[32]). We think that thesekind of ode are the most difficult to solve and therefore any simplification is welcome. We show two Abel odesthat do not admit symmetries (see [33]-[37]). In the first of them we use the D.A. to obtain a particular solutionthat allows us to obtain a Bernoulli ode but in the second one we are not able to find any particular solution andtherefore a c.v. which brings us to obtain a simpler ode. We end with some conclusion as well as pointing outsome of the limitations of the proposed tactic.

We would like to emphasize the pedagogical character of this chapter (at least we try) for this reason we haveabused of superfluous calculations and we have omitted some technical details since the supposed audience aremainly engineers and/or physicists but not for mathematicians.

3.2 The method

In this section we will explain how the D.A. works in order to solve odes through two simple examples. Themain idea is to introduce dimensional constants that make the ode under study verify the principle of dimen-sional homogeneity. These dimensional constants help us to find c.v. which bring us to obtain ode with sepa-rating variables. These c.v. obtained through D.A. correspond to invariant solution (or at least to a particularsolution) and therefore they are induced by one symmetry. We compare our tactic with the Lie one.

Ejemplo 3.2.1 Solve the homogeneous equation

(x2 + y2

)dx = 2xydy. (3.1)

Solution. In order to solve it we will use three different ways, the traditional, the dimensional and the Liemethod.

Page 37: Ecuaciones Diferenciales Mediante Simetrías · vi CONTENTS Lieperomuchísimomássencillodeaplicaryaqueunonodebeaplicarningúnalgoritmoparaobtener las simetrías que inducen dichos

3.2. THE METHOD 27

Traditional method. Making the c.v. u = y/x we have:

u′ =1 − u2

2ux=⇒ 2udu

1 − u2 =dx

x=⇒ ln(1 − u2) = ln x =⇒ y2 = x2 + x, (3.2)

Dimensional Analysis. We go next to consider eq. (3.1), written as follows

y′ =

(a2x2 + y2)

2xy, (3.3)

where the dimensional constant a, makes the ode verify the dimensional principle of homogeneity (d.p.h.) if

[a] =[ y

x

]= X−1Y, (3.4)

where [·] stands for the dimensional equation of the quantity ·.

Applying the Pi theorem we obtain the dimensionless variables that help us to simplify the original ode. There-fore taking into account the following dimensional matrix we take

y x ax 0 1 −1y 1 0 1

=⇒ π1 =ax

y, =⇒ y = ax, (3.5)

as we will see later this solution is a particular solution of this ode. We would like to emphasize that as we haveonly needed one constant then the equation is scale-invariant in such a way that the generator of this group isX = x∂x + y∂y as it is observed from eq. (3.5). This fact will be probed studying this equation under the Liegroup tactic, see below.

In this way the new variables are (t, u(t)) :(

t = x, u(t) = ax

y

), =⇒

(x = t, y = a

t

u(t)

), (3.6)

this change of variables brings us to rewrite eq. (3.3) as follows:

u′

u (1 − u2)=

12t

=⇒ u√1 − u2

=12

ln t, (3.7)

and hencey2 = a2x2 + C1x. (3.8)

As we can see in this trivial example, the D.A. induces a c.v. which helps us to obtain an ode simpler than theoriginal one.

We can also think in the following way

ay′ = bx

2y+ a

y

2x, (3.9)

where[a] = x, [b] = y2x−1, (3.10)

and hence (t =

x

a, u(t) =

y2

bx

)=⇒

(x = at, y =

√abtu(t)

), (3.11)

therefore eq. (3.9) yieldsu′ = 1 =⇒ u = t + C1, (3.12)

in this way we obtain the solution

y2 =b

ax2 + C1x, (3.13)

Page 38: Ecuaciones Diferenciales Mediante Simetrías · vi CONTENTS Lieperomuchísimomássencillodeaplicaryaqueunonodebeaplicarningúnalgoritmoparaobtener las simetrías que inducen dichos

28 CHAPTER 3. ECUACIONES DE PRIMER ORDEN

once we have obtained the solution, the constants a, b are makig equal to 1, i.e. a = b = 1.

In the same way we may consider the following change of variables(

t =x

a, u(t) =

y√bx

)=⇒

(x = at, y = u(t)

√abt

), (3.14)

which brings us to the following ode

2u′u = 1 =⇒ u2 − t + C1 = 0, (3.15)

and therefore we obtain again the solution (3.13). As we will see below all these c.v. are generated by theircorresponding (respective) symmetry.

As we can see, this last change of variables is better than the first and the tactic is the same: to introduce dimen-sional constants that make the equation dimensional homogeneous.

Lie Method. In order to find the symmetry generator of a ode

y′ = f (x, y) , (3.16)

we need to solve the following pde

ηx + (ηy − ξx) f − ξy f 2 − ξ(x, y) fx − η(x, y) fy = 0, (3.17)

where

fx =d f

dx, fy =

d f

dy. (3.18)

In this case we have to solve

ηx +12

(ηy − ξx)(x2 + y2)

x y− 1

4

ξy(x2 + y2)2

x2 y2 − ξ(x, y)(1y− x2 + y2

2 x2y) − η(x, y)(

1x− x2 + y2

2 x y2 ) = 0, (3.19)

we find that(X = ξ∂x + η∂y

):

X1 =x

2 y∂y, X2 =

(− x2

2 y+

y

2

)∂y, X3 = x∂x + y∂y,

X4 = ∂x +y

2 x∂y, X5 =

(x2 + y2

)∂x + 2xy∂y, (3.20)

observing that X5 is a trivial symmetry.

Each symmetry induces a change of variable (canonical variables) which are obtained through the followingformula

Xt = 0, Xu = 1. (3.21)

In this way the change of variables that induces the field X4 = ∂x + y2 x ∂y is:

(t =

y√x

, u(t) = x

)=⇒

(x = u(t), y = t

√u(t)

), (3.22)

in such a way that eq. (3.75) yields:u′ = 2t =⇒ u(t) = t2 + C1, (3.23)

and therefore

x =y2

x+ C1, (3.24)

which is the same solution. The c.v. induced by this symmetry is similar to the obtained one in (3.14).

Page 39: Ecuaciones Diferenciales Mediante Simetrías · vi CONTENTS Lieperomuchísimomássencillodeaplicaryaqueunonodebeaplicarningúnalgoritmoparaobtener las simetrías que inducen dichos

3.2. THE METHOD 29

The c.v. that induces the field X1 = x2 y ∂y is the following one

(t = x, u(t) =

y2

x

)=⇒

(x = t, y =

√u(t)t

), (3.25)

therefore eq.(3.1) yieldsu′ = b2, (3.26)

this c.v. is the same than the obtained one in (3.11).

For example the c.v. that induces the field X3 = x∂x + y∂y is

(t =

y

x, u(t) = ln x

)=⇒

(x = eu(t), y = teu(t)

), (3.27)

which brings us to the following ode :

u′ = − 2t

t2 − 1=⇒ u(t) = − ln (−1 + t) − ln (1 + t) + C1, (3.28)

now writing the solution in the original variables we take

ln x = C1 + ln(

x

(y − x) (y + x)

). (3.29)

Now if we consider the invariants that induce each symmetry

dx

ξ=

dy

η, 7−→ y′ :=

dy

dx=

η

ξ, (3.30)

we take that:X1 7−→ I1 = x, X2 7−→ I2 = x, X3 7−→ I3 =

y

x, X4 7−→ I4 =

y√x

. (3.31)

For example the symmetry X3 = x∂x + y∂y generates the following invariant:

dx

ξ=

dy

η=⇒ dx

x=

dy

y=⇒ ln x = ln y =⇒ I3 =

y

x7−→ y = ax, (3.32)

this would be the solution that suggests us precisely the direct use of the Pi theorem (if the ode is scale invariant,as in this case, the solution obtained applying by the Pi theorem coincides with the invariant solution thatgenerates the scale symmetry, in this case X3). We see that we only obtain a particular solution, but that this isinvariant, in fact if we think about the ode as a dynamical system we see that the fixed point of such equationwould be precisely the solution y = ±x.

Ejemplo 3.2.2 Solve the linear ode (1 − x2

)y′ + xy = 1. (3.33)

Solution. Solution through D.A. Our first step will be to introduce dimensional constants in such a way thateq. (3.33) verifies the principle of dimensional homogeneity. In this way we rewrite the equation as follows

(A − x2

)y′ + xy = B, (3.34)

where[A] = X2, [B] = XY, (3.35)

Page 40: Ecuaciones Diferenciales Mediante Simetrías · vi CONTENTS Lieperomuchísimomássencillodeaplicaryaqueunonodebeaplicarningúnalgoritmoparaobtener las simetrías que inducen dichos

30 CHAPTER 3. ECUACIONES DE PRIMER ORDEN

i.e.

π1 =x2

Aπ2 =

xy

B, (3.36)

where

y =1

Bxϕ

(x2

A

), (3.37)

being ϕ a unknown function. It is possible to make the following assumption

y =1

Bx

(x2

A

)n

, n ∈ R. (3.38)

Since y = x−1 is not a particular solution of (3.33) we need to combine the π − monomias in order to find aparticular solution, the simplest way is as follows:

(t =

x2

A, u(t) =

A

B

y

x

)=⇒

(x =

√At, y =

B

Au(t)

√At

)(3.39)

observing thatu(t) = π2 · π−1

1 , (3.40)

where y = cx, derived from π2 · π−11 is a particular solution of (3.33).

Observación 3.2.1 (Recipe) When we have two π − monomia, we have to check if they induce any particular (or invari-ant) solution. If they are not particular solutions then we must combine them in order to obtain such solution, this solutionusually is obtained by combining them in a very simple way.

In this way our ode is written now as follows:

2u′

u − 1=

1t (t − 1)

=⇒ u = 1 + C1

√t + 1t

, (3.41)

and in the original variables the solution of eq. (3.33) yields:

y =A

B

(x ± C1

√A − x2

), (3.42)

where C1 is an integration constant. Now making A = B = 1, we have the ordinary solution to eq. (3.33).

Lie method. Following the standard procedure we have to solve the pde:

ηx +(ηy − ξx) (1 − x y)

1 − x2 −ξy (1 − x y)2

(1 − x2)2 − ξ(x, y) (− y

1 − x2 +2 (1 − x y) x

(1 − x2)2 ) +η(x, y) x

1 − x2 = 0, (3.43)

which solutions are:

X1 =√−1 + x2∂y, X2 = (−y + x) ∂y, X3 =

((2xy − y2 − 1)(−y + x)

(x − 1)(x + 1)

)∂y, (3.44)

and their corresponding invariants are:

X1 7−→ I1 = x, X2 7−→ I2 = x, X3 7−→ I3 = x. (3.45)

For example the symmetry X1 generates the following c.v.:(

t = x, u(t) =y√

−1 + x2

)=⇒

(x = t, y = u(t)

√−1 + t2

), (3.46)

Page 41: Ecuaciones Diferenciales Mediante Simetrías · vi CONTENTS Lieperomuchísimomássencillodeaplicaryaqueunonodebeaplicarningúnalgoritmoparaobtener las simetrías que inducen dichos

3.3. CUADRATURAS. 31

therefore eq. (3.33) is written as:

u′ = −√−1 + t2

1 − 2t2 + t4 , (3.47)

which solution is:

u(t) = −(−1 + t2)3/2

4 (t + 1)2 +

(−1 + t2)3/2

4 (t − 1)2 + C1, (3.48)

and hencey√

x2 − 1=

C1(x2 − 1) + x√

x2 − 1x2 − 1

=⇒ y = x ± c√

x − 1√

x + 1. (3.49)

Now, if for example we consider the symmetry X2 then we have:

(t = x, u(t) = − ln(−y + x)) =⇒(

x = t, y =teu(t) − 1

eu(t)

), (3.50)

therefore:u′ = − t

−1 + t2 , (3.51)

finding in this way that its solution is:

u(t) = −12

ln(t − 1) − 12

ln(t + 1) + C1, (3.52)

hence in the original variables the solution yields:

− ln(−y + x) = −12

ln(x − 1) − 12

ln(x + 1) + C1, (3.53)

which after a simple simplification it yields:

y = x ± c√

x − 1√

x + 1, (3.54)

as we already know.

With these two simple examples we have tried to show how the D.A. works in order to introduce c.v. whichbrings us to obtain simpler odes than the original one. As we have emphasized in the recipe, the trick is to lookfor a particular solution. Sometimes this particular solution will be furthermore invariant solution (induced bya concrete symmetry). If this is the case, then our ode will be reduced to an ode with separating variables but ifthis solution is only a particular then, as we will see below, we have not any guarantee of reducing our ode to anode with separate variables, nevertheless we will obtain a simpler ode than the original one. In the next sectionwe will study some examples.

3.3 Cuadraturas.

Decimos que una ecuación diferencial está en forma de cuadraturas cuando sucede alguna de las siguientessituaciones:

1. la ecuación es de primer orden y el lado derecho de la ecuación depende sólo de x

y′ = F(x) (3.55)

la solución es por lo tanto de la forma

y(x) =∫

F(x) dx + C1. (3.56)

Page 42: Ecuaciones Diferenciales Mediante Simetrías · vi CONTENTS Lieperomuchísimomássencillodeaplicaryaqueunonodebeaplicarningúnalgoritmoparaobtener las simetrías que inducen dichos

32 CHAPTER 3. ECUACIONES DE PRIMER ORDEN

2. la ecuación es de primer orden y el lado derecho de la ecuación depende sólo de y(x):

y′ = F(y(x)) (3.57)

la solución es por lo tanto de la forma

∫ y(x) 1F(a)

da + C1 = x (3.58)

3.3.1 Simetrías.

Desde el punto de vista de las simetrías vemos que la ecuación (3.55) sólo puede tener las siguientes simetrías

ηx + (ηy − ξx) F(x) − ξy F(x)2 − ξ(x, y) F′ = 0

encontrando que:[ξ = 0, η = 1]

mientras que la ecuación (3.57) sólo puede tener las simetrías.

ηx + (ηy − ξx) F(y) − ξy F(y)2 − η(x, y) Fy = 0

encontrando que:[ξ = 1, η = 0]

3.3.2 Ejemplos

Veremos unos sencillos e inmediatos ejemplo de cada tipo de ecuació descrito en la anterior subsección.

Ejemplo 3.3.1 Resolver la ecuación en cuadraturas

x + x y′2 = 1

Solución. Vemos que

y = −√−x2 + x − 1

2arcsin(2 x − 1) + C1,

y =√−x2 + x +

12

arcsin(2 x − 1) + C1

ya que

ηx +(ηy − ξx)

√−x (x − 1)

x+

ξy (x − 1)

x− ξ(x, y) (−

√−x (x − 1)

x2 +−2 x + 1

2 x√−x (x − 1)

) = 0

y por lo tanto[ξ = 0, η = 1]

como ya sabíamos.

Intentaremos obtener alguna solución particular a la ecuación

y′ =

√a − bx

x

Page 43: Ecuaciones Diferenciales Mediante Simetrías · vi CONTENTS Lieperomuchísimomássencillodeaplicaryaqueunonodebeaplicarningúnalgoritmoparaobtener las simetrías que inducen dichos

3.4. ECUACIONES EN VARIABLES SEPARABLES Y REDUCIBLES A ELLAS. 33

donde las constantes dimensionales a y b tienen las siguientes dimensiones: [a] = y2x−1, [b] = y2x−2. En estecaso la solución que sugiere el AD es de la forma

y2 = ax · ϕ

(bx2

y2

),

observando que semejante sugerencia está muy lejos de ser una solución a la ODE propuesta. Intentaremos otratáctica dimensional, forzando a que la función desconocida ϕ sea de la forma πn

b , i.e.

y2 = ax ·(

bx2

y2

)n

=⇒ y2n+2 = abnx2n+1

de esta forma

y =(

abnx2n+1) 1

2n+2

que tampoco funciona.

Ejemplo 3.3.2 Resolver la ecuación en cuadraturas

y′ = y2

Solución. Vemos que

y = − 1x − C1

ya queηx + (ηy − ξx) y2 − ξy y4 − 2 η(x, y) y = 0

y por lo tanto[ξ = 1, η = 0]

como ya sabíamos.

En este ejemplo la solución que nos sugiere el AD es la siguiente. Introducimos la constante a para hacer que laEDO sea d.h. i.e. y′ = ay2, con [a] = y−1x−1, de esta forma llegamos a

y a xy 1 −1 0x 0 −1 1

=⇒ y =1ax

observando que sólo después de haber sustituido esta solución en la ODE original nos daríamos cuenta del signoi.e. y = −1

ax , ya que el AD “no entiende” ni de signos ni de constantes numéricas, sólo de órdenes de magnitud.

Claim 3.3.1 Resaltamos que a pesar de no tener simetrías de escala hemos podido obtener una solución por AD.

3.4 Ecuaciones en variables separables y reducibles a ellas.

Son de la formay′ = f (x)g(y) (3.59)

Page 44: Ecuaciones Diferenciales Mediante Simetrías · vi CONTENTS Lieperomuchísimomássencillodeaplicaryaqueunonodebeaplicarningúnalgoritmoparaobtener las simetrías que inducen dichos

34 CHAPTER 3. ECUACIONES DE PRIMER ORDEN

3.4.1 Método tradicional.

Tenemos que separar, viendo de forma inmediata que:

y′

g(y)= f (x) (3.60)

por lo tantody

g(y)= f (x)dx =⇒

∫dy

g(y)=

∫f (x)dx + C1 (3.61)

3.4.2 Método dimensional.

En primer lugar deberemos reescribir la ecuación introduciendo cuantas constantes dimensionales sena nece-sariaas para hacer que la ecuación verifique el principio de homogeneidad de tal forma que

[dy

g(y)

]= [A1 f (x)dx] (3.62)

donde la constante A1 hace que la anterior ecuación sea dimensionalmente homogénea.

3.4.3 Método de Lie.

En esta ocasión el método de Lie nos lleva a obtener las siguientes simetrías para la ecuación (3.59) al resolver lasiguiente ecuación:

ηx + (ηy − ξx) f (x)g(y) − ξy f (x)2g(y)2 − ξ(x, y) f ′g(y) − η(x, y) f (x)g′ = 0 (3.63)

cuyas soluciones son:

[ξ = 0, η = g(y)], [ξ =1

f (x), η = 0], [ξ =

1f (x)

, η = g(y)]

que conducen a la siguiente solución:

dx

ξ(x, y)=

dy

η(x, y)=⇒

∫dy

g(y)=

∫f (x)dx + C1

Resaltamos que de las simetrías obtenidas ξ = 1f (x)

, η = g(y), podremos tener simetrías de escala (las que genera

el AD) sii f (x) = 1x y g(y) = y.

3.4.4 Ejemplo.

Ejemplo 3.4.1 Vamos a estudiar la siguente ode:

y′ = −sin(x)y

Solución. Resolveremos la ecuación planteada por los tres métodos, intentando mostar las ventajas y desventa-jas de cada método.

Page 45: Ecuaciones Diferenciales Mediante Simetrías · vi CONTENTS Lieperomuchísimomássencillodeaplicaryaqueunonodebeaplicarningúnalgoritmoparaobtener las simetrías que inducen dichos

3.4. ECUACIONES EN VARIABLES SEPARABLES Y REDUCIBLES A ELLAS. 35

1. Método tradicional.

Vemos de forma inmediata que la solución es:

∫dy

y=

∫− sin xdx =⇒ y = C1ecos(x)

2. Método dimensional.

Tenemos que introdir constantes para hacer que la ecuación sea dimensionalmente homogénea, encon-trando así que:

y′ = −A1sin(x)y

donde [A1] = x−1 y donde estamos considerendo que [sin(A1x)] = x0y0, i.e. es adimensional. Por lo tantola soluión que sugiere el AD es:

y = y0ψ(sin(A1x))

donde ψ hace referencia a una unción desconocida de sin(A1x). Con este ejemplo vemos que en este casoel AD no nos acerceca ni de lejos a una solución a una ecuación tan sencilla como la planteada.

3. Método de Lie.

Veamos ahora las simetrías de la ecuación al resolver la ecuación:

ηx − (ηy − ξx)y sin(x) − ξyy2 sin(x)2 + ξ(x, y) y cos(x) + η(x, y) sin(x) = 0

todas las posibles simetrías son:

[ξ = 0, η = y], [ξ = 0, η = ecos(x)], [ξ =1

sin(x), η = 0], [ξ =

1sin(x)

, η = −y].

por lo tanto: ∫− sin xdx =

∫dy

y

llegando así a la solución:y = C1ecos(x)

Las variables canónicas generadas por [ξ = 0, η = y] nos llevan al siguiente resultado:

(t = x, u(t) = ln y) −→(

x = t, y = eu(t))

por lo tanto la ecuación diferencial original queda reducida a:

u′ = − sin t =⇒ u = cos t + C1

por lo tanto la solución en las variables originales es:

ln y = cos x + C1 =⇒ y = C1ecos(x).

Podemos observar las ventajas y desventajas que presentan cada método.

Ejemplo 3.4.2 Vamos a estudiar la siguente ode:y′ = xy

Solución. Resolveremos la ecuación planteada por los tres métodos, intentando mostar las ventajas y desventa-jas de cada método.

Page 46: Ecuaciones Diferenciales Mediante Simetrías · vi CONTENTS Lieperomuchísimomássencillodeaplicaryaqueunonodebeaplicarningúnalgoritmoparaobtener las simetrías que inducen dichos

36 CHAPTER 3. ECUACIONES DE PRIMER ORDEN

1. Método tradicional.

Vemos de forma inmediata que la solución es:∫

dy

y=

∫xdx =⇒ y (x) = C1e

12 x2

2. Método dimensional.

Como en l ejemplo anterior reescribimos la ecuación de tal forma que

y′ = A1xy

con [A1] = x−2

y = y0 ϕ(

A1x2)

bastante pobre.

3. Método de Lie.

Veamos ahora las simetrías de la ecuación al resolver la ecuación:

ηx − (ηy − ξx)yx − ξyy2x2 + ξ(x, y)y + η(x, y)x = 0

una de las posibles simetrías es:

[ξ = 0, η = y], [ξ = 0, η = e−12 x2

], [ξ =1x

, η = 0], [ξ =1x

, η = y].

por lo tanto: ∫xdx =

∫dy

y

llegando así a la solución:

y (x) = C1e12 x2

El empleo de las variables canónicas (v.c) que generan la simetría [ξ = 0, η = y] nos lleva a los siguientesresultados:

(t = x, u(t) = ln y) −→(

x = t, y = eu(t))

por lo tanto la ecuación diferencial original queda reducida a:

u′ = t =⇒ u =12

t2 + C1

por lo tanto la solución en las variables originales es:

ln y =12

x2 + C1 =⇒ y (x) = C1e12 x2

.

3.5 Ecuación homogénea

En primer lugar deberemos distinguir los siguientes casos:

1. Homogénea clase Ady

dx= f

( y

x

)(3.64)

Page 47: Ecuaciones Diferenciales Mediante Simetrías · vi CONTENTS Lieperomuchísimomássencillodeaplicaryaqueunonodebeaplicarningúnalgoritmoparaobtener las simetrías que inducen dichos

3.5. ECUACIÓN HOMOGÉNEA 37

2. Homogénea clase B

F(

y′,y

x

)(3.65)

3. Homogénea clase C

y′ = F

(ax + by + c

rx + sy + t

)(3.66)

4. Homogénea clase D

y′ =y

x+ g(x) f

( y

x

)(3.67)

5. Homogénea clase G

y′ =y f

( yxa

)

x(3.68)

3.5.1 Método tradicional.

Describos a continuación los métodos de resolución de cada tipo de ecuación.

1. Clase A. La forma de resolver la ecuación (3.64) es la de hacer el típico c.v.

u =y

x(3.69)

de tal forma que la ecuación a resolver es:

xu′ = f (u) − u =⇒ du

f (u) − u=

dx

x(3.70)

que es de variables separables y por lo tnato facilmente integrable.

2. Clase B. La forma de resolver la ecuación (3.65) es la de hacer el típico c.v.

ϕ(t) =y

x, y′ = ψ(t) (3.71)

de esta forma derivando y = xϕ(t) respecto de x aparece una ecuación en variables separadas.

3. Clase C. La forma de resolver la ecuación (3.66) deberemos tener en cuenta las siguientes posibilidades:

(a) Que las dos rectas ax + by + c y rx + sy + t se corten en un punto (x0, y0) . En este caso se efectua elc.v. X = x − x0, Y = y − y0. De esta forma la ecuación resultante se reduce a una homogénea de claseA.

(b) Que las rectas ax + by + c y rx + sy + t sean paralelas. En este caso se efectua el c.v. z = ax − by. Deesta forma la ecuación resultante es de variables separadas.

4. Clase D. La forma de resolver la ecuación (3.67) es la de hacer el típico c.v.

5. Clase G. La forma de resolver la ecuación (3.68) es la de hacer el típico c.v.

Page 48: Ecuaciones Diferenciales Mediante Simetrías · vi CONTENTS Lieperomuchísimomássencillodeaplicaryaqueunonodebeaplicarningúnalgoritmoparaobtener las simetrías que inducen dichos

38 CHAPTER 3. ECUACIONES DE PRIMER ORDEN

3.5.2 Método dimensional.

La ecuación (3.64) será homogénea dimensionalmente si no se discrimina entre x e y, en cuyo caso

π =y

A1x= ξ (3.72)

es adimensional, [A1] = yx−1.

Expresándola en función de ξ y de x, tendremos y = A1xξ,

y′ = ξ + A1xdξ

dx, (3.73)

eliminando y′, y en (3.64) obtenemos

ξ + A1xdξ

dx= f (ξ) =⇒ dx

x=

f (ξ) − 1, (3.74)

consiguiéndose así la separación de variables que permite su integración. Como se observa es el mísmo métodoque el clásico.

3.5.3 Método de Lie.

Como en el caso del método tradicional distinguiremos cada tipo:

1. Clase A. Tenemos que resolver la ecuación

ηx + (ηy − ξx) f (y

x) − ξy f (

y

x)2 +

ξ(x, y)D( f )( yx )y

x2 − η(x, y)D( f )( yx )

x= 0

que nos conduce a:

[ξ = 0, η = −y + x f (y

x)], [ξ = x, η = y]

y por lo tanto la osolución general ela ecuación es:

∫ y/x 1a − f (a)

da + ln(x) + C1 = 0

Resaltamos que esta tipo de ecuaciones admite simetrías de escala [ξ = x, η = y] (AD) y que por lo tanto lamenos seremos capaces de encontrar soluciones particulares (o la general comleta en el mejor de los casos)por medio de AD.

2. Clase B. En este caso las simetrías que genera la ecuación son:

[ξ = x, η = y]

y la solución general es del tipo:

∫ y/x 1F( y

x , a) + ada + ln(x) + C1 = 0

3. Clase C. La ecuación a resolver es:

ηx + (ηy − ξx)F

(ax + by + c

rx + sy + t

)− ξyF

(ax + by + c

rx + sy + t

)2

+

Page 49: Ecuaciones Diferenciales Mediante Simetrías · vi CONTENTS Lieperomuchísimomássencillodeaplicaryaqueunonodebeaplicarningúnalgoritmoparaobtener las simetrías que inducen dichos

3.5. ECUACIÓN HOMOGÉNEA 39

+ξ(x, y)D( f )

(ax + by + c

rx + sy + t

) (a

rx + sy + t− (ax + by + c) r

(rx + sy + t)2

)−

−η(x, y)D( f )

(ax + by + c

rx + sy + t

) (b

rx + sy + t− (ax + by + c) s

(rx + sy + t)2

)= 0

encontrando que las simetrías en este caso son:

[ξ =x(br − sa) + bo − sc

br − sa, η =

rc − ao + (rb − as)y

br − sa]

que conducen a la siguiente solución (invariantes):

dx

ξ(x, y)=

dy

η(x, y)=⇒ dx

x(br − sa) + bo − sc=

dy

rc − ao + (rb − as)y,

4. Clase D. El sistema a resolver es:

ηx + (ηy − ξx)( y

x+ g(x) f

( y

x

))− ξy

( y

x+ g(x) f

( y

x

))2−

−ξ(x, y)

(− y

x2 + g′(x) f( y

x

)− gD( f )

( yx

)y

x2

)− η(x, y)

(1x

+gD( f )

( yx

)

x

)= 0

encontrando que las simetrías en este caso son:

[ξ = 0, η = x f (y

x)], [ξ =

x

g(x), η =

y

g(x)]

que conducen a la siguiente solución (invariantes):

dx

ξ(x, y)=

dy

η(x, y)=⇒ g(x)dx

x=

g(x)dy

y,

sin embargo, si efectuamos el siguiente c.v.

cv := x = t, y = u(t) t

encontramos queut t + u(t) = u(t) + g(t) f (u(t))

cuya solución es∫

g(t)

tdt −

∫ u(t) 1f (a)

da + C1 = 0

y por lo tanto

−∫ y/x 1

f (a)da +

∫g(x)

xdx + C1 = 0

5. Clase G. El sistema a resolver es:

ηx + (ηy − ξx)

(y f

( yxa

)

x

)− ξy

(y f

( yxa

)

x

)2

−ξ(x, y)

(y2D f

( yxa

)a

xax2 − f( y

xa

)y

x2

)− η(x, y)

(D f

( yxa

)

x− yD f

( yxa

)

xax2

)= 0.

Las simetrías que encontramos están generadas por:

[ξ = 0, η = −y (− f (y

xn) + n)], [ξ = x, η = y n],

Page 50: Ecuaciones Diferenciales Mediante Simetrías · vi CONTENTS Lieperomuchísimomássencillodeaplicaryaqueunonodebeaplicarningúnalgoritmoparaobtener las simetrías que inducen dichos

40 CHAPTER 3. ECUACIONES DE PRIMER ORDEN

si efectuamos el siguiente c.v.cv := x = t, y = u(t) t,

encontramos una nueva ecuación diferencial

ut t + u(t) = u(t) f (u(t) t

tn),

cuya solución es:

ln(t) − C1 +∫ u(t)/n−1 1

a(n − f (a))da,

volviendo a las variables originale sencontramos que:

ln(x) + C1 +∫ y/xn 1

a( f (a)− n)da = 0.

3.5.4 Ejemplos.

Ejemplo 3.5.1 Queremos integrar la ecuación homogénea

(x2 + y2

)dx = 2xydy (3.75)

Solución. Utilizaremos tres tácticas.

1. Método tradicional.

Haciendo el c.v. u = y/x entonces

u′ =1 − u2

2ux=⇒ 2udu

1 − u2 =dx

x=⇒ ln(1 − u2) = ln x =⇒ y2 = x2 + x

2. Solución mediante Análisis dimensional.

Consideramos, en vez de (3.75), la ecuación(

b2x2 + y2)

dx = 2xydy (3.76)

que será discriminadamente homogénea (d.h.) si

[b] =[ y

x

](3.77)

Obtendremos como variable que reemplaza a y la adimensional ξ, obtenida obviamente de la siguientematriz (ec. 3.78 )

b x yLx −1 1 0Ly 1 0 1

(3.78)

De manera que podemos escribir

ξ = bx

y, y = b

x

ξ,

dy

dx= −b

x

ξ2dξ

dx+

b

ξ=

b

ξ

(1 − x

ξ

dx

). (3.79)

Page 51: Ecuaciones Diferenciales Mediante Simetrías · vi CONTENTS Lieperomuchísimomássencillodeaplicaryaqueunonodebeaplicarningúnalgoritmoparaobtener las simetrías que inducen dichos

3.5. ECUACIÓN HOMOGÉNEA 41

Sustituyendo y, dydx en (3.76)

b2(

x2 +x2

ξ2

)=

2bx2

ξ

b

ξ

(1 − x

ξ

dx

)(3.80)

de donde resulta

− ξ2 + 1 =2x

ξ

dx. (3.81)

Separando variables

dx

2x=

ξ(

1 − ξ2) =

ξ+

ξdξ

1 − ξ2 =⇒ (3.82)

12

ln x + ln A = ln ξ − 12

ln(

1 − ξ2)

=⇒ A√

x =ξ√

1 − ξ2, (3.83)

y sustituyendo ξ = b xy resulta tras sencillas operaciones

y2 = b2(

x2 +x

A2

)(3.84)

que es la solución de (3.76). La solución de (3.75) corresponde a b = 1 y será

y2 = x2 + cx (3.85)

siendo c una constante arbitraria.

También podemos seguir este otro método dimensional alternativo al anterior. Reescribimos la ecuación(3.75) como

y′ =

(c1x2 + y2)

2xy, (3.86)

donde c1 es cierta constante con dimensiones [c1] = y2x−2. De esta forma intentamos encontar una soluciónaplicando directamente el teorema Pi a las magnitudes (y, x, c1) , obteniendo:

y x c1x 0 1 −2y 1 0 2

=⇒ y = c1/21 x, (3.87)

viendo de esta forma que hemos obtenido una solución particular a nuestra ecuación diferencial por unmétodo pedestre, comprobamos que se verifica .

c1/21 =

c1x2 + c1x2

2xc1/21 x

=2c1x2

2c1/21 x2

. (3.88)

Por lo tanto podemos tomar las sigientes variables

t = x, u(t) =y

x, (3.89)

obteniendo así la siguiente ODE

u′t =12

(1u

+ u

)− u, =⇒ u2 − 1 =

C1

t, (3.90)

que en las variables originales ( y

x

)2− 1 =

C1

x, (3.91)

es la solución anterior.

Page 52: Ecuaciones Diferenciales Mediante Simetrías · vi CONTENTS Lieperomuchísimomássencillodeaplicaryaqueunonodebeaplicarningúnalgoritmoparaobtener las simetrías que inducen dichos

42 CHAPTER 3. ECUACIONES DE PRIMER ORDEN

3. Solución mediante grupos de Lie.

La solución:y = ±

√x2 + x _C1,

Cálculo de las simetrías. Tenemos que resolver la siguiente ecuación en derivadas parciales:

ηx +12

(ηy − ξx) (x2 + y2)

x y− 1

4

ξy (x2 + y2)2

x2 y2 − ξ(x, y) (1y− x2 + y2

2 x2 y) − η(x, y) (

1x− x2 + y2

2 x y2 ) = 0

las posibles soluciones a esta ecuación son:[

ξ = 0, η =x

2 y

],[

ξ = 0, η = − x2

2 y+

y

2

], [ξ = x, η = y] ,

[ξ = 1, η =

y

2 x

],[ξ = x2 + y2, η(x, y) = 2xy

]

Por ejemplo si seguimos la táctica de los invariantes encontramos que el campo X = x∂x + y∂y genera lasiguiente solución::

dx

ξ=

dy

η=⇒

dx

x=

dy

y=⇒ ln x = ln y =⇒ y = ax

esta sería la solución que nos sugiere precisamente el uso directo del teorema Pi. Vemos que sólo obten-emos una solución particular, pero que ésta es invariante, de hecho si pensamos en la ode como un sistemadinámico vemos que el punto fijo de dicho sistema sería precisamente la solución y = ±x.

De esta forma vemos el alcance de cada método. Veamos ahora la solución que sugiere las variablescanónicas (v.c.).

Las variables canónicas que obtenemos con el campo X = ∂x + x2y ∂y son:

(t =

y√x

, u(t) = x

)=⇒

(x = u(t), y = t

√u(t)

)

por lo tanto la ode original queda expresada en estas nuevas variables como:

u′ = 2t =⇒ u(t) = t2

deshacemos el c.v. obteniendo

x =y2

x+ C1

que es precisamente la solución que hemos ido obteniendo hasta ahora. Veamos por último las variablescanónicas que induce la simetría X = x∂x + y∂y,

(t =

y

x, u(t) = ln x

)=⇒

(x = eu(t), y = teu(t)

)

por lo tanto la ode original queda expresada en estas nuevas variables como:

u′ = − 2t

t2 − 1=⇒ u(t) = − ln (−1 + t) − ln (1 + t) + C1

deshacemos el c.v. obteniendo

ln x = − ln( y

x− 1

)− ln

( y

x+ 1

)+ C1

ln x = C1 + ln(

x

(y − x) (y + x)

).

El cv que induce la simetría X = x2y ∂y, es parecido al obtenido mediante el MD i.e. t = x, u(t) = y2

x .

Page 53: Ecuaciones Diferenciales Mediante Simetrías · vi CONTENTS Lieperomuchísimomássencillodeaplicaryaqueunonodebeaplicarningúnalgoritmoparaobtener las simetrías que inducen dichos

3.5. ECUACIÓN HOMOGÉNEA 43

Ejemplo 3.5.2 Resolver la ecuación: (y + xy′

)2= 4x2y′ (3.92)

Solución. Atacaremos el problema por tres métodos:

1. Método tradicional.

Haciendo el c.v. u = y/x entonces

u′ =2(1 − u ±

√1 − u)

x=⇒ du

(1 − u ±√

1 − u)=

2dx

x

efectuando el c.v.1 − u = z, entonces:∫

dz

z ±√z

= −∫ 2dx

x=⇒ ln |z − 1| ∓ ln

∣∣∣∣√

z − 1√z + 1

∣∣∣∣ = lnc

x2

deshaciendo todos los c.v. obtenemos:

y = ±2C − C2

x

2. Solución mediante AD.

es una ecuación no lineal que no es homogénea dimensionalmente si se discrimina. Sí lo será si introduci-mos una constante [a] = L−1

x Ly (y + xy′

)2= 4ax2y′ (3.93)

Con lo que, desde el A.D., podemos plantear un cambio de variable a través del monomio, eviden-temente adimensional ξ = y

ax del que obtenemos y = axξ , y′ = aξ + axξ ′ que podemos sustituir en (3.93),resultando (

2ξ + xξ ′)2

= 4(ξ + xξ ′

)(3.94)

Resolviendo la ecuación de segundo grado en ξ ′ que hemos obtenido

ξ ′ =2x

[(1 − ξ) ±

√1 − ξ

](3.95)

y una vez más se obtiene la separación de variables, que permite la integración:

d (1 − ξ)

(1 − ξ) ±√1 − ξ

= − 2x

dx =⇒ dz

z ±√z

= lnc

x2 (3.96)

ln |z − 1| ∓ ln∣∣∣∣√

z − 1√z + 1

∣∣∣∣ = lnc

x2 (3.97)

En definitiva se obtienen 2 soluciones con sentido físico, para los dos signos,

c

x2 =(√

z − 1)2 7−→ z =

(1 +

√c

x

)2

, ξ = 1 −(

1 +C

x

)2

=⇒ y = −2aC − aC2

x(3.98)

c

x2 =(√

z + 1)2 7−→ z =

(√c

x− 1

)2

, ξ = 1 −(

C

x− 1

)2

=⇒ y = 2aC − aC2

x(3.99)

Las soluciones puramente matemáticas son

y = ±2C − C2

x. (3.100)

Page 54: Ecuaciones Diferenciales Mediante Simetrías · vi CONTENTS Lieperomuchísimomássencillodeaplicaryaqueunonodebeaplicarningúnalgoritmoparaobtener las simetrías que inducen dichos

44 CHAPTER 3. ECUACIONES DE PRIMER ORDEN

Vemos la alternativa pedestre: (y + xy′

)2= 4ax2y′

esta ecuación es d.h. sii [a] = x−1y, por lo tanto la solución que sugiere el AD (método pedestre(m.p.)) es:

y a xx 0 −1 1y 1 1 0

=⇒ y = ax,

comprobamos que se trata de una solución, como y′ = 2ax entonces:

(ax + xa)2 = 4ax2(a)

por lo tanto es solución. Este monomio induce el siguiente cv.

t = x, u(t) =y

x,

entonces (2u + tu′)2

= 4(u + tu′) ,

i.e. lo mismo pero más sencillo y claro.

3. Método de Lie.

La ecuación es homogenea ,D’alembert .Tenemos que resolver la siguiente ecuación

ηx +(ηy − ξx) (2 x − y − 2

√x2 − y x)

x−

ξy (2 x − y − 2√

x2 − y x)2

x2

−ξ(x, y)

2 − 2 x−y√x2−y x

x− 2x − y − 2

√x2 − yx

x2

η(x, y) (−1 + x√x2−yx

)

x= 0

cuyas soluciones son:

[ξ = 0, η =

√x (x − y)

x], [ξ = 0, _η = x − y +

√x (x − y)],

[ξ = 0, η = −2 x2 + 2 y x − 2√

x (x − y) x + y√

x (x − y)],

[ξ = x, η = y], [ξ = 1, η = 2 − y

x],

[ξ = x2 (4 x − 3 y), η = y2 x], [ξ = x (2 x − y), η = y2]

y la soluciones son:

y = x, y =C1(−C1 + 2 x)

x, y = −C1(−C1 + 2x)

x

Vemos que una de las posibles simetrías de la ecuación son las de escala generadas por [ξ = x, η = y], estasimetría genera la solución que hemos obtenido mediante AD (m.p.). Vemos las diferencias entre ambosmétodos.

Las variables canónicas nos llevan a los siguientes resultados: El campo X =

√x (x−y)

x ∂y induce el siguientecambio de variables;

(t = x, u(t) = −2

√x (x − y)

)=⇒

(x = t, y =

14

4t2 − u(t)2

t

)

la ode queda:u′ = ±2 =⇒ u(t) = 2t + C1

si deshacemos el c.v. obtenemos−2

√x (x − y) = 2x + C1

Page 55: Ecuaciones Diferenciales Mediante Simetrías · vi CONTENTS Lieperomuchísimomássencillodeaplicaryaqueunonodebeaplicarningúnalgoritmoparaobtener las simetrías que inducen dichos

3.5. ECUACIÓN HOMOGÉNEA 45

Ejemplo 3.5.3 Estudiar la ecuación

y′ =y

x+

√y2

x2 − 1 (3.101)

Solución.

1. Método tradicional.

Haciendo el c.v. u = y/x entonces

u′ =

√u2 − 1

x=⇒ du√

u2 − 1=

dx

x

cuya solución es:ln x − ln(u +

√u2 − 1) + C1 = 0

si expresamos esta solución en función de las variables originales, entonces obtenemos:

x = C1

√(y2 (x)

x2 − 1)

+C1

xy (x) .

2. Solución mediante AD.

Esta ode no será d.h. si entendemos que el número 1 dentro de la ecuación es precisamente un númeropuro. Con la experiencia anterior podemos darle las dimensiones de una constante [A] = L−2

x L2y , obtener el

monomio adimensional ξ = x2 Ay2 , y de nuevo hacer el cambio de la variable dependiente y de su derivada:

y = x

(A

ξ

)1/2

, y′ =

(A

ξ

)1/2

− 12

A1/2

ξ3/2 ξ ′ (3.102)

sustituimos y simplificamos

(A

ξ

)1/2

− x

2A1/2

ξ1/2ξ ′

ξ=

(A

ξ

)1/2

+

√A

ξ− A =⇒ (3.103)

− x

2ξ ′

ξ=

√1 − ξ =⇒ dξ

ξ√

1 − ξ= −2

dx

x(3.104)

que integrando

ln∣∣∣∣√

1 − ξ − 1√1 − ξ + 1

∣∣∣∣ = ln1x2 + ln

∣∣∣∣−1c2

∣∣∣∣ (3.105)

es decir √1 − ξ − 1√1 − ξ + 1

= − 1c2x2 (3.106)

y obtenemos la solución de (3.101):

ξ = 1 −(

1 − x2c2

1 + x2c2

), (3.107)

y por lo tanto:

y = ±(√

A

2c+

x2c√

A

2

)i.e. y = ±

(12c

+x2c

2

)(3.108)

Page 56: Ecuaciones Diferenciales Mediante Simetrías · vi CONTENTS Lieperomuchísimomássencillodeaplicaryaqueunonodebeaplicarningúnalgoritmoparaobtener las simetrías que inducen dichos

46 CHAPTER 3. ECUACIONES DE PRIMER ORDEN

El m.p. en este caso nos conduce a la siguiente solución particular:

y a xx 0 −2 1y 1 2

=⇒ y2 = ax2 =⇒ y = a′x,

se comprueba con facilicidad que esta solución particular verifica la ecuación diferencial original. El c.v.que induce este monomio es el siguiente:

t = x, u(t) =y

x,

y por lo tanto obtenemos la siguiente ODE

u′t =√

u2 − 1, =⇒ ln x − ln

(y

x+

√y2

x2 − 1

)+ c1 = 0.

3. Solución mediante GL.

La ecuacion que debemos resolver es la siguiente:

ηx + (ηy − ξx)(y

x+

√y2

x2 − 1) − ξy

(y

x+

√y2

x2 − 1

)2

−ξ(x, y)

− y

x2 − y2√

y2

x2 − 1x3

− η(x, y)

1

x+

y√y2

x2 − 1 x2

= 0

donde las posibles solucione son:

[ξ = 0, η =

√y2

x2 − 1x], [ξ = 0, η = −x2 + y2 −√

y2

x2 − 1 yx],

[ξ = x, η = y], simetrías de escala

[ξ = −x y, η = x2 − 2 y2], [ξ = x (x2 − 2 y2), η = y (3 x2 − 4 y2)]

la solucion que induce la simetría X = x∂x + y∂y a trasvés de las variables canónicas es:

(t =

y

x, u(t) = ln(x)

)=⇒

(x = eu(t), y = teu(t)

)

u′ =1√

t2 − 1=⇒ u(t) = ln

(t +

√t2 − 1

)+ C1

ln(x) = ln

(y

x+

√y2

x2 − 1

)+ C1. (3.109)

De esta forma vemos las ventajas y desventajas de cada método.

3.6 Ecuación lineal de primer orden

Tiene la formady

dx+ P (x) y = Q (x) (3.110)

Page 57: Ecuaciones Diferenciales Mediante Simetrías · vi CONTENTS Lieperomuchísimomássencillodeaplicaryaqueunonodebeaplicarningúnalgoritmoparaobtener las simetrías que inducen dichos

3.6. ECUACIÓN LINEAL DE PRIMER ORDEN 47

3.6.1 Método tradicional.

Separando variablesy = u(x)v(x) (3.111)

que nos lleva au′v + u

(Pv + v′

)= Q (3.112)

La solución por lo tanto es:

y =

(∫Qe

∫Pdx + C1

)e−

∫Pdx (3.113)

3.6.2 Método dimensional.

La ODE será discriminalmente homogénea (d.h.) si

[P (x)] =[

x−1]

, [Q (x)] =[yx−1

](3.114)

si descomponemos y (x) en el producto y (x) = u (x) v (x) tendremos libertad para forzar a una de ellas, p. e.u (x) , a que satisfaga alguna condición, que convendrá sea d.h.. Así plantearemos una relación entre u, du

dx ,P (x) que no puede ser otra (3.115) que

u dudx P(x)

x 0 −1 −1u 1 1 0

(3.115)

du

dx[P (x) u]−1 = Cte (3.116)

lo más sencillo será hacer la constante igual a −1, resultando

du

dx+ P (x) u = 0 (3.117)

Teniendo en cuenta quedy

dx= u

dv

dx+ v

du

dx(3.118)

sustituiremos en (3.114) obteniendo

v

(du

dx+ uP

)+ u

dv

dt= Q (3.119)

y teniendo en cuenta (3.116) resultará

udv

dx= Q (3.120)

Tanto (3.116) como (3.117) permiten la integración por separación de variables obteniéndose u (x), v (x) y enconsecuencia la solución y (x) = uv de la (3.114).

3.6.3 Método de Lie.

El sistema que se gebera es el siguiente:

ηx + (ηy − ξx) (−Py + Q) − ξy (−Py + Q)2 − ξ(x, y) (−P′y + Q′) − η(x, y)(−P) = 0

la simetría que se genera es: [ξ = 0, η = e−

∫Pdx

]

Page 58: Ecuaciones Diferenciales Mediante Simetrías · vi CONTENTS Lieperomuchísimomássencillodeaplicaryaqueunonodebeaplicarningúnalgoritmoparaobtener las simetrías que inducen dichos

48 CHAPTER 3. ECUACIONES DE PRIMER ORDEN

esta simetría conduce al siguiente cambio de variables:(

t = x, u(t) =y

e−∫

Pdx

)=⇒

(x = t, y =

u(t)

e∫

P(t)dt

)

por lo tanto la ode original se escribe ahora en estas nuevas varibles como:

u′ =Q(t)

e−∫

P(t)dt=⇒ u(t) =

∫Q(t)e

∫P(t)dtdt + C1

reescribiendo esta solución en las variables originales obtenemos que:

y =

(∫Qe

∫Pdx + C1

)e−

∫Pdx

3.6.4 Ejemplos.

Ejemplo 3.6.1 Sea de nuevo la ecuación

y′ = 3y

x+ x.

Solución. Como en ejemplos anteriores estudiaremos este ejemplo empleandovarias tácticas

1. Solución mediante AD.

Reescribimos la ecuación, que no es d.h., pero sí lo será si la ponemos en la forma

dy

dx− 3

y

x= Ax (3.121)

donde a la constante que introducimos, A, la suponemos dimensiones [A] = L−2x Ly . Es inmediato que

con ella podemos construir el monomio adimensional ξ = A x2

y estableciendo la variable dependiente y suderivada

y = Ax2

ξ

dy

dx=

2Axξ − Ax2ξ ′

ξ2 .

Sustituidas en (3.121) resulta2Axξ − Ax2ξ ′

ξ2 − 3Ax

ξ= Ax

Reordenando y simplificando términos obtenemos

ξ ′ = − 1x

[ξ + ξ2

](3.122)

donde es posible la separación de variables e integración inmediata:

ξ (ξ + 1)= −dx

x=⇒ ln

ξ

ξ + 1= ln

c

x=⇒

cAx2

y− x3 A

y+ c = 0 =⇒ y = −Ax2 +

A

cx3 (3.123)

que sería la solución de (3.121), correspondiente a un eventual problema físico, representado por ella. Parala solución de (3.121) basta sustituir A por su valor numérico

y = −x2 + Cx3 (3.124)

Page 59: Ecuaciones Diferenciales Mediante Simetrías · vi CONTENTS Lieperomuchísimomássencillodeaplicaryaqueunonodebeaplicarningúnalgoritmoparaobtener las simetrías que inducen dichos

3.6. ECUACIÓN LINEAL DE PRIMER ORDEN 49

donde C es una constante indeterminada. Esta solución no corresponde a ningún problema físico,evidentemente.

También podemos seguir este otro método dimensional alternativo al anterior. Reescribimos la ecuación(3.75) como

y′ =3y

x+ Ax (3.125)

donde A es cierta constante con dimensiones [A] = yx−2. De esta forma intentamos encontar una soluciónaplicando directamente el teorema Pi a las magnitudes (y, x, c1) , obteniendo:

y x Ax 0 1 −2y 1 0 1

=⇒ y = Ax2 (3.126)

viendo de esta forma que hemos obtenido una solución particular a nuestra ecuación diferencial por unmétodo pedestre. El cv que obetemos es:

t = x, u(t) =y

x2 ,

y por lo tanto

u′ =u + 1

t, u = −1 + C1t,

y

x2 = −1 + C1x.

llegando así al mismo resultado pero de forma mucho más sencilla.

2. Solución mediante GL.

En primer lugar decimos que la ecuación es de tipo lineal y que la solución formal es:

y = −x2 + C1 x3

Tenemos que resolver la ecuacion:

ηx + (ηy − ξx) (3 y

x+ x) − ξy (

3 y

x+ x)2 − ξ(x, y) (−3 y

x2 + 1) − 3 η(x, y)

x= 0

cuya solucion es:

[ξ = 0, η = x3], [ξ = 0, η = y + x2], [ξ = x, η = 2 y],

[ξ = x2, η = 3x y], [ξ = −3 y + x2, η = −9 y2

x]

vemos que la simetría [ξ = x, η = 2 y], induce por le método de los invariantes i.e.

dx

x=

dy

2y=⇒ y = x2,

la misma solución que la obtenida aplicando el teorema Pi.

Las variables canónicas obtenidas por el campo X = x2∂x + 3x y∂y son:(

t =y

x3 , u(t) = − 1x

)=⇒

(x = − 1

u(t), y = − t

u(t)3

),

con estas variables la ode se escribe como:

u′ = 1 =⇒ u(t) = t + C1,

por lo tanto la solución es:

− 1x

=y

x3 + C1 =⇒ y = −x2 + x3C1.

Page 60: Ecuaciones Diferenciales Mediante Simetrías · vi CONTENTS Lieperomuchísimomássencillodeaplicaryaqueunonodebeaplicarningúnalgoritmoparaobtener las simetrías que inducen dichos

50 CHAPTER 3. ECUACIONES DE PRIMER ORDEN

Ejemplo 3.6.2 Sea la ecuación (1 − x2

) dy

dx+ xy = 1 (3.127)

Solución. Atacaremos este ejemplo mediante dos tácticas

1. Solución mediante AD.

Se comprueba fácilmente que no es d.h.. Para conseguir la homogeneidad dimensional debemos introducirdos constantes dimensionadas [A] = L2

x y [B] = LxLy , obteniendo

(A − x2

) dy

dx+ xy = B (3.128)

De la resolución de la matriz (3.129)x y A B

Lx 1 0 2 1Ly 0 1 0 1ξ −1 1 1 −1η 2 0 −1 0

(3.129)

obtenemos las dos variables adimensionales ξ y η, pudiéndose escribir, de acuerdo con el A. D.

ξ =A

B

y

x= f

(x2

A

)= f (η) =⇒ y =

B

Ax f (η) (3.130)

por lo tanto

y′ =B

Af +

B

Ax

d f

dx=

B

A

(f + 2η f ′

)(3.131)

Sustituyendo en (3.128) (A − x2

) B

A

(f + 2η f ′

)+

B

Ax2 f = B (3.132)

Reordenando, con las simplificaciones oportunas, obtenemos la posibilidad de separar variables y su inte-gración. En síntesis:

2η (1 − η) f ′ = 1 − f =⇒ 2d f

1 − f=

η (1 − η)7−→ (3.133)

f = 1 ∓ c

√1 − η

η=⇒ y =

B

Ax ∓ c

B

A

√A − x2 (3.134)

que será la solución de (3.128). Con A = 1, B = 1 , obtenemos la solución de (3.127)

y = x ∓ c√

1 − x2 (3.135)

donde c será una constante arbitraria.

Vemos que hemos obtenido dos monomios pero que ninguno de ellos es solución particular, por lo quedeberemos buscar una combinación que nos de una solución particular, ésta es:

π1 =x2

A, π2 =

yx

B, π2π−1

1 =y

x,

de la que obtenemos el siguiente c.v.

t = x, u(t) =y

x,

Page 61: Ecuaciones Diferenciales Mediante Simetrías · vi CONTENTS Lieperomuchísimomássencillodeaplicaryaqueunonodebeaplicarningúnalgoritmoparaobtener las simetrías que inducen dichos

3.6. ECUACIÓN LINEAL DE PRIMER ORDEN 51

u + tu′ =tu2 − 1t2 − 1

, =⇒ u = 1 +

√(t + 1) (t − 1)

t

y por lo tantoy

x= 1 +

√(x + 1) (x − 1)

x.

2. Solución mediante GL.

La ecuación es lineal y la solucion formal es:

y = x ± c√

x − 1√

x + 1 (3.136)

Tenemos que resolver la siguiente ecuacion en derivadas parciales:

ηx +(ηy − ξx) (1 − x y)

1 − x2 −ξy (1 − x y)2

(1 − x2)2 − ξ(x, y) (− y

1 − x2 +2 (1 − x y) x

(1 − x2)2 ) +η(x, y) x

1 − x2 = 0, (3.137)

cuyas soluciones son:

[ξ = 0, η =

√−1 + x2

],[

ξ = 0, η =√

(x − 1) (x + 1)

],

[ξ = 0, η = −y + x] ,[

ξ = 0, η =(2xy − y2 − 1)(−y + x)

(x − 1)/(x + 1)

](3.138)

que generan las siguientes soluciones:

Por ejemplo con[ξ = 0, η =

√−1 + x2

]obtenemos:

(t = x, u(t) =

y√−1 + x2

)=⇒

(x = t, y = u(t)

√−1 + t2

)

la ode se escribe ahora como:

u′ = −√−1 + t2

1 − 2t2 + t4

cuya solución es:

u(t) = −(−1 + t2)3/2

4 (t + 1)2 +

(−1 + t2)3/2

4 (t − 1)2 + C1

por lo tanto la solución en las variables originales resultará:

y√x2 − 1

=C1(x2 − 1) + x

√x2 − 1

x2 − 1=⇒ y = x ± c

√x − 1

√x + 1.

Otro cambio de variables lo podemos obtener de [ξ = 0, η = −y + x] de tal forma que:

(t = x, u(t) = − ln(−y + x)) =⇒(

x = t, y =teu(t) − 1

eu(t)

)

la ode se escribe ahora como:u′ = − t

−1 + t2

cuya solución es:

u(t) = −12

ln(t − 1) − 12

ln(t + 1) + C1

por lo tanto la solución en las variables originales resultará:

− ln(−y + x) = −12

ln(x − 1) − 12

ln(x + 1) + C1

Page 62: Ecuaciones Diferenciales Mediante Simetrías · vi CONTENTS Lieperomuchísimomássencillodeaplicaryaqueunonodebeaplicarningúnalgoritmoparaobtener las simetrías que inducen dichos

52 CHAPTER 3. ECUACIONES DE PRIMER ORDEN

sólo queda simplificar esta expresión, obteniéndose:

ln1

−y + x= C1 + ln

1√(x − 1)

√(x + 1)

y = x ± c√

x − 1√

x + 1

como ya sabíamos.

3.7 Ecuaciones exactas.

Llamaremos exacta a una ODE del tipo

P(x, y(x))dx + Q(x, y(x))dy = 0, i.e. y′ = − P(x, y(x))

Q(x, y(x))(3.139)

dondeD2(P)(x, y(x)) = D1(Q)(x, y(x)). (3.140)

La forma general de resolverlas es la siguiente. Queremos encontrar una función F(x, y) tal dF = Pdx + Qdy(1-forma de Pfafft) ∫ x

P(x, y)dx +∫ y

Q(x0, y)dy = C.

Entonces la solución será F(x, y) = C1 i.e.

F(x, y(x)) + C1 = 0, (3.141)

dicha función siempre existe si se verifica la condición (3.140). Podemos reescribir la ecuación (3.139) comosigue:

∂xF + ∂yF(y′

)= 0 (3.142)

donde obviamente ∂xF = P y ∂yF = Q.

Lo mejor es que veamos algún ejemplo.

Ejemplo 3.7.1 Integrar la ecuación exacta

(sin(xy) + xy cos(xy)) dx + x2 cos(xy)dy = 0 (3.143)

Solución. Veremos como siempre distintas tácticas

1. Método tradicional. Lo primero es comprobar que realmente es una ode exacta i.e.

∂yP = ∂xQ

viendo de forma inmediata que:

∂yP = ∂y (sin(xy) + xy cos(xy)) = 2 (cos xy) x − x2y sin xy

∂xQ = ∂x

(x2 cos(xy)

)= 2 (cos xy) x − x2y sin xy

Page 63: Ecuaciones Diferenciales Mediante Simetrías · vi CONTENTS Lieperomuchísimomássencillodeaplicaryaqueunonodebeaplicarningúnalgoritmoparaobtener las simetrías que inducen dichos

3.7. ECUACIONES EXACTAS. 53

por lo tanto la ode dada es exacta. Calculamos la función F(x, y) como sigue:

F(x, y) =∫ x

(sin(xy) + xy cos(xy)) dx +∫ y

x20 cos(x0y)dy

por lo tanto la solución es:(sin xy) x = C1

2. Grupos de Lie. En primer lugar calculamos la ecuación con la que podremso obtener los infinitesimales

ηx −(ηy − ξx) (sin(x y) + x y cos(x y))

x2 cos(x y)−

ξy (sin(x y) + x y cos(x y))2

x4 cos(x y)2 −

−ξ (−2 cos(x y) y − x y2 sin(x y)

x2 cos(x y)+

2 (sin(x y) + x y cos(x y))

x3 cos(x y)− (sin(x y) + x y cos(x y)) (x y) y

x2 cos(x y)2 )

−η (−2 cos(x y) x − x2 y sin(x y)

x2 cos(x y)− (sin(x y) + x y cos(x y)) sin(x y)

x cos(x y)2 ) = 0

de esta forma obtenemos que las posibles soluciones a esta ecuación son:

[ξ = 0, η =1

x2 cos(x y)], [ξ = 0, η =

−1 + cos(2x y)

cos(x y)]

[ξ = 0, η =sin(x y)

x cos(x y)], [ξ = −x, η = y]

de esta forma vemos que por ejemplo el campo X = −x∂x + y∂y induce el siguiente cambio de varibles:

(t = xy, u(t) = − ln x) =⇒(

x =1

eu(t), y = teu(t)

)

por lo tanto la ecuación (3.143) se reescribe como:

u′ =cos t

sin t=⇒ u(t) = ln(sin t) + C1

deshaciendo el cambio de variables obtenemos:

− ln x = ln(sin(xy)) + C1 =⇒ sin(xy)x = C1

El método de los invariantes nos llevaría a buscar una solución del tipo

−dx

x=

dy

y=⇒ yx = C1

que obviamente no es solución, cualquiera de las otras simetría conduciría a una solución del tipo y =const.

Algunas ecuaciones no verifican la relación ∂yP = ∂xQ, sin embargo se puede intentar buscar cierta función(factor integrante) µ(x, y) que haga que la ecuación

µ (P(x, y)dx + Q(x, y)dy) = 0

sea exacta. Dicho factor lo deberemos buscar de la siguiente manera.

Page 64: Ecuaciones Diferenciales Mediante Simetrías · vi CONTENTS Lieperomuchísimomássencillodeaplicaryaqueunonodebeaplicarningúnalgoritmoparaobtener las simetrías que inducen dichos

54 CHAPTER 3. ECUACIONES DE PRIMER ORDEN

Como∂y (µP) = ∂x (µQ) ,

entoncesQ∂x (µ) − P∂y (µ) =

(∂yP − ∂xQ

)µ,

por lo tantoQ∂x ln µ − P∂y ln µ = ∂yP − ∂xQ,

observándose que si∂yP − ∂xQ

Q= h(x) =⇒ µ(x) = e(

∫h(x)dx),

mientras que si−∂yP + ∂xQ

P= h(y) =⇒ µ(y) = e(

∫h(y)dy).

El siguiente teorema estable la relación entre el factor integrante y las simetrías de una ode de primer orden.

Teorema 3.7.1 (Factor integrante de Lie) . La ecuación P(x, y)dx + Q(x, y)dy = 0 admite un grupo uniparametrícoG cuyo generador infinitesimal X es tal X = ξ∂x + η∂y sii

µ =1

ξP + ηQ, ξP + ηQ 6= 0, (3.144)

es un factor integrante de la ecuación dada.

Veamos el siguiente ejemplo.

Ejemplo 3.7.2 Integrar la ecuación: (x + y2

)dx − 2xydy = 0

Solución. Como siempre veremos al menos dos tácticas:

1. Método tradicional. Vemos que ∂yP 6= ∂xQ ya que

∂yP = 2y y ∂xQ = −2y

como en nuestro caso se tiene que

∂yP − ∂xQ

Q=

12x

= h(x) =⇒ µ(x) = e(∫ 1

2x dx) =1x2

entonces1x2

((x + y2

)dx − 2xydy

)= 0

es exacta y su solución es:

x = C1ey22 .

2. Método de Lie. En primer lugar calcularemos la ecuación

ηx +12

(ηy − ξx)(x + y2)

x y− 1

4

ξy (x + y2)2

x2 y2 − ξ(1

2 x y− x + y2

2 x2 y) − η (

1x− x + y2

2 x y2 ) = 0

Page 65: Ecuaciones Diferenciales Mediante Simetrías · vi CONTENTS Lieperomuchísimomássencillodeaplicaryaqueunonodebeaplicarningúnalgoritmoparaobtener las simetrías que inducen dichos

3.8. ECUACIÓN DE BERNOULLI 55

cuyas soluciones son:

[ξ = 0, η =x

2 y], [ξ =

x2

x + y2 , η = 0], [ξ = 2x, η = y]

si por ejemplo nos fijamos en la simetría X = 2x∂x + y∂y , vemos que nos conduce al siguiente cambio devariables (variables canónicas);

(t =

y√x

, u(t) =12

ln x

)=⇒

(x = e2u(t), y = t

√e2u(t)

)

la ode original se escribe en estas variables como

u′ = t =⇒ u(t) =t2

2+ C1

que en las variables originales se escribe como:

12

ln x =y2

2x+ C1 =⇒ x = C1e

y22 .

Si atendemos al teorema del factor integrante de Lie, vemos que con [ξ = x2

x+y2 , η = 0] obtenemos

µ =1

ξP + ηQ=

1x2

x+y2 (x + y2)=

1x2

tal y como habíamos calculado antes.

Observamos que la solución que induce la simetría X = 2x∂x + y∂y por el método de los invariantes es:

y = C1√

x

que NO es solución de la ode dada.

3.8 Ecuación de Bernoulli

Se escribey′ + P (x) y = Q (x) yn (3.145)

o alternativamente:y′ = fn(x)yn + f1(x)y (3.146)

3.8.1 Método tradicional.

Se pueden seguir varias tácticas como las siguientes:

El cambio de variabley(x) = u(t)(

11−n ), x = t (3.147)

que conduce a la siguiente ecuación diferencial

u′(t) =(

u(t)P(t) + Q(t)(

u(t)(1

1−n ))n

u(t)(n

1−n ))

(n − 1) (3.148)

Page 66: Ecuaciones Diferenciales Mediante Simetrías · vi CONTENTS Lieperomuchísimomássencillodeaplicaryaqueunonodebeaplicarningúnalgoritmoparaobtener las simetrías que inducen dichos

56 CHAPTER 3. ECUACIONES DE PRIMER ORDEN

que es una ode de tipo lineal.

Un segundo método sugiere hacery(x) = u(x)v(x)

variables separadas. Se iguala a cero el coeficiente en u (v′ + f1v = 0) determinando así v, apareciendo ahorauna ecuación en u(x) de variables separadas.

La solución queneral de una ode tipo Bernoulli viene dada por el siguiente formulón:

y =e(

1n−1

∫P(x)dx)

(−n

∫ e∫

P(x)dxQ(x)

e(n∫

P(x)dx)dx +

∫ e∫

P(x)dxQ(x)

en∫

P(x)dxdx + C1

) 1n−1

en

∫P(x)dxnn−1

(3.149)

o de forma algo más compacta:

y1−n =(1 − n)

∫Q(x)e(n−1

∫−P(x)dx) + C1

e(n−1∫−P(x)dx)

(3.150)

3.8.2 Método dimensional.

Para su estudio dimensional conviene dividir por el segundo miembro, resultando

1Qyn

dy

dx+

P

Qyn−1 = 1 (3.151)

Las dimensiones de PQ serán [

P

Q

]=

[yn−1

](3.152)

y para que el segundo término resulte adimensional procederá tomar como nueva variable

t = y1−n (3.153)

si también se desea que sea lineal en t.

Para eliminar dydx derivaremos (3.153) respecto x

dt

dx=

1 − n

yn

dy

dx(3.154)

y de esta ecuación y la (3.153) resultará

− 1(n − 1)

dt

dx+ P (x) t = Q (x)

que es una ecuación de primer orden lineal en t, de cuya solución hemos tratado en el apartado anterior.

3.8.3 Método de Lie.

Tenemos que resolver la siguiente ecuación etc...

ηx + (ηy − ξx) (−p(x) y + q(x) yn) − ξy (−p(x) y + q(x) yn)2 − ξ(x, y) (−p′ y + q′ yn)−

−η(x, y) (−p(x) +q(x) yn n

y) = 0

Page 67: Ecuaciones Diferenciales Mediante Simetrías · vi CONTENTS Lieperomuchísimomássencillodeaplicaryaqueunonodebeaplicarningúnalgoritmoparaobtener las simetrías que inducen dichos

3.8. ECUACIÓN DE BERNOULLI 57

cuyas posibles soluciones son:

[ξ = 0, η = e(∫

p(x) (−1+n) dx) yn],

[ξ =

e(∫

p(x) dx (−1+n))

q(x), η = −yp(x) e(

∫p(x) dx (−1+n))

q(x)

]

el cambio de variables que induce la primera de las simetrías es:(

u(t) =y(1−n)

(1 − n)e((−1+n)∫

p(x) dx), t = x

)

por lo tanto la transformación inversa es:y = e

(− ln(u(t) e(−

∫p(t) dt+n

∫p(t) dt)−u(t) e(−

∫p(t) dt+n

∫p(t) dt) n)

−1+n

)

, x = t

la ecuación original queda por lo tanto:

ut = −q(t)

((−u(t) e(

∫p(t) dt (−1+n)) (−1 + n))(− 1

−1+n ))n

(−1 + n) u(t)

(−u(t) e(∫

p(t) dt (−1+n)) (−1 + n))(− 1−1+n )

cuya solución es:u(t) − C1−

−∫ t

−q(b)((−u(t)e((n−1)

∫p(b) db )(n − 1))(− 1

n−1 ))n

(n − 1) u(t)(−u(t)e((n−1)

∫p(b)db)(n − 1)

)( 1−1+n )

db = 0

deshaciendo el cambio de variables obtenemos por lo tanto nuestro formulón (3.150):

También podemos probar a calcular el factor integrante de Lie siendo éste:

µ := e(−∫

p(x) dx (−1+n)) y(−n)

y por lo tanto la solución de la ode exacta resultante es:

e(−∫

p(x) dx (−1+n)) y(1−n)

1 − n−

∫q(x) e(

∫p(x) dx−n

∫p(x) dx) dx + C1 = 0

Veamos algunos ejemplos.

3.8.4 Ejemplos.

Ejemplo 3.8.1 Resolver la ecuación:

xy′ + y = y2 ln x

Solución. En este ejemplo veremos dos táctics tanto la tradicional como la de Lie, ya que el método dimensionalno funciona por haber funciones del tipo (ln x) .

1. Método tradicional nos lleva a efectuar el siguiente cambio de variables y = uv de donde

xu′v + u(

xv′ + v)

= u2v2 ln x

Page 68: Ecuaciones Diferenciales Mediante Simetrías · vi CONTENTS Lieperomuchísimomássencillodeaplicaryaqueunonodebeaplicarningúnalgoritmoparaobtener las simetrías que inducen dichos

58 CHAPTER 3. ECUACIONES DE PRIMER ORDEN

resolviendo(

xv′ + v)

= 0 =⇒ v =1x

entonces

u′ =u2

x2 ln x =⇒ − 1u

=ln x

x− 1

x− C1

por lo tanto

u =x

1 + C1x + ln x⇐⇒ y =

11 + C1x + ln x

2. Con el método de Lie tendremos que resolver la siguiente ecuación en derivadas parciales (edp):

ηx +(ηy − ξx) (−y + y2 ln(x))

x−

ξy (−y + y2 ln(x))2

x2 − ξ (y2

x2 − −y + y2 ln(x)

x2 ) − η (−1 + 2 y ln(x))

x= 0

cuyas soluciones son:

[ξ = 0, η = y2 x], [ξ = 0, η = y (−1 + y ln(x) + y)],

[ξ = 0, η =(−1 + y ln(x) + y)2

x], [ξ = −x, η = y2]

la primera de las simetrias nos lleva a obtener las siguientes v.c.(

t = x, u(t) = − 1y x

)=⇒

(x = t, y = − 1

u(t) t

)

la ode original queda en estas variables como:

ut =ln(t)

t2 =⇒ u(t) = − ln(t)

t− 1

t+ C1

que en las variables originales queda

− 1y x

= − ln(x)

x− 1

x+ C1

tal y como queríamos hacer ver.

Ejemplo 3.8.2 Resolver la ecuación:

y′ = − y

x + 1− (x + 1)3 y2

2

Solución. En este caso sólo utilizaremos la táctica de lso grupos de Lie.

1. El método de Lie nos lleva a la siguiente edp

ηx + (ηy − ξx) (− y

x + 1− (x + 1)3 y2

2)−

−ξy (− y

x + 1− (x + 1)3 y2

2)2 − ξ (

y

(x + 1)2 − 3 (x + 1)2 y2

2) − η (− 1

x + 1− (x + 1)3y) = 0

Page 69: Ecuaciones Diferenciales Mediante Simetrías · vi CONTENTS Lieperomuchísimomássencillodeaplicaryaqueunonodebeaplicarningúnalgoritmoparaobtener las simetrías que inducen dichos

3.8. ECUACIÓN DE BERNOULLI 59

cuyas soluciones son:

[ξ = 0, η =(x + 1) y2

2], [ξ = 0, η =

(4 y x3 + 6 y x2 + 3 y x + y x4 − 6)2

x + 1]

[ξ = 0, η =(y x4 + 4 y x3 + 6 y x2 − 6 y x − 6 − 9 y) y

2], [ξ = x + 1, η = −4 y]

por lo tanto el método de las variables canónicas nos conduce a (X = (x+1) y2

2 ∂y):

(t = x, u(t) = − 2

y (x + 1)

)=⇒

(y = − 2

u(t) (t + 1), x = t

)

ut = −t2 − 2 t − 1 =⇒ u(t) = −13

t3 − t2 − t + C1

la solución en las variables originales resulta:

− 2y (x + 1)

= −13

x3 − x2 − x + C1,

tal y como queríamos ver.

2. Método dimensional. Reescribiendo la ODE de la siguiente forma:

y′ = − y

x + a− b (x + a)3 y2

2,

donde claramente, [a] = x, [b] = x−4y−1, podemos probar con el siguiente cv (observando además la formade la ODE)

t = x, u(t) = (x + 1)4y,

donde (x + 1)4y = 1, es solución particular de la ODE. De esta forma obtenemos

(t + 1) u′ = 3u − 12

u2, =⇒ 1u

=16

+C1

(t + 1)3 ,

en las variables originales tendremos

1(x + 1)4y

=16

+C1

(x + 1)3 ,

tal y como querímos hacer ver.

Ejemplo 3.8.3 Resolver la ecuación:xy′ + xy2 − y = 0

Solución. En este caso sólo utilizaremos la táctica de los grupos de Lie.

1. El método de Lie nos lleva a la siguiente edp

ηx + (ηy − ξx)

(−xy2 + y

x

)− ξy

(y − xy2

x

)2

− ξ

(y2

x− y − xy2

x2

)− η

(−2xy + 1x

)= 0

Page 70: Ecuaciones Diferenciales Mediante Simetrías · vi CONTENTS Lieperomuchísimomássencillodeaplicaryaqueunonodebeaplicarningúnalgoritmoparaobtener las simetrías que inducen dichos

60 CHAPTER 3. ECUACIONES DE PRIMER ORDEN

cuyas soluciones son:

[ξ = y, η = −y3], [ξ = 0, η =y2

x]

[ξ = 0, η = −2y + xy2], [ξ = x, η = − y]

[ξ = 0, η = x (xy − 2)2], [ξ =1y− x, η =

1x]

por lo tanto el método de las variables canónicas nos conduce a (X = x∂x − y∂y):

(t = xy, u(t) = ln x) =⇒(

y =t

eu(t), x = eu(t)

)

ut = − 1t (t − 2)

=⇒ u(t) = −12

ln (t − 2) +12

ln (t) + C1

la solución en las variables originales resulta:

ln x = −12

ln (xy − 2) +12

ln (xy) + C1

i.e.y =

2x

x2 + 2C1,

tal y como queríamos ver.

El factor integrante de la ode es:

µ =1y2

Claim 3.8.1 La solución por invariantes que induce el campo X = x∂x − y∂y, es solución de la ode dada, i.e.

y = 2x−1.

2. El método dimensional nos lleva a reescribir la ODE como

y′ = −y2 +y

x,

por lo que sólo habrá que considerar una única constante dimensional. El cv que suguiere el AD es elsiguiente

t = x, u(t) = xy,

donde xy = 1, es solución particular de la ODE. De esta forma obtenemos

−2u + tu′ + u2 = 0, =⇒ 1u

=12

+C1

t2 ,

en las variables originales tendremos1

xy=

12

+C1

x2 ,

tal y como querímos hacer ver.

Ejemplo 3.8.4 Solve the following Bernoulli ode.

y′ = − y

x + 1− (x + 1) y2

2. (3.155)

Page 71: Ecuaciones Diferenciales Mediante Simetrías · vi CONTENTS Lieperomuchísimomássencillodeaplicaryaqueunonodebeaplicarningúnalgoritmoparaobtener las simetrías que inducen dichos

3.8. ECUACIÓN DE BERNOULLI 61

Observación 3.8.1 Historically it was Bernoulli the first person who introduced c.v. in order to solve odes (now bearinghis name). He managed to reduce this equation to a simpler linear equation.

Solution. It is observed that with[a] = X, [b] = X−2Y−1, (3.156)

eq. (3.155) yields d.h. i.e.

y′ = − y

x + a− b (x + a) y2

2, (3.157)

in this way it is obtained the following variables(

t =x

a, u(t) =

1abxy

)=⇒

(x = at, y =

1a2btu

), (3.158)

with this change of variables eq. (3.155) yields

2u + 2t (1 + t) u′ = (t + 1)2 , (3.159)

which is linear and its solution is:

u =

(t

2+ C1

) (1 +

1t

), (3.160)

and therefore in the original variables the solution yields

y =2

(bx + C1) (x + a), (3.161)

in this case D.A. does not bring us to obtain a good change of variables but helps us to obtain a simpler ode thanthe original one. We can try now with the following c.v.

(t = x, u(t) =

1x2y

)=⇒

(x = t, y =

1t2u

), (3.162)

which brings us to obtain a linear ode

t(t + 1)(2u′t + 4u

)= 2t2u + t2 + 2t + 1, (3.163)

but with this tactic we are not able to obtain a simpler ode.

As we can see D.A. induces us to the c.v. 1abxy but y =

1x

or y =1x2 , are not particular solutions of (3.155).

Nevertheless solutions as y =1

x + 1or y =

1

(x + 1)2 are particular and invariant solutions for this reason these

c.v. induce us to obtain an ode in separate variables, but unfortunately D.A. is unable to construct such c.v.. Forthis reason we can only reduced our ode to a linear ode as in the theoretical case i.e. employing the theoreticalmethod purposed by Bernoulli.

Observación 3.8.2 As in the above example if we consider the following cv then we get

t = x, u(t) = (x + 1)y,

where (x + 1)y = 1, is a particular solution. Thus we get

u′ = −12

u2, =⇒ 1u

=t

2+ C1,

in the original variables we obtain1

(x + 1)y=

x

2+ C1,

as it is expected.

Page 72: Ecuaciones Diferenciales Mediante Simetrías · vi CONTENTS Lieperomuchísimomássencillodeaplicaryaqueunonodebeaplicarningúnalgoritmoparaobtener las simetrías que inducen dichos

62 CHAPTER 3. ECUACIONES DE PRIMER ORDEN

If we study this ode under the Lie group method, it is observed that eq. (3.155) admits the following symmetriesobtained from:

ηx + (ηy − ξx)

(− y

x + 1− (x + 1) y2

2

)− ξy

(− y

x + 1− (x + 1) y2

2

)2

− ξ

(y

(x + 1)2 − 12

y2

)− η

(−y − xy − 1

x + 1

)= 0 (3.164)

X1 =y2 (x + 1)

2∂y, X2 = (x + 1) ∂x − 2y∂y, X3 = ∂x −

y

(x + 1)∂y, X4 = x∂x −

(1 + 2x) y

(x + 1)∂y,

(3.165)and which respective invariants are:

I1 = x, I2 = y (x + 1)2 , I3 = y (x + 1) , I4 = yx (x + 1) ,

that induces the following canonical variables. For example from X1 :(

t = x, u(t) = − 2y (x + a)

)=⇒

(x = t, y = − 2

u (t + a)

), (3.166)

in such a way that eq. (3.155) yieldsu′ = −b =⇒ y = bt + C1, (3.167)

and in the original variables i.e. (x, y) this solution yields

y =2

(bx + C1) (x + a), (3.168)

as we already know.

3.9 Ecuaciones tipo Riccati.

Este tipo de ecuaciones tine la siguiente pinta:

y′ = f2y2 + f1y + f0, (3.169)

donde las ( fi)2i=0 = fi(x) i.e. son funciones que sólo dependen de la variable independiente x. Para resolverlas

por los métodos tradicionales es necesario conocer una solución particular yp (lo cual no siempre es fácil deencontrar). Si éste es el caso entonces haciendo el cambio de función y = yp + z entonces la ode (3.169) secombierte en una de tipo Bernoulli con n = 2.

Se conocen las siguientes condiciones de integrabilidad:

1. ∑i fi = 0

2. Existen (di)2i=0 ∈ R

d1 f2 + d1d2 f1 + d22 f0 = 0

3. Sean α(x), β(x) funciones tales que

α2 f2 + αβ f1 + β f0 = αβ

(log

α

β

)′

En general es bastante complicado obtener una solución teórica de este tipo de ecuaciones por eso sólo veremosunos cuantos ejemplos empleando exclusivamente el método de Lie.

Page 73: Ecuaciones Diferenciales Mediante Simetrías · vi CONTENTS Lieperomuchísimomássencillodeaplicaryaqueunonodebeaplicarningúnalgoritmoparaobtener las simetrías que inducen dichos

3.9. ECUACIONES TIPO RICCATI. 63

3.9.1 Ejemplos.

Ejemplo 3.9.1 Solve the following Riccati ode

y′ = xy2 − 2y

x− 1

x3 . (3.170)

Solution. It is observed that introducing the following dimensional constants eq. (3.170) verifies the principle ofdimensional homogeneity (p.d.h.),

y′ =xy2

a− 2y

x− a

x3 , (3.171)

where [a] = X2Y. As we can see if we need only one constant then our equation is scale invariant and thereforethis constant indicates us that the generator of this symmetry is X = −2x∂x + y∂y as we will see below. Thereforewe have the following variables

(t = x, u(t) =

a

yx2

)=⇒

(x = t, y =

a

tu2

), (3.172)

with this new variables eq. (3.170) yields

u′

u2 − 1=

1t

=⇒ ln t + arctanh(u) + C1 = 0, (3.173)

and in the original variables this solutions is written as:

ln x + arctanh(a

yx2 ) + C1 = 0. (3.174)

We would like to emphasize that in this case, as we have only one dimensional constant, we can find the partic-ular solution y = ax−2 applying the Pi theorem, and that such solution (invariant particular solution) is inducedby the scale symmetry.

Eq. (3.170) has the following symmetries:

ηx + (ηy − ξx)(x y2 − 2 y

x− 1

x3 ) − ξy(xy2 − 2 y

x− 1

x3 )2 − ξ (y2 +2 y

x2 +3x4 ) − η

(2xy − 2

x

)= 0 (3.175)

and hence

X1 =1x

∂x −2x4 ∂y, X2 = x4 (y +

1x2 )2∂y, , X3 = x∂x − 2y ∂y

X4 =

(− 1

x2 + y2 x2)

∂y, X5 = x3∂x −(

2 + 4y x2)

∂y. (3.176)

and which corresponding invariants are:

I1 =yx2 − 1

x2 , I2 = I4 = x, I3 = yx2, I5 = yx2(

x2 + 1)

. (3.177)

The method of canonical variables brings us to obtain the following ode, for example, for the symmetry X5, wehave (

t = x2 (y x2 + 1), u(t) = − 12 x2

), (3.178)

ut =1t2 =⇒ u(t) = −1

t+ C1, (3.179)

Page 74: Ecuaciones Diferenciales Mediante Simetrías · vi CONTENTS Lieperomuchísimomássencillodeaplicaryaqueunonodebeaplicarningúnalgoritmoparaobtener las simetrías que inducen dichos

64 CHAPTER 3. ECUACIONES DE PRIMER ORDEN

and hence

− 12 x2 = − 1

x2 (y x2 + 1)+ C1, (3.180)

as we already know.

The transformation induced by X3 (scaling symmetry) is the following one:

(t = y x2, u(t) = ln(x)

)=⇒

(x = eu(t), y =

t

(eu(t))2

), (3.181)

ut =1

t2 − 1=⇒ u(t) = −arctanh(t) + C1, (3.182)

thereforeln(x) = −arctanh(y x2) + C1, (3.183)

which looks a little different than the other solution.

Ejemplo 3.9.2 Solve the following Riccati ode:

y′ =y + 1

x+

y2

x3 . (3.184)

Solution. The ode verifies the p.d.h. if we introduce the following dimensional constant:

y′ =y + a

x+ b

y2

x3 , (3.185)

where [a] = Y, [b] = X2Y−1. In this way it is obtained the c.v.

(t =

ax2

by2 , u(t) =y

a

)=⇒

(x =

√abtu2, y = au

), (3.186)

and therefore eq. (3.184) yieldsu

u′ + t =ut

(ut + t + 1), (3.187)

which is a Bernoulli ode and its solution is:

1u−√

t(

arctan√

t + C1

)= 0, (3.188)

hence in the original variables the solution is

a

y−

√ax2

by2

(arctan

√ax2

by2 + C1

)= 0. (3.189)

It is observed that we have the particular solution y = Cx, obtained from the relationship ax2

by2 .

Since this c.v. is not very good for our purposes we may now proceed as follows:

axy′ = ay + 1 + b2 y2

x2 (3.190)

where[a] = y−1, [b] = y−1x (3.191)

Page 75: Ecuaciones Diferenciales Mediante Simetrías · vi CONTENTS Lieperomuchísimomássencillodeaplicaryaqueunonodebeaplicarningúnalgoritmoparaobtener las simetrías que inducen dichos

3.9. ECUACIONES TIPO RICCATI. 65

in such a way that these constants induce the following change of variables:(

t =a

bx, u(t) =

x

by

)=⇒

(x =

b

ax, y =

t

au(t)

)(3.192)

which brings us to obtain

− u′

u2 + 1=

1t2 =⇒ −1

t+ arctan(u) + C1 = 0 (3.193)

and hence

− b

ax+ arctan(

x

by) + C1 = 0 (3.194)

which is a better approximation. In the same way we can obtain a quadrature simply considering t =b

axand

u(t) = xby . in this way it is obtained u′ = u2 + 1.

The Lie method bring us to solve the following pde

ηx + (ηy − ξx)(y + 1

x+

y2

x3 ) − ξy(y + 1

x+

y2

x3 )2 − ξ(−y + 1x2 − 3 y2

x4 ) − η(1x

+2 y

x3 ) = 0, (3.195)

and which solutions are:

X1 =

(x +

y2

x

)∂y, X2 =

((ycos( 1

x ) + xsin( 1x ))2

x

)∂y

X3 =

(− sin( 1

x )2 (y sin( 1x ) − cos( 1

x ) x)2

x (−1 + cos( 1x )2)

)∂y, X4 = x2∂x + xy∂y (3.196)

and which invariants are:I1 = I2 = I3 = x, I4 =

y

x. (3.197)

The symmetry X1 =(

x + y2

x

)∂y generates the following c.v.:

(t = x, u(t) = arctan

( y

x

))=⇒ (y = tan(u(t)) t, x = t) , (3.198)

ut =1t2 =⇒ u(t) = −1

t+ C1, (3.199)

and hence the solution is

arctan( y

x

)= − 1

x+ C1, (3.200)

as we already know.

We would like to emphasize that this ode is not scale invariant and nevertheless we have been able to obtain ac.v. that reduces the ode to a quadrature.

Ejemplo 3.9.3 Resolver la ecuación

y′ =(y − x ln x)2

x2 + ln x

Solución. El método de Lie nos lleva a resolver la siguiente edp

ηx + (ηy − ξx)((y − x ln(x))2

x2 + ln(x)) − ξy((y − x ln(x))2

x2 + ln(x))2−

Page 76: Ecuaciones Diferenciales Mediante Simetrías · vi CONTENTS Lieperomuchísimomássencillodeaplicaryaqueunonodebeaplicarningúnalgoritmoparaobtener las simetrías que inducen dichos

66 CHAPTER 3. ECUACIONES DE PRIMER ORDEN

−ξ

(2 (y − x ln(x)) (−ln(x) − 1)

x2 − 2 (y − x ln(x))2

x3 +1x

)− 2 η

(y − x ln(x))

x2 = 0

cuyas soluciones son[

ξ = 0, η =(y − 1

2 (2 ln(x) + 1 +√

5) x)2

x(√

5) x

],

[ξ = 0, η =

x(√

5) (y − 12 (2 ln(x) + 1 −

√5) x)2

x

]

[ξ = 0, η = −x − y +y2

x− 2 y ln(x) + x ln(x)2 + x ln(x)], [ξ = x, η = x + y]

el método de las variables canónicas generadas por la simetría X = x∂x + (x + y) ∂y nos facilita el siguiente c.v.(

t =y − x ln(x)

x, u(t) = ln(x)

)=⇒

(x = eu(t), y = t eu(t) + eu(t) u(t)

)

ut =1

−t − 1 + t2 =⇒ u(t) = −25

√5 arctanh(

(2 t − 1)√

55

) + C1

por lo tanto la solución es:

ln(x) = −25

√5 arctanh(

15

(2 (y − x ln(x))

x− 1)

√5) + C1

tal y como queríamos hacer ver

3.10 Ecuaciones tipo Abel y Chini.

La expresión más general de una ode tipo Abel tiene la siguiente pinta:

y′ = f3y3 + f2y2 + f1y + f0 (3.201)

mientras que la expresión más general para una ode tipo Chini es:

y′ = fnyn − f1y + f0 (3.202)

donde las funciones ( fi)3i=0 = f (x) son función sólo de la variable independiente.

3.10.1 Método tradicional

Hasta el momento (que yo sepa) no existe un método para resolverlas aunque existen casos particulares que sison susceptibles de ser atacados. Para estos casos el método más general hasta ahora es el denominado métodode los invariantes de Abel (método éste debido a Chini).

El invariante de una ode tipo Abel con f2 = 0 es:

− 127

(− f ′0 f3 + f0 f ′3 + 3 f0 f3 f1)3

f 43 f 5

0(3.203)

Si este invariante no depende de x entonces la ode se puede resolver directamente. Para una ode Abel conf2 6= 0, nos podemos cargar este término utilizando la siguiente transformación

y = u(x) − 13

f2

f3. (3.204)

Page 77: Ecuaciones Diferenciales Mediante Simetrías · vi CONTENTS Lieperomuchísimomássencillodeaplicaryaqueunonodebeaplicarningúnalgoritmoparaobtener las simetrías que inducen dichos

3.10. ECUACIONES TIPO ABEL Y CHINI. 67

Si f0 = f1 = 0 y(

f3f2

)′= a f2 entonces en este caso se conoce la solución. Como la ode se queda reducida a

y′ = f3y3 + f2y2, (3.205)

entonces mediante el siguiente cambio de variables

x = t, y =f2(t)r(t)

f3(t), (3.206)

y teniendo en cuenta la condición(

f3f2

)′= a f2 i.e.

f ′3 f2 − f ′2 f3

f 22

= a f2, (3.207)

entonces la ecuación (3.205) se transforma en una ode tipo variables separables

f2r′ f3 − ra f 32

f 23

=f 32 r3 + r2 f 3

2

f 23

. (3.208)

tenemos por lo tnato la siguiente proposición:.

Proposición 3.10.1 Una ode tipo Abel de la forma

y′ = f3y3 + f2y2

puede ser integrada de forma exacta sii las funciones f3 y f2 satisfacen la relación

(f3

f2

)′= K f2 / K = const.

Otro caso es el siguiente pero tiene la desventaja de depender del conocimiento previo de una solución particulary1 además de satisfacerse las siguiente relaciones:

Teorema 3.10.1 Si las funciones ( fi)3i=0 satisfacen la relación

(f3

G(x)

)′= KG(x) / K = const.

tal que

G(x) = ±√

f 22 − 3 f3 f1 = 3 f3y1 + f2

donde y1 es una solución particular de la ode (3.201) entonces la solución viene dada por la siguiente expresión:

y = ± G

f3V +

(− f2 ± G

3 f3

)

con

V =

(f3

3 f3y1 + f2

)(y − y1)

Encontramos las siguientes ecuaciones de tipo ABEL.

Page 78: Ecuaciones Diferenciales Mediante Simetrías · vi CONTENTS Lieperomuchísimomássencillodeaplicaryaqueunonodebeaplicarningúnalgoritmoparaobtener las simetrías que inducen dichos

68 CHAPTER 3. ECUACIONES DE PRIMER ORDEN

1. Segunda especie clase A.

En este caso la ode tiene la siguiente pinta:

(y + g(x)) y′ = f2y2 + f1y + f0 (3.209)

para este caso sif0 = f1g − f2g2 (3.210)

entonces existe solución

y(x) = −g(x) ó (3.211)

y(x) =

(∫(− f2g + f1) e−

∫f2dxdx + C1

)e∫

f2dx (3.212)

también encontramos que sif1 = 2 f2g − g′ (3.213)

la ode entonces se combierte en:

(y + g(x)) y′ = f2y2 +(2 f2g − g′

)y + f0 (3.214)

para este caso se puede buscar o un factor integrante o en su defecto simetrías

X =e2

∫f2dx

y + g∂y (3.215)

encontrándose una solución explícita:

y =12

−2e(−2

∫f2dx)g −−2

√√√√√√e(−2∫

f2dx)g2 + 2e(−2∫

f2dx)

∫f0(

e(∫

f2dx))2 dx + 2e(−2

∫f2dx)C1

(3.216)

2. Segunda especie clase C

En este caso la ode tiene la siguiente pinta:

(g1y + g0) y′ = f3y3 + f2y2 + f1y + f0 (3.217)

La forma de atacar este tipo de odes es la de hacer un cambio de variables(

x = t, y =1

u(t)g1(t)− g0(t)

g1(t)

)(3.218)

qu etrnasforma la ode original en una ode tipo Abel pero de primera clase y se intenta estudiar su tipología.

La expresión más general de una ode tipo Chini es:

y′ = fnyn − f1y + f0 (3.219)

encontrándose que si n = 2 entonces se combierte en una de tipo Riccati y si n = 3 se combierte en una de tipoAbel.

Al igual que en el caso de las ode tipo abel si el invariante de Chini

f−n−1n f−2n+1

0 ( fn f ′0 − f ′n f0 − n f1 fn f0)n

nn(3.220)

es independiente de x entonces la ode se resuleve de manera directa.

En general y por desgracia este tipo de odes no se deja estudiar por la técnica de grupos de Lie ya que no seencuentran soluciones a la edp con la que se obtienen las simetrías. No obstante veremos algunos ejemplillos.

Page 79: Ecuaciones Diferenciales Mediante Simetrías · vi CONTENTS Lieperomuchísimomássencillodeaplicaryaqueunonodebeaplicarningúnalgoritmoparaobtener las simetrías que inducen dichos

3.10. ECUACIONES TIPO ABEL Y CHINI. 69

3.10.2 Ejemplos

Ejemplo 3.10.1 Solve the following Abel ode

y′ =1 − y2

x y+ 1 (3.221)

Solution. Introducing the following dimensional constants, we make that eq. (3.221) verifies the p.d.h.

(xy) y′ = −y2 + bxy + a, (3.222)

where [b] = yx−1, [a] = y2. Therefore the c.v. that suggests the D.A. is the following one:(

t =bxy

a, u(t) =

y2

a

)=⇒

(x =

at

b√

ua, y =

√ua

), (3.223)

hence eq. (3.221) yieldsu′ (2 + t − u) t = 2u (1 + t − u) , (3.224)

and its solution is:

u =t2

−2t + 2 ln(1 + t) − C1, (3.225)

in the original variables it yields:

− b2x2

2a= − bxy

a+ ln(1 +

bxy

a) + C1. (3.226)

Once again we emphasize that the particular solution, y = Cx−1, (in this case invariant solution , see below) hasbeen obtained from the relationship bxy

a .

Alternatively we can try the following c.v. Writing the original Abel ode, eq.(3.221) in the following form:

axyy′ = −y2

a+ xy + b, (3.227)

where [a] = XY−1, [b] = XY, therefore we find the next c.v.(

t =xy

b, u(t) =

x2

ab

)=⇒

(x =

√abu, y =

bt√abu

), (3.228)

that brings us to rewrite eq. (3.227) as follows:

u′ =2t

1 + t=⇒ u = 2t − 2 ln(1 + t) + C1, (3.229)

and hence in the original variables (x, y) we get:

x2

ab= 2

xy

b− 2 ln(1 +

xy

b) + C1, (3.230)

as we already know. We can check that the following c.v. also works well(

t =x2

ab, u(t) =

xy

b

)=⇒

(x =

√abt, y =

bu√abt

), (3.231)

since in these variables eq. (3.227) is written as

2u′u = 1 + u =⇒ t − 2u + 2 ln(1 + u) + C1 = 0, (3.232)

Page 80: Ecuaciones Diferenciales Mediante Simetrías · vi CONTENTS Lieperomuchísimomássencillodeaplicaryaqueunonodebeaplicarningúnalgoritmoparaobtener las simetrías que inducen dichos

70 CHAPTER 3. ECUACIONES DE PRIMER ORDEN

and thereforex2

ab− 2

xy

b+ 2 ln(1 +

xy

b) + C1 = 0, (3.233)

obtaining the solution.

Applying the Lie method we need to solve the following pde:

ηx + (ηy − ξx)(a − y2

x y+ 1) − ξy (

a − y2

x y+ 1)2 +

ξ (a − y2)

x2 y+ η(

2x

+a − y2

xy2 ) = 0, (3.234)

which solution is:

X1 = − 1x

∂x +y

x2 ∂y =⇒ I1 = xy. (3.235)

This symmetry induces the following c.v.:(

u(t) = − x2

2, t = x y

), (3.236)

and therefore in this variables eq. (3.227) is written as follows:

u′ = − t

a + t=⇒ u(t) = −t + a ln(a + t) + C1, (3.237)

hence in the original variables we get:

− x2

2= −x y + a ln(a + x y) + C1, (3.238)

This is another example of an ode that it is not scale invariant, and nevertheless, we have been able to reduce itto a quadrature.

Ejemplo 3.10.2 Solve the following Abel ode.

x (y + 4) y′ = y2 + 2y + 2x. (3.239)

Solution. Following our pedestrian method, we begin by introducing dimensional constants and rewriting theodes as follows

x (y + 4a) y′ = y2 + 2ay + 2bx, (3.240)

where [a] = y, and [b] = x−1y2, therefore D.A. suggest us the following c.v. (note that y =√

x, is a particularsolution of (3.239)) (

t =y2

bx, u(t) =

y

a

)=⇒

(x =

u2a2

ct, y = ua

), (3.241)

in such a way that eq. (3.239) yieldst (t + 4) u′ = t(u + 2) + 2u, (3.242)

which is a linear ode and its solution is:u = t + C1

√t (t + 4), (3.243)

hencey

a=

y2

bx+ C1

√y2

bx

(y2

bx+ 4

), (3.244)

and simplifying, it is obtained the solution:

bx

a= y + C1

√(y2 + 4bx). (3.245)

Page 81: Ecuaciones Diferenciales Mediante Simetrías · vi CONTENTS Lieperomuchísimomássencillodeaplicaryaqueunonodebeaplicarningúnalgoritmoparaobtener las simetrías que inducen dichos

3.10. ECUACIONES TIPO ABEL Y CHINI. 71

Another particular solution could be found from the following relationship y2

bx

( ya

)−1 i.e. y = Cx. But thisparticular solution bring us to the following c.v.

(t =

ay

bx, u(t) =

y

a

)=⇒

(x =

ua2

ct, y = ua

), (3.246)

which transforms eq. (3.239) into a Bernoulli ode,

t2 (u + 4) u′ = (ut + 2t + 2)(u′t − u

), (3.247)

but this situation is not desirable since it is always more difficult to solve a Bernoulli ode than a linear ode.

Applying the Lie method, following the standard procedure, we need to solve the following pde

ηx +(ηy − ξx) (y2 + 2 y + 2 x)

x (y + 4)−

ξy (y2 + 2 y + 2 x)2

x2 (y + 4)2 −

− ξ (2

x (y + 4)− y2 + 2 y + 2 x

x2 (y + 4)) − η(

2 y + 2x (y + 4)

− y2 + 2 y + 2 x

x (y + 4)2 ) = 0, (3.248)

and which solutions are:

X1 =

((2 y + 4 − x) (y − x)

y + 4

)∂y, X2 =

((y2 + 4 x) (y − x)

x (y + 4)

)∂y, X3 =

(4x + x2

)∂x + x (y + 4) ∂y

(3.249)where their correspond invariants are:

I1 = I2 = x, I3 =y + 4x + 4

(3.250)

For example the c.v. that induces X3 is:(

t =y + 44 + x

, u(t) =14

ln(x) − 14

ln(4 + x)

), (3.251)

hence: (y = −4 (t − 1 + e(4 u(t)))

−1 + e(4 u(t)), x = − 4 e(4 u(t))

−1 + e(4 u(t))

), (3.252)

finding that:

u′ =t

2(2t2 − 3t + 1), (3.253)

which solution is:

u(t) =12

ln(t − 1) − 14

ln(2 t − 1) + C1, (3.254)

therefore, in the original variables it yields:

14

ln(x) − 14

ln(4 + x) =12

ln(y + 44 + x

− 1) − 14

ln(2 (y + 4)

4 + x− 1) + C1, (3.255)

as we already know.

Ejemplo 3.10.3 Solve the Abel ode.

y′ = Cx3y3 + Bxy2 − Ay

x, (3.256)

Page 82: Ecuaciones Diferenciales Mediante Simetrías · vi CONTENTS Lieperomuchísimomássencillodeaplicaryaqueunonodebeaplicarningúnalgoritmoparaobtener las simetrías que inducen dichos

72 CHAPTER 3. ECUACIONES DE PRIMER ORDEN

Solution. If we rewrite eq. (3.256) introducing the following dimensional constants,

y′ = a2Cx3y3 + aBxy2 − Ay

x, (3.257)

where A, B, C ∈ R, and [a] = X−2Y−1. As we can see this ode is scale invariant since we have needed introduceonly one constant. D.A. suggests us the following c.v.

(t = x, u(t) = ax2y

)=⇒

(x = t, y =

u

at2

), (3.258)

in such a way that eq. (3.256) is written in the following form:

tu′ = u(

u2 + u + 1)

, (3.259)

and its solution is:

ln t +12

ln(

u2 + u + 1)

+

√3

3arctan

((32

u +13

)√3)− ln u + C1 = 0, (3.260)

therefore in the original variables it yields:

ln x +12

ln((

ax2y)2

+ ax2y + 1)

+

√3

3arctan

((32

(ax2y

)+

13

)√3)− ln

(ax2y

)+ C1 = 0. (3.261)

In second place, we study eq. (3.256)

y′ = Cx3y3 + Bxy2 − Ay

x, (3.262)

with respect to the dimensional base B = T . This ode verifies the principle of dimensional homogeneity with

respect to this dimensional base. Note that [y] =[

1H′

]= T2, and [x] = [H] = T−1 hence [y′] = T3. Therefore

rewriting the equation in a dimensionless way we find that y ∝ x−2

But if we study this equation with respect to the dimensional base B = X, Y , we need to introduce newdimensional constants that make the equation verify the principle of dimensional homogeneity

y′ = αCx3y3 + βBxy2 − Ay

x(3.263)

where[α1/2

]= [β] = X−2Y−1, hence

y β xX 0 −2 1Y 1 −1 0

=⇒ y ∝β

x2 , (3.264)

As we can see we have obtained the same solution than in the case of the invariant solution. This is because theinvariant solution that induces a scaling symmetry is the same as the obtained one through the Pi theorem.

This ode admits the following symmetry (scale-invariant)

X = x∂x − 2y∂y, =⇒ I = x2y (3.265)

which is a scaling symmetry and it induces the following change of variables,

r = x2y, s(r) = ln(x), =⇒ x = es(r), y =r

e2s(r), (3.266)

which brings us to obtain the next ode in quadratures

s′ =1

r (Cr2 + Br + 2 − A), (3.267)

Page 83: Ecuaciones Diferenciales Mediante Simetrías · vi CONTENTS Lieperomuchísimomássencillodeaplicaryaqueunonodebeaplicarningúnalgoritmoparaobtener las simetrías que inducen dichos

3.10. ECUACIONES TIPO ABEL Y CHINI. 73

and which solution is:

s(r) = − ln r

A − 2+

12

ln(Cr2 + Br + 2 − A

)

A − 2−

Barctanh(

2Cr+B√B2+4C(A−2)

)

(A − 2)√

B2 + 4C(A − 2)+ C1, (3.268)

and hence in the original variables (x, y):

ln x = − ln(

x2y)

A − 2+

12

ln(Cx4y2 + Bx2y + 2 − A

)

A − 2−

Barctanh(

2Cx2y+B√B2+4C(A−2)

)

(A − 2)√

B2 + 4C(A − 2)+ C1, (3.269)

which is the most general solution for this ode.

3.10.3 Pathological cases.

In this section we will present two examples of odes that do not admit symmetries (Lie point symmetries).Nevertheless in the first of them D.A. helps us to obtain a simple c.v. that will bring us to obtain a simpler ode(through a particular solution). In the second case we will show that unfortunately sometimes one finds odesthat at this time have no solution, or at least we do not know how to solve them.

Ejemplo 3.10.4 Solve the Abel ode

(x2y + x5 − x

)y′ = xy2 −

(x4 + 1

)y. (3.270)

Solution. Following our pedestrian method we beginn by introducing dimensional constants

ax2yy′ + bx5y′ − cxy′ = dxy2 − ex4y + f y (3.271)

where[a] = [d] = x−2y−1, [b] = [e] = x−5, [c] = [ f ] = x−1, (3.272)

therefore[c5] =

[f 5] = [b] = [e] , having only two dimensional constants. Since y = x−2 is not a particular

solution of eq.(3.270) then we look for a combination between the monomias finding in this way that

π1 =1cx

, π2 =axy

c(3.273)

where y = x−1 is a particular solution of (3.270). The c.v. that induces the D.A. is the following one:

(t = xy, u(t) = x) =⇒(

x = u, y =t

u

), (3.274)

hence using these new variables eq. (3.270) is written now as:

2u′t (t + 1) + u(

1 − t − u4)

= 0, (3.275)

which is a Bernoulli ode and its solution is

u =

√t√−2t − 2 ln(t − 1) + C1√

−2t − 2 ln(t − 1) + C1, (3.276)

Page 84: Ecuaciones Diferenciales Mediante Simetrías · vi CONTENTS Lieperomuchísimomássencillodeaplicaryaqueunonodebeaplicarningúnalgoritmoparaobtener las simetrías que inducen dichos

74 CHAPTER 3. ECUACIONES DE PRIMER ORDEN

undoing the c.v. we find that the solution to eq. (3.270) is:

x =

√xy

√−2xy − 2 ln (xy − 1) + C1√

−2xy − 2 ln (xy − 1) + C1. (3.277)

Applying the Lie method we see that the pde to solve is:

ηx + (ηy − ξx)y

(xy − x4 − 1

)

x (xy + x4 − 1)− ξy

y2 (xy − x4 − 1

)2

x (xy + x4 − 1)2 −

−ξ

(y

(y − 4x3)

x (xy + x4 − 1)− y

(xy − x4 − 1

)

x2 (xy + x4 − 1)− y

(xy − x4 − 1

) (y + 4x3)

x (xy + x4 − 1)2

)−

− η

( (xy − x4 − 1

)

x (xy + x4 − 1)+

y

(xy + x4 − 1)− y

(xy − x4 − 1

)

(xy + x4 − 1)2

)= 0. (3.278)

but in this case we have not found any solution, i.e. eq. (3.270) does not admit symmetries. Nevertheless, onealways may try to find, as if by magic, any c.v. that brings to find a simpler ode.

In this example we would like to show that there are some odes which are very intractable. Our pedestrianmethod does not work in this case, we have not been able to find any particular solution. The Lie method doesnot work, i.e. this ode does not admit any symmetry and the theoretical methods do not work either. At thistime we do not know how to solve it.

Ejemplo 3.10.5 Try to solve the Abel ode

y′ = −Ax2y3 − By2 − y

x, (3.279)

where A and B are dimensional constant.

Solution. In this occasion we already have the dimensional constants

[A] = x−3y−2, [B] = y−1x−1, (3.280)

in such a way that all the terms of the equation has dimensions of yx . We check if one of the dimensional relation-

ship induces any particular solution finding that this is not the case. Therefore we go next to look for any trivialcombination between them but we are not able to find any particular solution. Nevertheless we try to obtainany result with the following c.v.:

(t =

B2

Ax, u(t) =

1Byx

)=⇒

(x =

B2

At, y =

At

B3u(t)

)(3.281)

which bring us to obtain the next ode:t2u′u + tu + 1 = 0, (3.282)

but unfortunately we have not advanced.

Another try is the following one:

(t =

1Byx

, u(t) =B2

Ax

)=⇒

(x =

B2

Au(t), y =

Au(t)

B3t

)(3.283)

and hence:tu2 + u′ (1 + ut) = 0, (3.284)

Page 85: Ecuaciones Diferenciales Mediante Simetrías · vi CONTENTS Lieperomuchísimomássencillodeaplicaryaqueunonodebeaplicarningúnalgoritmoparaobtener las simetrías que inducen dichos

3.10. ECUACIONES TIPO ABEL Y CHINI. 75

but as we supposed these attemps do not simplify our ode.

Now we change the strategy and we are going to suppose that the ode has dimension of y. In this case we needto introduce the following dimensional constants in order to make eq. (3.279) verify the p.d.h.

ay′ = −bx2y3 − cy2 − ay

x, (3.285)

where [a] = x, [b] = y−2x−2 and [c] = y−1, which bring us to the following c.v.(

t = x, u(t) =1y

)=⇒

(x = t, y =

1u(t)

)(3.286)

therefore we obtain this new odeu′ut − 2u2 − t5 − t2u = 0. (3.287)

but as in the above tactic we have not advanced. This c.v. is precisely the suggested one by the theoretic methods.

Following the Lie method we have to solve the pde:

ηx + (ηy − ξx)(−Ax2y3 − By2 − y

x

)− ξy

(−Ax2y3 − By2 − y

x

)2−

− ξ

(1x2 y − 2Axy3

)− η

(−2By − 1

x− 3Ax2y2

)= 0, (3.288)

but this ode does not admit any symmetry.

To end, we would like to show that the theoretical method does not work either. For this purpose we follow stepby step the method beginning with a generic Abel ode of first order written as follows:

y′ = f3y3 + f2y2 + f1y + f0, (3.289)

where in our case:

f3 = −x2, f2 = −1, f1 = − 1x

, f0 = 0. (3.290)

The c.v. suggested by the theoretical method is the following one(

t = x, u(t) =1y

)=⇒

(x = t, y =

1u(t)

), (3.291)

(note that this c.v. is the same than the suggested one by D.A. (3.286)). In this way our ode is now rewritten as:

u′u = h2u2 + h1u + h0, (3.292)

whereh2 = − f1, h1 = − f2, h0 = − f3, (3.293)

finding therefore that in our case we have:

u′u =u2

t+ u + t2, (3.294)

which is an Abel of second order (this ode has no solution). To try to find a solution of this ode we make thefollowing c.v.

(r = t, s(r) = u(t)E) =⇒(

t = r, u(t) =s(r)

E

), (3.295)

obtaining this new ode:ss′ = F1s + F0, (3.296)

Page 86: Ecuaciones Diferenciales Mediante Simetrías · vi CONTENTS Lieperomuchísimomássencillodeaplicaryaqueunonodebeaplicarningúnalgoritmoparaobtener las simetrías que inducen dichos

76 CHAPTER 3. ECUACIONES DE PRIMER ORDEN

whereE = exp(−

∫h2), F1 = h1E, F0 = h0E2, (3.297)

hencess′ =

s

r+ 1, (3.298)

but unfortunately we do not know how to solve this apparently simple ode.

As we have seen this ode seems very pathological since none of the followed tactics have helped us to obtainany solution.

3.11 Ecuaciones de D’Alambert-Lagrange y de Clairaut

La ecuación de D’Alambert-Lagrange Tiene la forma

y + x f(y′

)+ ϕ

(y′

)= 0, (3.299)

mientras que la de Clairaut Tiene la forma

y − y′x + ϕ(y′

)= 0. (3.300)

3.11.1 Método tradicional.

En ambos casos la solución se obtiene efectuando el siguiente cambio de variable

y′ = p.

3.11.2 Método dimensional.

En el caso de la ecuación de D’Alambert-Lagrange se observa que se transforma en una ecuación dimensional-mente homogénea, si no se discrimina espacialmente, mediante su simple derivación respecto a x. Si ademáshacemos z = dy

dx , la (3.299) y la que resulta de derivar se escribirán

y + x f (z) + ϕ (z) = 0,

z + f (z) +[x f ′ (z) + ϕ′ (z)

] dz

dx= 0, (3.301)

es decir:dx

dz+

f ′ (z)

z + f (z)x = − ϕ′ (z)

z + f (z), (3.302)

que es una ecuación lineal de primer orden en x, que sabemos integrar. Sea x = F (z, C) su solución. Bastaráeliminar z entre ella y (3.301) para encontrar la integral general φ (x, y, C) de la ecuación dada.

La ecuación de Clairaut resulta, por lo tanto un caso particular de la ecuación de D’Alambert-Lagrange, quevenimos de estudiar, en el que

f

(dy

dx

)= − dy

dx, (3.303)

Page 87: Ecuaciones Diferenciales Mediante Simetrías · vi CONTENTS Lieperomuchísimomássencillodeaplicaryaqueunonodebeaplicarningúnalgoritmoparaobtener las simetrías que inducen dichos

3.11. ECUACIONES DE D’ALAMBERT-LAGRANGE Y DE CLAIRAUT 77

siguiendo el camino del apartado anterior obtendremos para (3.300) y su derivada respecto x

y − xz + ϕ (z) = 0,[−x + ϕ′ (z)

] dz

dx= 0. (3.304)

Esta última ecuación resultará satisfecha si dzdx = 0 , o sea z = C1 , que lleva a

y = C1x + C2, (3.305)

que indica que la solución de (3.300) corresponde a una familia de rectas. Pero también se satisface si ϕ′ (z) = x.Entonces de la eliminación de z entre esta y (3.300,3.304) resultará una curva f (x, y) = 0 que puede demostrarseque es la envolvente de la familia de rectas dada por (3.305).

Veamos dos ejemplos.

3.11.3 Ejemplos

Ejemplo 3.11.1 Resolver la ode

y = x y′ +m

y′.

Solución. Veamos que en este caso la edp a resolveres:

ηx +12

(ηy − ξx) (y +√

y2 − 4 x m)

x− 1

4

ξy (y +√

y2 − 4 x m)2

x2 −

−ξ (−y +√

y2 − 4 x m

2 x2 − m

x√

y2 − 4 x m) − 1

2

η (1 + y√y2−4 x m

)

x= 0

las soluciones son:

[ξ = y, η = 2 m], [ξ = 0, η =√

y2 − 4 x m],

[ξ = 0, η =4 x m − y2 −

√y2 − 4 x m y

x],

[ξ = 0, η = 2 x m − y2

2+

√y2 − 4 x m y

2],

[ξ = 2 x, η = y], [ξ = yx, η = −2xm + y2],

[ξ = −2 x2m + y2x, η = −3yxm + y3]

la simetría [ξ = y, η = 2 m] indice el siguiente c.v. (variables canónicas)(

t = y2 − 4 x m, u(t) =y

2 m

)

por lo tanto (x =

14−t + 4 u(t)2 m2

m, y = 2u(t)m

)

ut =±1

4√

t m=⇒ u(t) =

12

√t

m+ C1

Page 88: Ecuaciones Diferenciales Mediante Simetrías · vi CONTENTS Lieperomuchísimomássencillodeaplicaryaqueunonodebeaplicarningúnalgoritmoparaobtener las simetrías que inducen dichos

78 CHAPTER 3. ECUACIONES DE PRIMER ORDEN

la solución es:y

2 m=

12

√y2 − 4 x m

m+ C1

simplificando se obtiene

y2 − 4 x m = 0, e y = xC1 +m

C1.

Observamos que existe una simetría de escala [ξ = 2 x, η = y] que induce la siguiente solución invariante:

dx

2x=

dy

y=⇒ y2 = ax

donde a es cierta constante numérica. Esta es precisamente la solución que obtendríamos utilizando el teoremaPi. Como se observa esta solución es precisamente la envolvente de la familia de rectas dada por: y = xC1 + m

C1.

El cv que induce el AD es el siguiente:

t = x, u(t) = y2x−1, =⇒√

ut =12

t (u + tu′)√ut

+2√

ut

u + tu′

cuya solución es:

ln t ± ln(

u − 2 +√

u2 − 4u)− C1 = 0,

y por lo tanto obtenderemos

ln x ± ln

(y2

x− 2 +

√y4

x2 − 4y2

x

)− C1 = 0,

tal y como queríamos hacer ver.

Ejemplo 3.11.2 Resolver la odey = xy′ − 2y′2

Solución. Veamos que

ηx + (ηy − ξx) (x

4+

√x2 − 8 y

4) − ξy (

x

4+

√x2 − 8 y

4)2 − ξ(

14

+x

4√

x2 − 8 y) +

η√x2 − 8 y

= 0

la solución es

[ξ = 1, η =x

4], [ξ = 0, η =

√x2 − 8 y],

[ξ = 0, η =x2

4+

x√

x2 − 8 y

4− 2 y],

[ξ = 0, η = −2 y x − y√

x2 − 8 y +x3

4+

x2√

x2 − 8 y

4],

[ξ = x, η = 2 y], [ξ = −4 y + x2, η = y x],

[ξ = x (−6 y + x2), η = y (−4 y + x2)]

Vemos que [ξ = 1, η = x4 ] indice el sigiente c.v.

(u(t) = x, t = y − x2

8

)

Page 89: Ecuaciones Diferenciales Mediante Simetrías · vi CONTENTS Lieperomuchísimomássencillodeaplicaryaqueunonodebeaplicarningúnalgoritmoparaobtener las simetrías que inducen dichos

3.12. CONCLUSIONS AND DISCUSSION. 79

por lo tanto

y = t +18

u(t)2, x = u(t)

ut = ± 2√−2 t

=⇒ u(t) = ±2√

2√

(−t) + C1

de esta forma llegamos a la siguiente solución en las variables originales:

y =x2

8, e y = C1x − 2C2

1

observamos que la primera de las soluciones es precisamente la que obtenemos como solución invariante in-ducida por [ξ = x, η = 2 y].

El cv que iduce el AD es el siguiente:

t = x, u(t) = y−1x2, =⇒ t2

u= t

(2t

u− t2u′

u2

)− 2

(2t

u− t2u′

u2

)2

cuya solución es:

ln t ± 12

ln(−4 + u +

√u2 − 8u

)− 1

2ln u − C1 = 0,

y por lo tanto obtenderemos

ln x ± 12

ln

(x2

y− 2 +

√x4

y2 − 8x2

y

)−−1

2ln

x2

y− C1 = 0,

tal y como queríamos hacer ver.

3.12 Conclusions and discussion.

We have seen how writing the odes in such a way that they verify the pdh i.e. introducing dimensional constants,we can obtain in a trivial way c.v. that bring us to obtain simpler ode than the original and therefore theirintegration is immediate. Furthermore, we have tried to show that these c.v. are not obtained as if by magicbut that they correspond to invariant solutions or to particular solutions and therefore they are generated by thesymmetries that admit the ode.

Nevertheless, the D.A. has strong limitations. For example D.A. is unable (at least at this time we do not knowhow to do it) to solve the following simple linear ode

y′ =

(x3 +

1x

+ 3)

y +

(3x2 − 1

x2

), (3.306)

for this reason one must not put all his confidence in this “tactic”. This is one of the greater inconvenience thatpresents the proposed method. But as we have noticed in the introduction, we think that our pedestrian methodcontinues having validity at least when one is studying ode derived from engineering problems or physicalproblems etc... where, as it is supposed, such odes must verify the pdh in such a way that for example the odey′ = x + 1, lacks of any sense (physical sense, since we cannot add a number to a physical quantity).

Nevertheless, and in spite of the limitations that we have not avoided to show, we continued believing in thekindness (goodness) of the method and that it can be applied to obtain solutions (at least particular solutions

Page 90: Ecuaciones Diferenciales Mediante Simetrías · vi CONTENTS Lieperomuchísimomássencillodeaplicaryaqueunonodebeaplicarningúnalgoritmoparaobtener las simetrías que inducen dichos

80 CHAPTER 3. ECUACIONES DE PRIMER ORDEN

and in concrete invariant solutions) to more complicated equations like the following ones:

y′′ =y′2

y− ay2, [a] = y−1x−2 =⇒ y =

1x2 , (3.307)

y′′′ =a

y3 , [a] = y4x−3 =⇒ y = x3/4, (3.308)

y′′′ = −ayy′′, [a] = y−1x−1 =⇒ y = x−1, (3.309)

y′′ =y′

x+ a

32

y2

x3 , [a] = y−1x =⇒ y = x, (3.310)

y′′′ =(y′′)2

y′ (1 + ay′), [a] = y−1x =⇒ y = x, (3.311)

but these are questions that we will approach in a forthcoming paper.

Page 91: Ecuaciones Diferenciales Mediante Simetrías · vi CONTENTS Lieperomuchísimomássencillodeaplicaryaqueunonodebeaplicarningúnalgoritmoparaobtener las simetrías que inducen dichos

Bibliography

[1] A.E. Ruak. Jour. of the Mitchell Socety. August (1935), 127-133.

[2] G. Birhoff. Electrical Egineering. December (1948), 1185-1188.

G. Birhoff. “Hydrodynamics”. Princeton (1950)

[3] F.A. Morrison, Jr. Bull Mech. Eng. Edu. 8, (1969), 289-300.

[4] M.J. Moran. Jour. Franklin Inst. 292, (1971), 423-432.

[5] J. Hainzl. Jour. Franklin Inst. 292, (1971), 463-470.

[6] T.Y. Na and A.G. Hansen. Jour. Franklin Inst. 292, (1971), 471-489.

[7] G.I. Barenblatt and Y.B. Zel‘dovich. (1972) 285-312

[8] K. Kurt. “Dimensional Analysis and Group Theory in astrophysics”. Pergamon Press Oxford (1972).

[9] R. Seshadri and T.Y. Na. “Group Invariance in Enginenering Boundary Value Problems”. Springer-Verlang. N.Y. (1985) .

[10] G.I. Bararenblatt. “Scaling, Self-Similarity and Intermediate Asymptotics”. CUP (1996)

[11] M. Castañs and C. Díaz. J. Non-Equilib. Thermodyn. 14, (1989) 315-329.

M. Castañs and C. Díaz. Il Nuovo Cimento D 13, (1991), 1061-1069.

[12] J.A. Belinchón and P. Dávila. Class. Quantum Grav. 17,(2000), 3183-98.

J.A. Belinchón . gr-qc/0404028.

[13] M. Castañs and P. Dávila. GIAD. Lanzarote 1999.

[14] E.J. Wilczynski. Trans. Am. Math Soc 23, (1900).

[15] A. Cohen. “An Introduction to the Lie Theory of one-parameter groups”. Heath, N.Y. (1911).

[16] A.J.A. Morgan. Quart. Jour. Math. Oxford (2), 3 (1952) 250-59

[17] L. P. Eisenhart. “Continuous Groups of Transformations”. Dover 2003.

[18] L.V. Ovsiannikov. “Group Analysis of differential Eqautions”. Academic. Press, N.Y. (1982).

[19] N.H. Ibragimov. “Elementary Lie Group Analysis and Ordinary Differential Equations”. Chicherter. John Wiley & Sons (1999).

N.H. Ibragimov. “Introduction to Modern Group Analysis”. Ufa. (2000).

[20] G.W.Bluman and J.D. Cole. “Similarity Methods for Differential Equations”. Springer-Verlang (1974).

G.W.Bluman and S. Kumei. “Symmetries and Differential Eqautions”. Springer-Verlang (1989).

G.W.Bluman and S.C. Anco. “Symmetry and Integration Methods for Differential Equations”. Springer-Verlang. (2002).

[21] P.E. Hydon. Symmetry Methods for Differential Equations. CUP, (2000)

81

Page 92: Ecuaciones Diferenciales Mediante Simetrías · vi CONTENTS Lieperomuchísimomássencillodeaplicaryaqueunonodebeaplicarningúnalgoritmoparaobtener las simetrías que inducen dichos

82 BIBLIOGRAPHY

[22] B.J. Cantwell. “Introduction to Symmetry Analysis". CUP. (2002).

[23] G. Baumann. Symmetry Analysis of Differential Equations with Mathematica. Springer Telos. (2000).

[24] H.Stephani. Differential Equations. Their Solution Using Symmetries. CUP (1989).

[25] G. Emanuel. Solution of Ordinary Differential Equations by Continuos Groups. Chapma&Hall/CRC (2001).

[26] J.M. Hill. Differential Equations and Group Methods forScintists and Engineers. CRC (1992).

[27] P.J. Olver. Applications of Lie Groups to Differential Equations. 2nd ed. Springer-Verlang (1993).

[28] E. Kamke. Differentialgleichungen: Losungsmethoden und Losungen, Akademische Verlagsgeselischaft Leipzig (1959).

[29] G.M. Murphy. Ordinary Differential Equations and their Solutions. Princeton: Van Nostrand, (1960).

[30] H.T. Davis Introduction to Nonlinear Differential and Integral Equation Dover N.Y. (1962).

[31] D. Zwillinger. Handbook of Differential Equations. 2nd edition. AcademicPress, (1992).

[32] A.D. Polyanin and V.F. Zaitsev. Handbook of Exact Solution for Ordinary Differential Equations. CRC Press Boca Raton FL (1995).

[33] E.S. Cheb-Terrab and A.D. Roche. : European Journal of Applied Mathematics (2003), vol. 14, pp. 217-229. math.GM/0002059.

E.S. Cheb-Terrab and A.D. Roche. Computer Physics Communications. math-ph/0001037

E.S. Cheb-Terrab and T. Kolokolnikov. math-phys/0007023.

[34] F. Schwarz. Studies in Applied Mathematics 100, (1998), 269-294.

[35] V.M. Boyko. nlin.SI/0404020

[36] M.K. Mak, H.W. Chan and T. Harko. Computers & Mathematics, 41 (2001) 1395-1401.

M.K. Mak and T. Harko. Computers & Mathematics, 43 (2002) 91-94.

T. Harko and M.K. Mak. Computers & Mathematics, 46 (2003) 849-853.

[37] M.L. Green and P. A. Griffiths. Houston Journal of Mathematics 28, (2002) 329-351.

Page 93: Ecuaciones Diferenciales Mediante Simetrías · vi CONTENTS Lieperomuchísimomássencillodeaplicaryaqueunonodebeaplicarningúnalgoritmoparaobtener las simetrías que inducen dichos

Chapter 4

Ecuaciones de segundo orden.

4.1 Introducción

Presentamos un breve resumen del método de Lie aplicado a ecuaciones de segundo orden. Explicamos comoreducir el orden, integrarlas o linealizarlas.

Reducción del orden.

Si una ecuación de segundo o mayor orden admite al menos una simetría entonces se puede reducir su orden enuno. Para una ODE

y′′ = f(

x, y, y′)

, (4.1)

admite una simetríaX = ξ (x, y) ∂x + η (x, y) ∂y, (4.2)

puede ser integrada una vez introduciendo variables canónicas o usando el método de la diferenciación invari-ante.

Variables canónicas.

La introducción de variables canónicas, elimina la dependencia explícita de (4.1) de una de las variables x o y.Por lo tanto, una ODE de segundo orden con una simetría conocida puede ser convetida en una de las siguientesformas

y′′ = f(y, y′

), o y′′ = f

(x, y′

), (4.3)

inmediatamente reducible a una ODE de primer orden mediante métodos elementales.

Diferenciación invariante.

El método basado en el siguiente teorema proporciona una generalización y una formulación covariante (i.e.independiente de cambios de variable de x o y) de métodos “ad hoc” para bajar el orden de una ecuación.

Teorema 4.1.1 La ODE (4.1) que admite un grupo uni-paramétrico, G, generado por la simetría, X, dada por la eq. (4.2)puede ser escrita en la forma de una ODE de primer orden

dv

du= F (u, v) , (4.4)

con una función arbitraria F (u, v) . Aquí, u = u (x, y) denota un invariante del grupo G y v = v (x, y, y′) su diferencialinvariante de primer orden.

83

Page 94: Ecuaciones Diferenciales Mediante Simetrías · vi CONTENTS Lieperomuchísimomássencillodeaplicaryaqueunonodebeaplicarningúnalgoritmoparaobtener las simetrías que inducen dichos

84 CHAPTER 4. ECUACIONES DE SEGUNDO ORDEN.

Integración por medio de dos simetrías.

De acuardo con la clasificación de grupos de Lie de ODEs, tenemos que el álgebra de Lie maximal Lr admitidapor una ODE (4.1) sólo puede tener dimensiones r = 1, 2, 3 u 8. El caso r = 8 indica que (4.1) es lineal olinealizable. Si (4.1) admite Lr con r ≥ 2, podemos escoger una subálgebra 2−dimensional L2 ⊂ Lr. Por lo tanto“asumiremos” que (4.1) admite una L2.

Teorema 4.1.2 Una ODE (4.1) admite una álgebra de Lie L2, con una base X1, X2, tal que el álgebra L1 generado por X1es un ideal en L2. Entonces la ecuación resultante de reducir la ODE (4.1) mediante X1 admite el álgebra cociente L2/L1.

Es decir, dada la base X1, X2, entonces calculamos [X1, X2] = cX1. Entonces X1 genera el ideal L1 mientras que elálgebra cociente L2/L1 puede seridentificada con el álgebra que genera X2. La ODE de primer orden resultantede reducir el orden mediante X1 toma la forma dada por (4.4) entonces esta ODE admite X2 pero escrito entérmino de las variables u, v.

Forma canónica de L2.

La forma canónica de L2 nos suministra el método más simple de integración de una ODE pues las reduce a sólo4 tipos.

Teorema 4.1.3 Toda L2 puede reducirse, eligiendo una base apropiada,

X1 = ξ1∂x + η1∂y, X2 = ξ2∂x + η2∂y,

a uno de los siguientes tipos:I [X1, X2] = 0 ξ1η2 − η1ξ2 6= 0,II [X1, X2] = 0 ξ1η2 − η1ξ2 = 0,III [X1, X2] = X1 ξ1η2 − η1ξ2 6= 0,IV [X1, X2] = X1 ξ1η2 − η1ξ2 = 0.

Este teorema nos permite obtener la siguiente tabla:

Estructura de L2 Forma canónica de L2 ODE

I [X1, X2] = 0, ξ1η2 − η1ξ2 6= 0, X1 = ∂x, X2 = ∂y, y′′ = f (y′) ,II [X1, X2] = 0, ξ1η2 − η1ξ2 = 0, X1 = ∂y, X2 = x∂y, y′′ = f (x) ,III [X1, X2] = X1, ξ1η2 − η1ξ2 6= 0, X1 = ∂y, X2 = x∂x + y∂y, y′′ = 1

x f (y′) ,IV [X1, X2] = X1, ξ1η2 − η1ξ2 = 0, X1 = ∂y, X2 = y∂y, y′′ = f (x) y′.

La receta

Los paso a seguir son por lo tanto los siguientes:

1. Calcular el álgebra de Lie que admite Lr.

2. Si r ≥ 2, entonces calcular L2 ⊂ Lr. Si r < 2 entonces no se puede aplicar el método.

3. Determinar el tipo de L2 (mirar la tabla anterior).

4. Encontrar el cambio de variable (c.v.) que tranforma L2 en su forma canónica.

5. integrar la ecuación resultante en las nuevas variables.

Page 95: Ecuaciones Diferenciales Mediante Simetrías · vi CONTENTS Lieperomuchísimomássencillodeaplicaryaqueunonodebeaplicarningúnalgoritmoparaobtener las simetrías que inducen dichos

4.2. DISTINTOS TIPOS DE ECUACIONES. 85

4.2 Distintos tipos de ecuaciones.

4.2.1 Lineales con coeficiente constantes.

Sea dada la ecuación diferenciala0yn + a1yn−1 + ......... + any = 0 (4.5)

donde las ai son constantes reales.

Consideramos la ecuación característica

a0λn + a1λn−1 + ......... + an = 0 (4.6)

supongamos que las (λi) son las raíces de la ecuación, entre las cuales puede haber múltiples. Se pueden pre-sentar los siguientes casos.

1. Las raíces son reales y distintas i.e.

λ1, ....., λn son reales y distintas

En este caso el sistema fundamental de soluciones tiene la forma

eλ1x, .........., eλnx (4.7)

y la solución general a la ecuación homogénea es :

yg = C1eλ1x + .......... + Cneλnx (4.8)

2. Las raices son reales pero algunas de ellas son múltiples. Sea por ejemplo

λ1 = λ2 = ... = λk = λ, (4.9)

de modo que λ es una raiz k−múltiple , mientras que todas las demás (n − k) raices son distintas. En estecaso el sistema fundamental de soluciones tiene la forma

eλx, xeλx, x2eλx, ........, xk−1eλx, eλk+1x, ........, eλnx, (4.10)

y la solución general es :

yg = C1eλx + C2xeλx + C3x2eλx + ... + Ckxk−1eλx + Ck+1eλk+1x + ... + Cneλnx. (4.11)

3. Supongamos que algunas de las raices son imaginarias. Para simplificar imaginemos que la situación es lasiguiente :

λ1 = α + iβ, λ2 = α − iβ, λ3 = γ + iδ, λ4 = γ − iδ, (4.12)

y las demás raices son todas reales. En este caso el sistema fundamental de ecuaciones es :

eαx cos β, eαx sen β, eγx cos δ, eγx sen δ, eλ5x, ......., eλnx (4.13)

y la solución general es :

yg = C1eαx cos β + C2eαx sen β + C3eγx cos δ + C4eγx sen δ + C5eλ5x + .. + Cneλnx (4.14)

4. Siλ1 = α + iβ

es una raiz k-múltiple

k <

(n

2

)

Page 96: Ecuaciones Diferenciales Mediante Simetrías · vi CONTENTS Lieperomuchísimomássencillodeaplicaryaqueunonodebeaplicarningúnalgoritmoparaobtener las simetrías que inducen dichos

86 CHAPTER 4. ECUACIONES DE SEGUNDO ORDEN.

entonces,λ2 = α − iβ

será tambien una raiz k-múltiple y el sistema fundamental de soluciones será de la forma :

eαx cos β, eαx sen β, xeαx cos β, xeαx sen β, ......, xk−1eαx cos β, xk−1eαx sen β,

eλ2k+1x, ......., eλnx (4.15)

por consiguiente la solución general es de la forma

yg = C1eαx cos β + C2eαx sen β + C3xeαx cos β + C4xeαx sen β + .......+

C2k−1xk−1eαx cos β + C2kxk−1eαx sen β + C2k+1eλ2k+1x + ............. + Cneλnx (4.16)

Ejemplos.

Vamos a estudiar unos ejemplos para el caso en el que la ecuación no es homogénea i.e. tiene la forma

a0yn + a1yn−1 + ......... + any = f (x), (4.17)

donde la f (x) es cierta función de x.

Atendiendo a la anterior clasificación i.e. adependiendo de los autovalores y sus respectivas multiplicidades nospodemos encontrar las siguiente tabla

c f (x) raices de la ecuación yp(x)

1 Pm(x)1. 0 /∈ σ(λ)2. 0 ∈ σ(λ), ν(0) = s.

Pm(x)xs Pm(x)

2 Pm(x)eαx 1. α /∈ σ(λ)2. α ∈ σ(λ), ν(α) = s.

Pm(x)eαx

xs Pm(x)eαx

3 Pm(x) cos βx + Qm(x) sin βx1. iβ /∈ σ(λ)2. iβ ∈ σ(λ), ν(iβ) = s.

Pm(x) cos βx + Qm(x) sin βxxs

(Pm(x) cos βx + Qm(x) sin βx

)

4 eαx [Pm(x) cos βx + Qm(x) sin βx]1. α + iβ /∈ σ(λ)2. α + iβ ∈ σ(λ), /

ν(α + iβ) = s.

eαx(

Pm(x) cos βx + Qm(x) sin βx)

eαxxs(

Pm(x) cos βx + Qm(x) sin βx)

Con esta tabla en mente pasaremos a estudiar algunos ejemplillos.

Ejemplo 4.2.1 Resolver la ecuacióny′′′ − y′′ + y′ − y = x2 + x (4.18)

Solution. Las raices de la ecuación homogénea son (1,−i, i) , por lo que la solución de la homogénera será de laforma

yh = C1ex + C2 sin x + C3 cos x. (4.19)

Al ser f (x) = Pm(x) = x2 + x i.e. un polinomio y 0 /∈ σ(λ), entonces la forma de la solución particular será

yp = A1x2 + A2x + A3,

esta ecuación debe debe verificar la ecuación original así que

y′′′p − y′′p + y′p − yp = x2 + x

Page 97: Ecuaciones Diferenciales Mediante Simetrías · vi CONTENTS Lieperomuchísimomássencillodeaplicaryaqueunonodebeaplicarningúnalgoritmoparaobtener las simetrías que inducen dichos

4.2. DISTINTOS TIPOS DE ECUACIONES. 87

− (2A1) + (2A1x + A2) −(

A1x2 + A2x + A3

)= x2 + x,

por lo tantoA1 = −1, A2 = −3, A3 = −1.

De esta forma la solución general es:

y = C1ex + C2 sin x + C3 cos x − x2 − 3x − 1. (4.20)

como queríamos demostrar.

Ejemplo 4.2.2 Resolver la ecuacióny′′′ − y′′ = 12x2 + 6x. (4.21)

Solution. Las raices de la ecuación homogénea son (1, 0, 0) , por lo que la solución de la homogénera será de laforma

yh = C3ex + C2x + C1. (4.22)

Al ser f (x) = 12x2 + 6x i.e. un polinomio y 0 ∈ σ(λ), ν(0) = s = 2, entonces la forma de la solución particularserá

yp = xsPm(x) = x2(

A1x2 + A2x + A3

)=

(A1x4 + A2x3 + A3x2

),

esta ecuación debe debe verificar la ecuación original así que

y′′′p − y′′p + y′p − yp = 12x2 + 6x

por lo tantoA1 = −1, A2 = −5, A3 = −15.

De esta forma la solución general es:

y = C1 + C2x + C3ex − x4 − 5x3 − 15x2, (4.23)

como queríamos demostrar.

Ejemplo 4.2.3 Resolver la ecuacióny′′ + y′ = 2x2ex. (4.24)

Solution. Las raices de la ecuación homogénea son (−1, 0) , por lo que la solución de la homogénera será de laforma

yh = C1 + C2e−x. (4.25)

Al ser f (x) = 2x2ex i.e. un polinomio y α = 1 /∈ σ(λ), entonces la forma de la solución particular será

yp = eαx Pm(x) = ex(

A1x2 + A2x + A3

),

por lo tantoA1 = 2, A2 = −6, A3 = 7.

De esta forma la solución general es:

y = C1 + C2e−x +(

2x2 − 6x + 7)

ex, (4.26)

como queríamos demostrar.

Page 98: Ecuaciones Diferenciales Mediante Simetrías · vi CONTENTS Lieperomuchísimomássencillodeaplicaryaqueunonodebeaplicarningúnalgoritmoparaobtener las simetrías que inducen dichos

88 CHAPTER 4. ECUACIONES DE SEGUNDO ORDEN.

Ejemplo 4.2.4 Resolver la ecuacióny′′ + 10y′ + 25y = 4e−5x. (4.27)

Solution. Las raices de la ecuación homogénea son (−5,−5) , por lo que la solución de la homogénera será dela forma

yh = (C1 + C2x) e−5x. (4.28)

Al ser f (x) = 4e−5x i.e. un polinomio y α = −5 ∈ σ(λ), ν(α) = s, entonces la forma de la solución particularserá

yp = eαx Pm(x)xs = e−5x (A1) x2,

por lo tantoA1 = 2, .

De esta forma la solución general es:

y = (C1 + C2x) e−5x + 2x2e−5x, (4.29)

como queríamos demostrar.

Ejemplo 4.2.5 Resolver la ecuacióny′′ + 3y′ + 2y = x sin x. (4.30)

Solution. Las raices de la ecuación homogénea son (−1,−2) , por lo que la solución de la homogénera será dela forma

yh = C1e−x + C2e−2x. (4.31)

Al ser f (x) = x sin x i.e. un polinomio y iβ /∈ σ(λ), entonces la forma de la solución particular será

yp = Pm(x) = e−5x (A1) x2,

por lo tantoA1 = 2, .

De esta forma la solución general es:

y = (C1 + C2x) e−5x + 2x2e−5x, (4.32)

como queríamos demostrar.

Ejemplo 4.2.6 Resolver la ecuacióny′′ + 3y′ + 2y = x sin x. (4.33)

Solution. Las raices de la ecuación homogénea son (−1,−2) , por lo que la solución de la homogénera será dela forma

yh = C1e−x + C2e−2x. (4.34)

Al ser f (x) = 4e−5x i.e. un polinomio y α = −5 ∈ σ(λ), ν(α) = s, entonces la forma de la solución particularserá

yp = eαx Pm(x)xs = e−5x (A1) x2,

Page 99: Ecuaciones Diferenciales Mediante Simetrías · vi CONTENTS Lieperomuchísimomássencillodeaplicaryaqueunonodebeaplicarningúnalgoritmoparaobtener las simetrías que inducen dichos

4.2. DISTINTOS TIPOS DE ECUACIONES. 89

por lo tanto

A1 = 2, .

De esta forma la solución general es:

y = (C1 + C2x) e−5x + 2x2e−5x, (4.35)

como queríamos demostrar.

4.2.2 Ecuación de Bessel.

La ecuación de Bessel tiene la forma

x2y′′ + xy′ +(

x2 − n2)

y = 0 (4.36)

y cuya solución es:

y = C1Bessel J(n, x) + C2BesselY(n, x) (4.37)

La ecuación de Bessel modificada es de la forma:

x2y′′ + xy′ −(

x2 + n2)

y = 0 (4.38)

y cuya solución es:

y = C1Bessel I(n, x) + C2BesselK(n, x) (4.39)

Estas ecuaciones no admiten simetrías de tal forma que el método de Lie es inoperante.

4.2.3 Ecuación de Duffing.

La ecuación de Duffing tiene la forma

y′′ + y′ + εy3 = 0 (4.40)

y cuya solución es:∫ y ±2da√

−4a2 − 2εa4 + 4C1− x − C2 = 0. (4.41)

Esta ecuación admite una simetría, X1 = ∂x, que nos lleva a obtener el siguiente c.v.

t = y, u = 1/y′, (4.42)

de donde obtenemos,

u′ = t(

1 + εt2)

u3, (4.43)

i.e. una ODE separable y cuya solución es:

u = ± 2√−4t2 − 2εt4 + 4C1

. (4.44)

Page 100: Ecuaciones Diferenciales Mediante Simetrías · vi CONTENTS Lieperomuchísimomássencillodeaplicaryaqueunonodebeaplicarningúnalgoritmoparaobtener las simetrías que inducen dichos

90 CHAPTER 4. ECUACIONES DE SEGUNDO ORDEN.

4.2.4 Ecuación de Emden y Emden-Fowler.

Vamos a estudiar tres tipos de ecuaciones tipo Emden. Para más detalles y más ecuaciones recomendamos ellibro de A. Polyanin y V. Zaitsev, Exact Solutions for ODEs, CRC 1995, páginas 241-301.

1. La ecuación de Emden tiene la forma

x2y′′ + 2xy′ + x2yn = 0, (4.45)

y cuya solución es:

y = te2

1−n

∫udt+C1 , x = e

∫udt+C1 , (4.46)

donde u(t) es solución de la ODE (tipo Abel)

u′ =u2

(n − 1)2

[−n2 (1 + utn) + 2n (ut + 3 + utn) − ut

(6 + tn−1

)− 5

]. (4.47)

que no admite simetrías y que no conocemos su solución. Vemos además que la ecuación (4.45) admiteuna única simetría, X1 = (1 − n)x∂x + 2y∂y, y por lo tanto debe admitir la solución particular

y = ax2/(1−n), (4.48)

observándose que la eq. (4.45) es DH sii introducimos una constante a, tal que

x2y′′ + 2xy′ + ax2yn = 0, [a] = y1−nx−2. (4.49)

2. La ecuación de Emden modificada tiene la forma

y′′ + f1(x)y′ + yn = 0, (4.50)

y cuya solución es:y =?, (4.51)

dependiendo de la forma de f (x) la ecuación podrá admitir una u otra simetría. También podemos calcularel sistema de EDPs para determinar sus simetrías e imponer alguna determinada, como por ejemplo lade escala, de esta forma hayamos las posibles formas que puede tomar f (x) para que la ecuación seaintegrable. Nótese que si imponemos la simetría de escala entonces f (x) = x−1, y por lo tanto estaríamosen la situación anteriror.

3. La ecuación de Emden-Fowler tiene la forma

xpy′′ + xp−1 py′ + xσyn = 0, (4.52)

i.e.y′′ = − p

xy′ − axσ−pyn, [a] = y1−nxp−σ−2.

de esta forma vemos que una solucion particular debe ser:

y = ax2+σ−p

1−n . (4.53)

Al igual que en la ecuación de Emden, en este caso, la ecuación de Emden-Fowler también admite unasimetría de escala, tal y como hemos visto, X = (1 − n)x∂x + (2 + σ − p) y∂y. El c.v. obtenido por lasvariables canónicas nos lleva a obtener una ODE tipo Abel sin solución.

Observación 4.2.1 En las ecuaciones que sólo admitan una simetría y ésta sea de escala, como en las ecuaciones tipoEmden y EF, el cv que induce el AD es el mismo que el de grupos de Lie. Si la ODE admite más simetrías y el camporesultante (el que es un ideal del álgebra) no es de escala, entonces el cv que iduce el AD no funciona, es más complicamucho la situación. Veremos algunos ejemplos más adelante.

Page 101: Ecuaciones Diferenciales Mediante Simetrías · vi CONTENTS Lieperomuchísimomássencillodeaplicaryaqueunonodebeaplicarningúnalgoritmoparaobtener las simetrías que inducen dichos

4.2. DISTINTOS TIPOS DE ECUACIONES. 91

4.2.5 Ecuación lineal exacta.

La ecuación lineal exacta tiene la formay′′ = f (x)y′ + f ′(x)y, (4.54)

yy′′ = f (x)y′ + f ′(x)y + g′(x), (4.55)

donde f ′ = d f /dx, y cuya solución de (4.55) es:

y =

(C2 +

∫ (C1 + ge

∫− f dx

)dx

)e−

∫− f dx. (4.56)

4.2.6 Ecuación no lineal exacta.

Un ejemplo de ecuación no lineal exacta puede ser el siguiente

y′′ = − x

y2 y′ +1y

, (4.57)

cuya solución de es:

12

ln(− x2 − C1 y x − y2

x2 ) +

C1 arctanh(

C1 x+2 y

x√

4+C21

)

√4 + C2

1

+ ln(x) + C2 = 0. (4.58)

Observamos que podemos rescribir la ecuación de la siguiente forma

y′′ = − ax

y2 y′ +a

y, [a] = y2x−2, (4.59)

por lo que una solución particular a dicha ecuación será:

y = x. (4.60)

Veamos cuales son las simetrias de la ODE

ξy y y3 = 0,

2 y ξy x + ηy, y y3 − 2 ξx y y3 = 0,

−2 η x − 3 ξy y2 + y ξx x + ξ y + 2 ηxy y3 − ξxx y3 = 0,

η y − 2 ξx y2 + ηy y2 + x y ηx + ηx x y3 = 0,

por lo tantoξ = x, η = y

el cambio de variables que induce esta simetría es el siguiente

(u(t) = ln(x), t =

y

x

)=⇒

(x = eu(t), y = t eu(t)

)

por lo tanto la ODE se rescribe como

ut, t =ut

2 (t2 + 1)

t2

Page 102: Ecuaciones Diferenciales Mediante Simetrías · vi CONTENTS Lieperomuchísimomássencillodeaplicaryaqueunonodebeaplicarningúnalgoritmoparaobtener las simetrías que inducen dichos

92 CHAPTER 4. ECUACIONES DE SEGUNDO ORDEN.

y cuya solución es

u(t) = −12

ln(t2 − 1 − C1 t) +C1 arctanh( 2 t−C1√

4+C21

)√

4 + C21

,

y por lo tanto en las variables originales obtenemos la solución antes expuesta.

La reducción de orden viene dada por el siguiente c.v.

t =y

x, u(t) =

x

−y + xy′,

por lo tanto la ODE original queda

ut =u(t)2 (t2 + 1)

t2

viendo que se trata de una ODE separable, cuya solución es:

u(t) = − t

t2 − 1 − C1 t,

haciendo el c.v. en las variables originales obtenemos

y′ =x

y+ C1

cuya solución es (4.58).

En este ejemplo hemos visto que la ODE bajo estudio sólo admite una simetría y que ésta, además es de escala.Por lo tanto el cv que induce el AD es el mismo que el de GL ie.

t =y

x, u(t) =

x

−y + xy′.

También podríamos emplear una táctica del estilo(t = x, u(t) = y

x

)y ver que pasa.

4.2.7 La ecuación lineal.

La ecuación lineal tiene la formay′′ = p(x)y′ + q(x)y. (4.61)

La condición de linealización nos lleva a:

ξ = Ay + B,

η =(

A′ + pA)

y2 + Cy + D,

donde

A′′ + (pA)′ − qA = 0,

B′′ + (pB)′ = 2C′,

C′′ − pC′ = 2qB′ + q′B,

D′′ − pD′ − qD = 0,

Page 103: Ecuaciones Diferenciales Mediante Simetrías · vi CONTENTS Lieperomuchísimomássencillodeaplicaryaqueunonodebeaplicarningúnalgoritmoparaobtener las simetrías que inducen dichos

4.2. DISTINTOS TIPOS DE ECUACIONES. 93

viendo que la ecuación para D es la propia ODE, mientras que A satisface la adjunta. Supongamos ahora que lasolución general es de la forma

y = K1y1 + K2y2, Ki ∈ R,

entonces

A = e−∫

pdx (c4y1 + c5y2) ,

B = e−∫

pdx(

c6y21 + 2c7y1y2 + c8y2

2

),

C = c1 + e−∫

pdx(c6y1y′1 + c7

(y′1y2 + y1y′2

)+ c8y2y′2

),

D = c2y1 + c3y2, ,

por lo que nuestra ODE admite las siguientes simetrías:

X1 = y∂y, X2 = y1∂y, X3 = y2∂y, X4 = e−∫

pdx(

yy1∂x + y′1y2∂y

),

X5 = e−∫

pdx(

yy2∂x + y′2y2∂y

), X6 = e−

∫pdx

(y2

1∂x + y1y′1y∂y

),

X7 = e−∫

pdx(2y1y2∂x +

(y2y′1 + y′2y1

)∂y

), X8 = e−

∫pdx

(y2

2∂x + y2y′2y∂y

).

4.2.8 Ecuación de Largerstrom.

La ecuación de Largerstrom tiene la forma

y′′ = −K

xy′ − εyy′, (4.62)

i.e.

y′′ = −(

K

x+ εy

)y′, [K] = x0y0, [ε] = y−1x−1,

y cuya solución es:y = te(

∫u(t)dt+C1), u′ = (2 − K − εt) tu3 + (3 − K − εt) u2, (4.63)

donde se observa que esta ODE es de tipo Abel y no tiene solución.

La ecuación (4.62) admite simetría de escala

X = x∂x − y∂y,

que induce el c.v.

u = − 1(y′x + y) x

, t = xy,

con el que se obtiene la ODE (4.63).

De igual forma vemos que la ODE (4.62) admite solución particular

y =c

x, c ∈ R.

4.2.9 Ecaución de Lienard.

La ecuación de Lienard tiene la formay′′ + f (x)y′ + y = 0 (4.64)

cuya solución es:y = e(

∫u(t)dt+C1), u′ = −u2 − f u − 1, (4.65)

Page 104: Ecuaciones Diferenciales Mediante Simetrías · vi CONTENTS Lieperomuchísimomássencillodeaplicaryaqueunonodebeaplicarningúnalgoritmoparaobtener las simetrías que inducen dichos

94 CHAPTER 4. ECUACIONES DE SEGUNDO ORDEN.

es una ODE tipo Riccati. La reducción de orden viene dada por el siguiente c.v.

u =y′

y, x = t. (4.66)

Veamos un ejemplo concreto.

Ejemplo 4.2.7 Resolver la ecuación

y′′ +1x

y′ + Ky = 0

vemos que su solución es:y = C1Bessel J (0, x) + C2Bessel(0, x).

4.2.10 La ecuación de Liouville.

La ecuación de Lioville tiene la formay′′ + g(y)y′2 + f (x)y′ = 0, (4.67)

esta ecuación admite las siguientes simetrías

X1 = e−∫

gdy∂y, X2 = e∫

f dx∂x, (4.68)

y cuya solución es: ∫ ye∫

g(b)dbdb − C1

∫e−

∫f (x)dxdx − C2 = 0. (4.69)

La reducción de oreden viene dada por el c.v.

u′ = f (t)u, t = x, u = e∫ y

g(b)db. (4.70)

4.2.11 La ecuación de Painleve.

Las ecuaciones de Painleve se dividen en 6 grupos. Exponemos a continuación ejemplos de cada una de lasclases.

1. Painleve Iy′′ = 6y2 + x, (4.71)

2. Painleve IIy′′ = 2y3 + xy + a, (4.72)

3. Painleve III

y′′ =y′′2

y− y′

x+

ay2 + b

x+ cy3 +

d

y, (4.73)

4. Painleve IV

y′′ =12

y′′2

y+

32

y3 + 4xy2 + 2(

x2 − a)

y +b

y, (4.74)

Page 105: Ecuaciones Diferenciales Mediante Simetrías · vi CONTENTS Lieperomuchísimomássencillodeaplicaryaqueunonodebeaplicarningúnalgoritmoparaobtener las simetrías que inducen dichos

4.2. DISTINTOS TIPOS DE ECUACIONES. 95

5. Painleve V

y′′ =

(1

2y+

1y − 1

)y′′2 − y′

x+

(y − 1)2(

ay + by

)

x2 +gy

x+

dy (y + 1)

y − 1, (4.75)

6. Painleve VI

y′′ =

(1y

+1

y − 1+

1y − x

)y′′2 −

(1x

+1

x − 1+

1y − x

)y′+

+

y (y − 1) (y − x)

(a + bx

y2 + g(x−1)

(y−1)2 + dx(x−1)

(y−x)2

)

x2 (x − 1)2 . (4.76)

Todas estas ecuaciones son complicadas de resolver y ninguna de ellas admite simetrías por lo que el método deLie no funciona.

4.2.12 La ecuación de Titchmarch.

La ecuación de Titchmarch tiene la forma

y′′ +(

λ − x2n)

y = 0 (4.77)

y cuya solución es:

y = e∫

u(t)dt+C1 . (4.78)

Esta ODE admite una simetría, X = y∂y, que induce el siguiente c.v.

t = x, u(t) =y′

y, (4.79)

por lo tanto obtenemosu′ = −u2 − λ + t2n, (4.80)

que es tipo Riccati, pero sin solución.

4.2.13 La ecuación de Van der Pol.

La ecuación de Van der Pol tiene la forma

y′′ − µ(

1 − y2)

y′ + y = 0 (4.81)

que admite una simetríaX = ∂x, (4.82)

que nos lleva a obtener el siguiente c.v.(

t = y, u(t) =1y′

), =⇒

(y = t, x =

∫udt + C1

), (4.83)

por lo que la ODE original se reduce a:

u′ = tu3 + µ(

t2 − 1)

u2, (4.84)

tipo Abel sin solución.

Page 106: Ecuaciones Diferenciales Mediante Simetrías · vi CONTENTS Lieperomuchísimomássencillodeaplicaryaqueunonodebeaplicarningúnalgoritmoparaobtener las simetrías que inducen dichos

96 CHAPTER 4. ECUACIONES DE SEGUNDO ORDEN.

4.3 Ejemplos

Ejemplo 4.3.1 Resolver la siguiente ecuación de segundo orden

y′′ =y′2

y+ y2 (4.85)

Solución. Aplicando el método de Lie lo primero que tenemos que planear el es sistema de edps que nosdeterminarán las simetrías, este sistema de edps es:

yξyy + ξy = 0,

η − yηy + y2ηyy − 2y2ξxy = 0,

−3y3ξy − 2ηx + 2yηxx − yξxx = 0,

−2yη − 2y2ξx + y2ηy − y2ηxx = 0,

cuyas soluciones son:X1 = ∂x, X2 = x∂x − 2y∂y, (4.86)

comprobando que[X1, X2] = X1.

De esta forma las variables canónicas que induce este campo X1 son:

(t = y, u(t) = x) =⇒ (x = u(t), y = t) (4.87)

en estas nuevas variables la ode original (4.85) se rescribe como:

u′′ = −u′ (1 + t3u′2)

t(4.88)

que no parece muy fácil de resolver, por esta razón volveremos a utilizar las variables canónicas para reducir elorden de la ode original (4.85), resultando

u′ = −u

t− u3t2 (4.89)

donde (t = y, u(t) =

1y′

), (4.90)

encontrando que la ecuación (4.89) es de tipo Bernoulli y cuya solución inmediata es:

u = ± 1t√

2t + C1, (4.91)

pero imaginemos que no sabemos resolverla. En dicho caso aplicamos otra vez el método de Lie obteniendo porlo tanto (simplemente aplicamos el procedimiento standard):

ηt + (ηu − ξt)(−u

t− u3t2

)− ξu

(−u

t− u3t2

)2− ξ

( u

t2 − 2u3t)− η

(−1

t− 3u2t2

)= 0

encontrando como solución los siguientes campos:

X1 = u3t2∂u, X2 =(

u − 2u3t3)

∂u, X3 = t∂t −32

u∂u, X4 = ∂t −u

t∂u

Page 107: Ecuaciones Diferenciales Mediante Simetrías · vi CONTENTS Lieperomuchísimomássencillodeaplicaryaqueunonodebeaplicarningúnalgoritmoparaobtener las simetrías que inducen dichos

4.3. EJEMPLOS 97

si por ejemplo elegimos X4 para calcular las v.c. encontramos:

(r = ut, z(r) = t) =⇒(

t = z(r), u =r

z(r)

)(4.92)

de tal forma que la ecuación (4.89) se rescribe en estas nuevas variables como:

z′ = − 1r3 =⇒ z(r) =

12r2 + C1

deshaciendo el c.v. (y simplificando)

u(t) = ± 1t√

2t + C1,

que es la solución (4.91) que ya habíamos obtenido.

Otra forma de atacar la ecuación (4.89) sería la de buscar un factor integrante µ que haga que (µ ∗ (4.89)) seaexacta, dicho factor integrante es:

µ =1

u3t2 .

Una vez obtenida la solución a la ecuación (4.89) entonces sólo nos queda deshacer el c.v. (4.90) obteniendo lasiguiente ecuación diferencial:

y′ = y√

2y + C1 (4.93)

que es difícil de integrar (si lo fuese entonces volveríamos a aplicar el método de Lie) encontrando que

y =1 + tan

(x+C22C1

)2

2C21

(4.94)

es la solución más general a la ecuación planteada (4.85).

Veremos ahora como obtener soluciones particulares. De las dos simetrías obtenidas al sistema de edps i.e. (4.86)tomamos X2 = x∂x − 2y∂y, i.e. una simetría de escala y calculamos la solución invariante que induce i.e.

dx

x= −dy

2y=⇒ y =

2x2

es una solución particular a la ecuación (4.85). A igual resultado podemos llegar si aplicamos nuestro métodopedestre (el AD) simplemente observando que dicha ecuación será dimensionalmente homogénea si introduci-mos una constante dimensional i.e. la ecuación (4.85) se rescribe como

y′′ =y′2

y+ Ky2 (4.95)

tal que [K] = y−1x−2 por lo tanto la solución que nos sugiere el AD es del tipo

y K xy 1 −1 0x 0 −2 1

=⇒ y ∼1

Kx2

tal y como ya sabíamos por la solución invariante.

Observación 4.3.1 Si insistemos en utilizar la simetría de escala para obetener un cv, veremos que la cosa se complicamuchísimo. Vemos que X2 = x∂x − 2y∂y induce el siguiente cv

t = yx2, u(t) =1

2 (2y + y′x),

Page 108: Ecuaciones Diferenciales Mediante Simetrías · vi CONTENTS Lieperomuchísimomássencillodeaplicaryaqueunonodebeaplicarningúnalgoritmoparaobtener las simetrías que inducen dichos

98 CHAPTER 4. ECUACIONES DE SEGUNDO ORDEN.

que nos lleva a obtener una ODE tipo Abel

u′ = −(

t2 − 2t)

u3 − u2 − u

t,

la cual sí tiene solución, el problema está en deshacer el cv en dicha solución. Por lo tanto el cv que induciría el AD (ie. elde la simetría de escala X2) NO FUNCIONA.

Ejemplo 4.3.2 Resolver la siguiente ecuación de segundo orden

y′′ =y′

y2 − 1xy

(4.96)

Solución. Aplicando el método de Lie lo primero que tenemos que planear el es sistema de edps que nosdeterminarán las simetrías, este sistema de edps es:

y3ξyy = 0,

−2ξy + y2ηyy − 2y2ξxy = 0,

3y2

xξy − 2η − yξx + 2y3ηxy − y3ξxx = 0,

− y

xη + 2

y2

xξx −

y2

xηy −

y2

xξ − yηx + y3ηxx = 0,

cuyas soluciones son:

X1 = x2∂x + xy∂y, X2 = x∂x +y

2∂y, (4.97)

comprobando que[X1, X2] = −X1.

De esta forma las variables canónicas que induce este campo X1 son:(

t =y

x, u(t) = − 1

x

)=⇒

(x = − 1

u(t), y = − t

u(t)

)(4.98)

en estas nuevas variables la ode original (4.96) se rescribe como:

u′′ = −u′2

t2 (4.99)

que parece fácil de resolver, por ejemplo podemos encontrar un factor integrante µ que haga que dicha ode seaexacta, este factor integrante es:

µ =1

u′2

de tal forma que la primera integral de la ode resultante es:

− 1u′ −

1t

+ C1 = 0

distinguiendo dos casos. Si C1 = 0 entonces

− 1u′ −

1t

= 0 =⇒ u(t) = − t2

2+ K

Page 109: Ecuaciones Diferenciales Mediante Simetrías · vi CONTENTS Lieperomuchísimomássencillodeaplicaryaqueunonodebeaplicarningúnalgoritmoparaobtener las simetrías que inducen dichos

4.3. EJEMPLOS 99

que deshaciendo el c.v. nos lleva a obtener:

y =√

2Kx2 + 2x (4.100)

mintras que si C1 6= 0 entonces

u(t) =t

C1+

ln (C1t − 1)

C1+ C2

deshaciendo el c.v. obtenemos por lo tanto:

− 1x

=y

xC1+

ln(C1

yx − 1

)

C1+ C2. (4.101)

En este milagroso ejemplo todo ha resultado extremadamente sencillo pero para demostrar como funciona elmétodo haremos como en el ejemplo anterior y volveremos a utilizar las variables canónicas para reducir elorden de la ode original (4.99), resultando

u′ = −u2

t2 (4.102)

donde (t =

y

x, u(t) =

1y′x − y

), (4.103)

encontrando que la ecuación (4.102) es de tipo separable y cuya solución inmediata es:

u =t

C1t − 1, (4.104)

pero imaginemos que no sabemos resolverla. En dicho caso aplicamos otra vez el método de Lie obteniendo porlo tanto (simplemente aplicamos el procedimiento standard):

ηt + (ηu − ξt)

(−u2

t2

)− ξu

(−u2

t2

)2

− 2ξ

(u2

t3

)+ 2η

( u

t2

)= 0

encontrando como solución los siguientes campos:

X1 = u2∂u, X2 = t2∂t, X3 =u(t + u)

t∂u, X4 =

(1 +

u(2t + u)

t2

)∂u

X5 = t∂t + u∂u, etc...

si por ejemplo elegimos X4 para calcular las v.c. encontramos:

(r = t, z(r) = − t2

t + u

)=⇒

(t = r, u = − r (z(r) + r)

z(r)

)(4.105)

de tal forma que la ecuación (4.102) se rescribe en estas nuevas variables como:

z′ = −1 =⇒ z(r) = −r + C1

deshaciendo el c.v. (y simplificando)

u(t) =t

C1t − 1,

que es la solución (4.104) que ya habíamos obtenido.

Otra forma de atacar la ecuación (4.102) sería la de buscar un factor integrante µ que haga que (µ ∗ (4.102)) seaexacta, dicho factor integrante es:

µ =1u2 .

Page 110: Ecuaciones Diferenciales Mediante Simetrías · vi CONTENTS Lieperomuchísimomássencillodeaplicaryaqueunonodebeaplicarningúnalgoritmoparaobtener las simetrías que inducen dichos

100 CHAPTER 4. ECUACIONES DE SEGUNDO ORDEN.

Una vez obtenida la solución a la ecuación (4.102) entonces sólo nos queda deshacer el c.v. (4.103) obteniendo lasiguiente ecuación diferencial:

y′ =y

x+

C1

x− 1

y(4.106)

que es difícil de integrar (es de tipo Abel) por lo tanto volvemos a aplicar el método de Lie, encontrando que

ηx +(

ηy − ξx

) (y

x+

C1

x− 1

y

)− ξy

(y

x+

C1

x− 1

y

)2

+ ξ

(y

x2 +C1

x2

)− η

(1x

+1y2

)= 0

encontrando sólo dos simetrías

X1 =

(x2

y− xC1

)∂y, X2 = x2∂x + xy∂y,

el c.v. que induce X2 = x2∂x + xy∂y es:(

r =y

x, z(r) = − 1

x

)=⇒

(x = − 1

z(r), y = − r

z(r)

)

la ecuación (4.106) se rescribirá como:z′ =

r

rC1 − 1(4.107)

que es una cuadratura

z =r

C1+

ln(rC1 − 1)

C21

+ C2

por último deshacemos el c.v. obteniendo por lo tanto la solución (4.101)

− 1x

=y

xC1+

ln(C1

yx − 1

)

C1+ C2 (4.108)

Veremos ahora como obtener soluciones particulares. De las dos simetrías obtenidas al sistema de edps i.e. (4.97)tomamos X2 = x∂x + y

2 ∂y, i.e. una simetría de escala y calculamos la solución invariante que induce i.e.

dx

x=

2dy

y=⇒ y = ±

√2x

es una solución particular a la ecuación (4.96). A igual resultado podemos llegar si aplicamos nuestro métodopedestre (el AD) simplemente observando que dicha ecuación será dimensionalmente homogénea si introduci-mos una constante dimensional i.e. la ecuación (4.96) se rescribe como

y′′ = K1y′

y2 − K2

xy(4.109)

tal que [K1] = y2x−1, [K2] = y2x−1, por lo tanto la solución que nos sugiere el AD es del tipo

y K1 xy 1 2 0x 0 −1 1

=⇒ y ∼√

Kx

tal y como ya sabíamos por la solución invariante.

Ejemplo 4.3.3 Resolver la siguiente ecuación de segundo orden

y′′ =y′2

y+

(y − 1

y

)y′ (4.110)

Page 111: Ecuaciones Diferenciales Mediante Simetrías · vi CONTENTS Lieperomuchísimomássencillodeaplicaryaqueunonodebeaplicarningúnalgoritmoparaobtener las simetrías que inducen dichos

4.3. EJEMPLOS 101

Solución. Aplicando el método de Lie lo primero que tenemos que planear el es sistema de edps que nosdeterminarán las simetrías, este sistema de edps es:

yξyy + ξy = 0,(

1 − y2)

yξy + y2ηyy − 2y2ξxy − yηy + η = 0,

2yηxy − yξxx − 2y2ηy +(

1 − y2)

ξx − 2ηx − η(

y2 − 1)

= 0,(

1 − y2)

ηx + yηxx = 0,

cuyas soluciones son:

X1 = ∂x, X2 =y1

(y2 + 1 − y1

)

y∂y, (4.111)

comprobando que[X1, X2] = 0.

Las variables canónicas que induce este campo X1 son:

(t = y, u(t) = x) =⇒ (x = u(t), y = t) (4.112)

en estas nuvas variables la ode original (4.110) se rescribe como:

u′′ = −(1 + u′t2 − u′) u′

t(4.113)

que no parece fácil de resolver, entonces volvemos a utilizar las variables canónicas para reducir el orden de laode original (4.110), resultando

u′ = −(t2 − 1

)u2

t− u

t(4.114)

donde (t = y, u(t) =

1y′

), (4.115)

encontrando que la ecuación (4.114) es de tipo Bernoulli y cuya solución inmediata es:

u =t

t2 + C1t + 1, (4.116)

Una vez obtenida la solución a la ecuación (4.114) entonces sólo nos queda deshacer el c.v. (4.115) obteniendo lasiguiente ecuación diferencial:

y′ = y2 + yC1 + 1 (4.117)

que es fácil de integrar (es de tipo cuadratura) por lo tanto la solución de (4.110) es

x +2√(

C21 − 4

) arctanh2y + C1√(

C21 − 4

) + C2 = 0 (4.118)

o alternativamente:

y = −C1 −√

C21 − 1 tanh

(√C2

1 − 1(x + C2)

)si C2

1 > 1,

y = C1 +√

1 − C21 tan

(√1 − C2

1(x + C2)

)si C2

1 < 1,

y = C1 − (x + C2)−1 si C2

1 = 0,

tal y como queríamos hacer ver.

Page 112: Ecuaciones Diferenciales Mediante Simetrías · vi CONTENTS Lieperomuchísimomássencillodeaplicaryaqueunonodebeaplicarningúnalgoritmoparaobtener las simetrías que inducen dichos

102 CHAPTER 4. ECUACIONES DE SEGUNDO ORDEN.

Ejemplo 4.3.4 Resolver la siguiente ecuación de segundo orden

y′′ =

(3x− 2x

)y′ + 4y (4.119)

Solución. Aplicando el método de Lie lo primero que tenemos que planear el es sistema de edps que nosdeterminarán las simetrías, este sistema de edps es:

ξyy = 0,

−2(

3x− 2x

)ξy + ηyy − 2ξxy = 0,

−12yξy − ξxx +

(3x2 + 2x

)ξ −

(3x− 2x

)ξx + 2ηxy = 0,

−8yξx − 4η + 4yηy −(

3x− 2x

)ηx + ηxx = 0,

cuyas soluciones son:

X1 = y∂y, X2 =(

x2 − 1)

∂y, X3 = e−x2∂y, X4 =

y

x3 ∂x −2y2

x2 ∂y,

X5 =x2 − 1

x3 ∂x +2y

x2 ∂y, X6 =e−x2

x3 ∂x −2ye−x2

x2 ∂y, (4.120)

X7 =y(x2 − 1)ex2

x3 ∂x −2y2e−x2

x2 ∂y, X8 =(x − 1)2 (x + 1)2 ex2

x3 ∂x −2y (x − 1) (x + 1) ex2

x2 ∂y,

en este caso el álgebra que generan estos 8 campos (Xi)8i=1 no es soluble

Las variables canónicas que induce este campo X2 son:(

t = x, u(t) =y

(x2 − 1)

)=⇒

(x = t, y = −u(t) + u(t)2t2

)(4.121)

en estas nuvas variables la ode original (4.119) se rescribe como:

u′′ = −(2t4 − t2 + 3

)u′

t (t2 − 1)(4.122)

que parece fácil de resolver,

u′′

u′ = −(2t4 − t2 + 3

)

t (t2 − 1)=⇒ u′ =

t3e−t2+C1

(−1 + t)2 (1 + t)2 =⇒

u = −12

e−t2+C1

t2 − 1=

C2e−t2

t2 − 1+ C1

si deshacemos el c.v. obtenemos:

y

(x2 − 1)=

C2e−x2

x2 − 1+ C1 =⇒ y = C1

(x2 − 1

)+ C2e−x2

(4.123)

Si volvemos a utilizar las variables canónicas para reducir el orden de la ode original (4.122), resultando

u′ = −(2t4 − t2 + 3

)u

t (t2 − 1)(4.124)

donde (t = x, u(t) = − 2yx + (1 − x2)y′

(x − 1)2 (x + 1)2

), (4.125)

Page 113: Ecuaciones Diferenciales Mediante Simetrías · vi CONTENTS Lieperomuchísimomássencillodeaplicaryaqueunonodebeaplicarningúnalgoritmoparaobtener las simetrías que inducen dichos

4.3. EJEMPLOS 103

encontrando que la ecuación (4.124) es de tipo separable y cuya solución inmediata es:

u =t3e−t2+C1

(−1 + t)2 (1 + t)2 , (4.126)

Una vez obtenida la solución a la ecuación (4.124) entonces sólo nos queda deshacer el c.v. (4.125) obteniendo lasiguiente ecuación diferencial:

y′ =x3e−x2+C1

(x2 − 1)+

2xy

(x2 − 1)(4.127)

que es fácil de integrar (es de tipo lineal) por lo tanto la solución de (4.119) es

y = C1

(x2 − 1

)+ C2e−x2

(4.128)

tal y como ya sabíamos.

Ejemplo 4.3.5 Resolver la siguiente ecuación de segundo orden

y′′ = p (x) y′ + q(x) (4.129)

Solución. Aplicando el método de Lie lo primero que tenemos que planear el es sistema de edps que nosdeterminarán las simetrías, este sistema de edps es:

ξyy = 0,

−2p(x)ξy + ηyy − 2ξxy = 0,

−3yq(x)ξy − ξxx + p′(x)ξ − p (x) ξx + 2ηxy = 0,

−2q(x)ξx + q(x)ηy − p(x)ηx + ηxx − q′(x)ξ = 0,

cuyas soluciones son:

X1 = ∂y, X2 =∫

e∫

p(x)dxdx∂y, (4.130)

Las variables canónicas que induce este campo X1 son:

(t = x, u(t) = y) =⇒ (x = t, y = u(t)) (4.131)

en estas nuvas variables la ode original (4.129) se rescribe como:

u′′ = p (t) u′ + q(t) (4.132)

que parece fácil de resolver,

u =∫

e∫

p(t)dt

(∫q(t)e−

∫p(t)dtdt + C1

)dt + C2

si deshacemos el c.v. obtenemos:

y =∫

e∫

p(x)dx

(∫q(x)e−

∫p(x)dxdx + C1

)dx + C2 (4.133)

Si volvemos a utilizar las variables canónicas para reducir el orden de la ode original (4.132), resultando

u′ = p(t)u + q(t) (4.134)

Page 114: Ecuaciones Diferenciales Mediante Simetrías · vi CONTENTS Lieperomuchísimomássencillodeaplicaryaqueunonodebeaplicarningúnalgoritmoparaobtener las simetrías que inducen dichos

104 CHAPTER 4. ECUACIONES DE SEGUNDO ORDEN.

donde (t = x, u(t) = y′

), (4.135)

encontrando que la ecuación (4.134) es de tipo lineal y cuya solución inmediata es:

u =

(∫q(t)e−

∫p(t)dtdt + C1

)e∫

p(t)dt, (4.136)

Una vez obtenida la solución a la ecuación (4.134) entonces sólo nos queda deshacer el c.v. (4.135) obteniendo lasiguiente ecuación diferencial:

y′ =

(∫q(x)e−

∫p(x)dxdx + C1

)e∫

p(x)dx, (4.137)

que es fácil de integrar (cuadraturas) por lo tanto la solución de (4.129) es

y =∫

e∫

p(x)dx

(∫q(x)e−

∫p(x)dxdx + C1

)dx + C2 (4.138)

tal y como ya sabíamos.

Ejemplo 4.3.6 Resolver la siguiente ecuación de segundo orden

y′′ =y′

x+

3y2

2x3 , (4.139)

tipo Emden-Fowler.

Solución. Aplicando el método de Lie lo primero que tenemos que planear el es sistema de edps que nosdeterminarán las simetrías, este sistema de edps es:

x2ξyy = 0,

−2xξy + x2ηyy − 2x2ξxy = 0,

−9y2

2xξy − xξx + ξ + 2x2ηxy − x2ξxx = 0,

−3y

xη − 3y2

xξx +

3y2

2xηy +

9y2

2x3 ξ − xηx + x2ηxx = 0,

cuya solución es:X1 = x∂x + y∂y. (4.140)

De esta forma las variables canónicas que induce este campo X1 son:(

t =y

x, u(t) = ln x

)=⇒

(x = eu(t), y = teu(t)

)(4.141)

en estas nuevas variables la ode original (4.139) se rescribe como:

u′′ = −(u′)3

t

(1 +

32

t

)(4.142)

que parece muy fácil de resolver,

u′′

(u′)3 = −t

(1 +

32

t

)=⇒ − 1

2u′2 = −12

t3 − 12

t2 + C1 (4.143)

Page 115: Ecuaciones Diferenciales Mediante Simetrías · vi CONTENTS Lieperomuchísimomássencillodeaplicaryaqueunonodebeaplicarningúnalgoritmoparaobtener las simetrías que inducen dichos

4.3. EJEMPLOS 105

por lo tanto

u′ =dt

2√

12 t3 + 1

2 t2 + C1

=⇒ u =∫

dt√2t3 + 2t2 + C1

(4.144)

si C1 = 0 entonces encontramos una solución particular:

u = −√

2 arctanh12

√(2t + 2)

√2 (4.145)

así que si deshacemos el c.v. obtenemos:

ln x =∫ y

x da√2a3 + 2a2 + C1

+ C2. (4.146)

Por esta razón volveremos a utilizar las variables canónicas para reducir el orden de la ode original (4.139),resultando

u′ = −12

u3t (2 + 3t) (4.147)

donde (t =

y

x, u(t) = − x

y − xy′

), (4.148)

encontrando que la ecuación (4.147) es de tipo separable y cuya solución inmediata es:

u = ± 1√t3 + t2 + C1

, (4.149)

Una vez obtenida la solución a la ecuación (4.147) entonces sólo nos queda deshacer el c.v. (4.148) obteniendo lasiguiente ecuación diferencial:

x

y − xy′=

1√y2

x2 + y3

x3 + C1

(4.150)

que no es difícil de integrar, encontrando que

ln x =∫ y

x da√2a3 + 2a2 + C1

+ C2. (4.151)

es la solución más general a la ecuación planteada (4.139).

Veremos ahora como obtener soluciones particulares. De las simetrías obtenidas al sistema de edps i.e. (4.140)tomamos X1 = x∂x + y∂y, i.e. una simetría de escala y calculamos la solución invariante que induce i.e.

dx

x=

dy

y=⇒ y = ax

es una solución particular a la ecuación (4.139) siendo a ∈ R en particular encontramos que a = −2/3. A igualresultado podemos llegar si aplicamos nuestro método pedestre (el AD) simplemente observando que dichaecuación será dimensionalmente homogénea si introducimos una constante dimensional i.e. la ecuación (4.139)se rescribe como

y′′ =y′2

x+ K

3y2

2x3 (4.152)

tal que [K] = y−1x por lo tanto la solución que nos sugiere el AD es del tipo

y K xy 1 −1 0x 0 1 1

=⇒ y ∼x

K

tal y como ya sabíamos por la solución invariante. Esta transformación nos llevaría a obtener el mismo cv queel obetenido mediante GL.

Page 116: Ecuaciones Diferenciales Mediante Simetrías · vi CONTENTS Lieperomuchísimomássencillodeaplicaryaqueunonodebeaplicarningúnalgoritmoparaobtener las simetrías que inducen dichos

106 CHAPTER 4. ECUACIONES DE SEGUNDO ORDEN.

Ejemplo 4.3.7 Resolver la siguiente ecuación de segundo orden

y′′ =y′

x+

32

y2x3, (4.153)

tipo Emden-Fowler.

Solución. Aplicando el método de Lie lo primero que tenemos que plantear el es sistema de edps que nosdeterminarán las simetrías, este sistema de edps es:

x2ξyy = 0,

−2xξy + x2ηyy − 2x2ξxy = 0,

−92

y2x5ξy − xξx + ξ + 2x2ηxy − x2ξxx = 0,

−3y2x5η − 3y2x5ξx +32

y2x5ηy +92

y2x4ξ − xηx + x2ηxx = 0,

cuya solución es:X1 = x∂x − 5y∂y. (4.154)

De esta forma las variables canónicas que induce este campo X1 son:(

t = x5y, u(t) = ln x)

=⇒(

x = eu(t), y = te−5u(t))

(4.155)

en estas nuevas variables la ode original (4.153) se rescribe como:

u′′ = −12u′2 + 35u′3 − 32

u′3t2 (4.156)

que no parece muy fácil de resolver.

Por esta razón volveremos a utilizar las variables canónicas para reducir el orden de la ode original (4.153),resultando

u′ = u3(

35t − 32

t2)− 12u2 (4.157)

donde (t =

y

x, u(t) = − x

y − xy′

), (4.158)

encontrando que la ecuación (4.157) es de tipo Abel y por desgracia es de las que no sabemos resolver. Siaplicamos el método de Lie a dicha ecuación veríamos que no somos capaces de encontrar simetría alguna.

Por esto la solución a la que podemos llegar ode original (4.153) es del tipo:

y =a

e∫

b(a)da+C1, (4.159)

donde

b′ = b3(

35a − 32

a2)− 12b2, b = b(a). (4.160)

Veremos ahora como obtener soluciones particulares. De las simetrías obtenidas al sistema de edps i.e. (4.154)tomamos X1 = x∂x − 5y∂y, i.e. una simetría de escala y calculamos la solución invariante que induce i.e.

dx

x= −dy

5y=⇒ y =

a

x5

es una solución particular a la ecuación (4.153) siendo a ∈ R en particular encontramos que a = 70/3. A igualresultado podemos llegar si aplicamos nuestro método pedestre (el AD) simplemente observando que dicha

Page 117: Ecuaciones Diferenciales Mediante Simetrías · vi CONTENTS Lieperomuchísimomássencillodeaplicaryaqueunonodebeaplicarningúnalgoritmoparaobtener las simetrías que inducen dichos

4.3. EJEMPLOS 107

ecuación será dimensionalmente homogénea si introducimos una constante dimensional i.e. la ecuación (4.153)se rescribe como

y′′ =y′2

x+ K

32

y2x3 (4.161)

tal que [K] = y−1x por lo tanto la solución que nos sugiere el AD es del tipo

y K xy 1 −1 0x 0 −5 1

=⇒ y ∼1

Kx5

tal y como ya sabíamos por la solución invariante.

Ejemplo 4.3.8 Resolver la siguiente ecuación de segundo orden

y′′ = −y′

x− ey (4.162)

Solución. Aplicando el método de Lie lo primero que tenemos que plantear el es sistema de edps que nosdeterminarán las simetrías, este sistema de edps es:

x2ξyy = 0,

2xξy + x2ηyy − 2x2ξxy = 0,

3x2eyξy + xξx − ξ + 2x2ηxy − x2ξxx = 0,

x2eyη + 2x2eyξx − x2eyηy + xηx + x2ηxx = 0,

cuyas soluciones son:X1 = x∂x − 2∂y, X2 = x ln x∂x − 2 (1 + ln x) ∂y, (4.163)

comprobando que[X1, X2] = X1.

De esta forma las variables canónicas que induce este campo X1 son:

(t = y + ln x, u(t) = ln x) =⇒(

x = eu(t), y = t − 2u(t))

(4.164)

en estas nuevas variables la ode original (4.162) se rescribe como:

u′′ =(u′)3

e(t−2u)

e−2u(4.165)

que no parece muy fácil de resolver, por esta razón volveremos a utilizar las variables canónicas para reducir elorden de la ode original (4.162), resultando

u′ = u3et (4.166)

donde (t = y + 2 ln x, u(t) =

12 + xy′

), (4.167)

encontrando que la ecuación (4.166) es de tipo separable y cuya solución inmediata es:

u = ± 1√−2et + C1

, (4.168)

Page 118: Ecuaciones Diferenciales Mediante Simetrías · vi CONTENTS Lieperomuchísimomássencillodeaplicaryaqueunonodebeaplicarningúnalgoritmoparaobtener las simetrías que inducen dichos

108 CHAPTER 4. ECUACIONES DE SEGUNDO ORDEN.

pero imaginemos que no sabemos resolverla. En dicho caso aplicamos otra vez el método de Lie obteniendo porlo tanto (simplemente aplicamos el procedimiento standard):

ηt + (ηu − ξt)(

u3et)− ξu

(u3et

)2− ξ

(u3et

)− 3η

(u2et

)= 0

encontrando como solución los siguientes campos:

X1 = u3∂u, X2 =(

u + 2u3et)

∂u, X3 = ∂t −u

2∂u, X4 = e−t∂t

si por ejemplo elegimos X3 para calcular las v.c. encontramos:

(r = uet/2, z(r) = t

)=⇒

(t = z(r), u =

r

e1/2z(r)

)(4.169)

de tal forma que la ecuación (4.166) se rescribe en estas nuevas variables como:

z′ =2

r (2r2 + 1)=⇒ z(r) = − ln

(2r2 + 1

)+ 2 ln r + C1

deshaciendo el c.v. (y simplificando)

t = − ln(

2(

uet/2)2

+ 1)

+ 2 ln(

uet/2)

+ C1,

que es la solución (4.168) que ya habíamos obtenido.

Otra forma de atacar la ecuación (4.166) sería la de buscar un factor integrante µ que haga que (µ ∗ (4.166)) seaexacta, dicho factor integrante es:

µ =1u3 .

Una vez obtenida la solución a la ecuación (4.166) entonces sólo nos queda deshacer el c.v. (4.167) obteniendo lasiguiente ecuación diferencial:

y′ =−2 +

√−2e(y+2 ln x) + C1

x(4.170)

que es difícil de integrar, por eso volvemos a aplicar el método de Lie encontrando que

ηx +(

ηy − ξx

) (−2 +

√−2e(y+2 ln x) + C1

x

)− ξy

(−2 +

√−2e(y+2 ln x) + C1

x

)2

+

(−2

√(−2eyx2 + C1) + C1√(−2eyx2 + C1)x2

)+ 3η

(xey

√(−2eyx2 + C1)

)= 0

encontrando como solución los siguientes campos:

X1 = x∂x − 2∂y, X2 =√−2x2ey + C1∂y,

si por ejemplo elegimos X1 para calcular las v.c. encontramos:

(r = y + 2 ln x, z(r) = ln x) =⇒ (x = ez, y = r − 2z) (4.171)

de tal forma que la ecuación (4.170) se rescribe en estas nuevas variables como:

z′ =1√

−2er + C1=⇒ z(r) = −

2arctanh(√−2er+C1√

C1

)

√C1

+ C2

Page 119: Ecuaciones Diferenciales Mediante Simetrías · vi CONTENTS Lieperomuchísimomássencillodeaplicaryaqueunonodebeaplicarningúnalgoritmoparaobtener las simetrías que inducen dichos

4.3. EJEMPLOS 109

deshaciendo el c.v. (y simplificando)

ln x = −2arctanh

(√−2eyx2+C1√

C1

)

√C1

+ C2 (4.172)

que es la solución de (4.170) y por lo tanto de (4.162).

Ejemplo 4.3.9 Resolver la siguiente ecuación de segundo orden

y′′y′yx6 − 2y′3x6 + 2y′2yx5 + y5 = 0 (4.173)

Solución. Aplicando el método de Lie lo primero que tenemos que planear el es sistema de edps que nosdeterminarán las simetrías, este sistema de edps es:

yξyy + 2ξy = 0,

y2xηyy + 2xη − 2y2xξxy + 4y2ξy − 2yxηy = 0,

2y2x6ξx − 4yx7ηx + 2y2x7ηxy − y2x7ξxx − 2y2x5ξ = 0,

2y2x6ηx + y2x7ηxx + 4y6xξy = 0,

−6y6ξ + 4xy5η + 3y6xξx − 2y6xηy = 0,

y6xηx = 0,

cuyas soluciones son:

X1 = y2∂y, X2 = x2∂x, X3 = x∂x +32

y∂y, (4.174)

comprobando que X1 X2 X3

X1 0 0 − 32 X1

X2 0 0 −X2X3

32 X1 X2 0

i.e. [Xi, Xj

]= αXk

De esta forma las variables canónicas que induce este campo X1 son:(

t = x, u(t) =−1y

)=⇒

(x = t, y = − 1

u

)(4.175)

en estas nuevas variables la ode original (4.173) se rescribe como:

u′′ = −2u′

t− 1

u′t6 (4.176)

que no parece muy fácil de resolver, por esta razón volveremos a utilizar las variables canónicas para reducir elorden de la ode original (4.173), resultando

u′ = −2u

t− 1

ut6 (4.177)

donde (t = x, u(t) =

y′

y2

), (4.178)

Page 120: Ecuaciones Diferenciales Mediante Simetrías · vi CONTENTS Lieperomuchísimomássencillodeaplicaryaqueunonodebeaplicarningúnalgoritmoparaobtener las simetrías que inducen dichos

110 CHAPTER 4. ECUACIONES DE SEGUNDO ORDEN.

encontrando que la ecuación (4.177) es de tipo Bernoulli y cuya solución inmediata es:

u = ±√

2t + t2C1

t3 , (4.179)

Una vez obtenida la solución a la ecuación (4.177) entonces sólo nos queda deshacer el c.v. (4.178) obteniendo lasiguiente ecuación diferencial:

y′ =y2

√2x + x2C1

x3 (4.180)

que no es difícil de integrar, encontrando que

1y−

(2x + x2C1

)3/2

3x3 − C2 = 0 (4.181)

es la solución más general a la ecuación planteada (4.173).

Ejemplo 4.3.10 Resolver la siguiente ecuación de segundo orden

y′′ + xy′ + xy′3 − y − yy′2 = 0 (4.182)

Solución. Aplicando el método de Lie lo primero que tenemos que plantear el es sistema de edps que nosdeterminarán las simetrías, este sistema de edps es:

ξ − yξy − xξx + 2xηy − ξyy = 0,

−η − yηy + 2xξy + +3xηx + ηyy − 2y2ξxy = 0,

−3yξy + xξx − 2yηx + 2ηxy − ξxx + ξ = 0,

2yξx − η + ηxx + yηy + xηx = 0,

cuya solución es:X1 = y∂x − x∂y, (4.183)

De esta forma las variables canónicas que induce este campo X1 son:(

t = x2 + y2, u(t) = arctan(

x

y

))(4.184)

en estas nuevas variables la ode original (4.182) se rescribe como:

u′′ = −12

u′ (4t2u′2 + 4t3u′2 + t + 3)

t(4.185)

que no parece muy fácil de resolver, por esta razón volveremos a utilizar las variables canónicas para reducir elorden de la ode original (4.182), resultando

u′ = −2u3t (1 + t) − u (t + 3)

2t(4.186)

donde (t = x2 + y2, u(t) =

xy′ − y

2 (x2 + y2) (x + yy′)

), (4.187)

encontrando que la ecuación (4.186) es de tipo Bernoulli y cuya solución inmediata es:

u = ± 1

t√−4 + tet + C1

, (4.188)

Page 121: Ecuaciones Diferenciales Mediante Simetrías · vi CONTENTS Lieperomuchísimomássencillodeaplicaryaqueunonodebeaplicarningúnalgoritmoparaobtener las simetrías que inducen dichos

4.3. EJEMPLOS 111

Una vez obtenida la solución a la ecuación (4.186) entonces sólo nos queda deshacer el c.v. (4.187) obteniendo lasiguiente ecuación diferencial:

xy′ − y

(x + yy′)=

2√−4 + (x2 + y2) e(x2+y2) + C1

(4.189)

i.e.

y′ =2x + y

√−4 + (x2 + y2) e(x2+y2) + C1(

x√−4 + (x2 + y2) e(x2+y2) + C1 − 2y

) (4.190)

que es difícil de integrar, encontrando que

arctan(

x

y

∫ x2+y2 1a√−4 + aea + C1

da − C2 = 0 (4.191)

es la solución más general a la ecuación planteada (4.182).

Si aplicamos el método de Lie a la ecuación (4.190) encontraríamos que una de las simetrías que admite dichaecuación es X2 = −y∂x + x∂y que a suvez induce un c.v.

(r = x2 + y2, z(r) = − arctan

(x

y

))(4.192)

de tal forma que (4.190) se escribe como

z′ = − 1r√−4 + er + rC1

(4.193)

que es una ode tipo cuadratura

z =∫

− 1r√−4 + er + rC1

dr + C2 (4.194)

por lo tanto deshaciendo el c.v. llegaríamos a obtener la solución (4.191).

Ejemplo 4.3.11 Resolver la siguiente ecuación de segundo orden

y′′ =y2

x2 − 2yy′

x+ y′2 (4.195)

Solución. Las simetrías que admite la ODE son

X1 = x∂x, X2 = x∂y, (4.196)

por lo que[X1, X2] = X2.

De esta forma las variables canónicas que induce este campo X2 son:

t = x, u(t) =y

x, (4.197)

en estas nuevas variables la ode original se rescribe como:

u′′ =u′ (u′t2 − 2

)

t(4.198)

Page 122: Ecuaciones Diferenciales Mediante Simetrías · vi CONTENTS Lieperomuchísimomássencillodeaplicaryaqueunonodebeaplicarningúnalgoritmoparaobtener las simetrías que inducen dichos

112 CHAPTER 4. ECUACIONES DE SEGUNDO ORDEN.

y cuya solución es:

u =∫

− dt

t2 (ln t − C1)+ C2,

i.e.y

x= −

∫− dx

x2 (ln x − C1)+ C2,

Para reducir la ODE utilizamos las variables canónicas, resultando

u′ = u2t − 2u

t, (4.199)

de tipo Bernoulli donde

t = x, u(t) =y′x − y

x2 , (4.200)

y cuya solución inmediata es:

u = − 1t2 (ln t − C1)

, (4.201)

Una vez obtenida la solución a la ecuación de Bernoulli, u(t), entonces sólo nos queda deshacer el c.v. obteniendola siguiente ecuación diferencial:

y′x − y

x2 = − 1x2 (ln x − C1)

(4.202)

i.e.

y′ = − 1x (ln x − C1)

+y

x(4.203)

que no es difícil de integrar (lineal), encontrando que

y =

(∫− dx

x2 (ln x − C1)+ C2

)x (4.204)

es la solución más general a la ecuación planteada.

Page 123: Ecuaciones Diferenciales Mediante Simetrías · vi CONTENTS Lieperomuchísimomássencillodeaplicaryaqueunonodebeaplicarningúnalgoritmoparaobtener las simetrías que inducen dichos

Chapter 5

Ecuaciones de tercer orden.

5.1 Ejemplos

Veamos algunos ejemplos d eecuaciones de tercer orden y de como funciona el método con ellas.

Ejemplo 5.1.1 Resolver la siguiente ecuación de tercer orden

y′′′ =1y3 (5.1)

Solución. Aplicando el método de Lie lo primero que tenemos que plantear el es sistema de edps que nosdeterminarán las simetrías, este sistema de edps es:

y3ξy = 0,

y3ξyy = 0,

−9y3ξxy + 3y3ηyy = 0,

3y3ηxy − 3y3ξxx = 0,

y3ξyyy = 0,

−3y3ξxyy + y3ηyyy = 0,

−3y3ξxxy + y3ηxyy = 0,

−4ξy + 3y3ηxxy − 3y3ξxxx = 0,

3y

η + ηy − 3ξx + y3ηxxx = 0,

cuyas soluciones son:

X1 = ∂x, X2 = x∂x +34

y∂y, (5.2)

comprobando que[X1, X2] = αX1

vemos que al encontrar sólo 2 simetrías, entonces nuestra ode no va a poder ser reducida a una ode tipo cuadrat-uras.

113

Page 124: Ecuaciones Diferenciales Mediante Simetrías · vi CONTENTS Lieperomuchísimomássencillodeaplicaryaqueunonodebeaplicarningúnalgoritmoparaobtener las simetrías que inducen dichos

114 CHAPTER 5. ECUACIONES DE TERCER ORDEN.

De esta forma las variables canónicas que induce este campo X1 son:

(t = y, u(t) = x) =⇒ (x = u, y = t) (5.3)

en estas nuevas variables la ode original (5.1) se rescribe como:

u′′′ =3t3u′′2 − u′5

t3u′ (5.4)

que no parece muy fácil de resolver, por esta razón volveremos a utilizar las variables canónicas para reducir elorden de la ode original (5.1), resultando

u′′ =2u′2

u− u4

t3 (5.5)

donde (t = y, u(t) =

1y′

), (5.6)

encontrando que la ecuación (5.5) es algo compicadilla de resolver así que aplicamos d enuevo el método de Liei.e.

−3uξu − u2ξu u = 0,

3η − 3uηu + u2ηu u − 2u2ξtu = 0,

3u6

t3 ξu − 6uηt + 2u2ηtu − u2ξtt = 0,

4u5

t3 η − 2u6

t3 ξt −u6

t3 ηu − 3u6

t4 ξ + u2ηtt = 0,

comprobando que admite una única simetría

X3 = 3t∂t + u∂u (5.7)

que induce el siguiente c.v. que nos permite obtener una ode de primer orden:

s′ =19

(9r5 − 5r2) s3

r− 7

3s2 − 3s

r(5.8)

donde (r =

u

t1/3 , s(r) = − 3t1/3

u − 3u′t

)(5.9)

que por desgracia al ser de tipo Abel no sabemos como resolverla.

Intentaremos encontrar una solución particular mediante el método de los invariantes. La solución que induceel campo X2 = x∂x + 3

4 y∂y es por lo tanto:

dx

x=

4dy

3y=⇒ y = ax3/4 (5.10)

donde a ∈ R, un simple cálculo nos nuestra que

a = ±2√

2 (15)3/4

15

El método pedestre nos lleva a obtener la misma solución después de homogeneizar dimensionalmente la odemediante la introducción de una constante dimensional

y′′′ =K

y3 (5.11)

Page 125: Ecuaciones Diferenciales Mediante Simetrías · vi CONTENTS Lieperomuchísimomássencillodeaplicaryaqueunonodebeaplicarningúnalgoritmoparaobtener las simetrías que inducen dichos

5.1. EJEMPLOS 115

donde [K] = y4x−3, por lo tantoy K x

y 1 4 0x 0 −3 1

=⇒ y ≈(

Kx3)1/4

(5.12)

como ya sabíamos.

Ejemplo 5.1.2 Resolver la siguiente ecuación de tercer orden

y′′′ = −yy′′ (5.13)

Solución. Aplicando el método de Lie lo primero que tenemos que plantear el es sistema de edps que nosdeterminarán las simetrías, este sistema de edps es:

ξy = 0,

ξyy = 0,

−9ξxy + 3ηyy + yξy = 0,

η + yξx − 3ξxx + 3ηxy = 0,

ξyyy = 0,

−3ξxyy + ηyyy − yξyy = 0,

3ηxyy − 3ξxxy + yηyy − 2yξxy = 0,

−ξxxx + 3ηxxy − yξxx + 2yηxy = 0,

ηxxx + yηxx = 0,

cuyas soluciones son:X1 = ∂x, X2 = x∂x − y∂y, (5.14)

comprobando que[X1, X2] = X1

vemos que al encontrar sólo 2 simetrías, entonces nuestra ode no va a poder ser reducida a una ode tipo cuadrat-uras.

De esta forma las variables canónicas que induce este campo X1 son:

(t = y, u(t) = x) =⇒ (x = u, y = t) (5.15)

en estas nuevas variables la ode original (5.13) se rescribe como:

u′′′ = −u′′ (tu′2 − 3u′′)

u′ (5.16)

que no parece muy fácil de resolver, por esta razón volveremos a utilizar las variables canónicas para reducir elorden de la ode original (5.13), resultando

u′′ =3u′2

u− u′ut (5.17)

donde (t = y, u(t) =

1y′

), (5.18)

Page 126: Ecuaciones Diferenciales Mediante Simetrías · vi CONTENTS Lieperomuchísimomássencillodeaplicaryaqueunonodebeaplicarningúnalgoritmoparaobtener las simetrías que inducen dichos

116 CHAPTER 5. ECUACIONES DE TERCER ORDEN.

encontrando que la ecuación (5.17) es algo compicadilla de resolver así que aplicamos d enuevo el método deLie i.e.

−3uξu − u2ξu u = 0,

3η + 2u3tξu − 3uηu + u2ηu u − 2u2ξtu = 0,

u2tη + u3tξt + u3ξ − 6uηt + 2u2ηtu − u2ξtt = 0,

u3tηt + u2ηtt = 0,

comprobando que admite una única simetría

X3 = t∂t − 2u∂u (5.19)

que induce el siguiente c.v. que nos permite obtener una ode de primer orden:

s′ =

(2r3 + 6r2) s3

r+

(r2 − 7r

)s2

r− 3s

r(5.20)

donde(

r = ut2, s(r) =1

t2 (u′t + 2u)

)(5.21)

que por desgracia al ser de tipo Abel no sabemos como resolverla.

Intentaremos encontrar una solución particular mediante el método de los invariantes. La solución que induceel campo X2 = x∂x − y∂y es por lo tanto:

dx

x= −dy

y=⇒ y =

a

x(5.22)

donde a ∈ R, un simple cálculo nos nuestra que a = 3.

El método pedestre nos lleva a obtener la misma solución después de homogeneizar dimensionalmente la odemediante la introducción de una constante dimensional

y′′′ = −Kyy′′ (5.23)

donde [K] = y−1x−1, por lo tanto

y K xy 1 −1 0x 0 −1 1

=⇒ y ≈1

Kx(5.24)

como ya sabíamos.

Ejemplo 5.1.3 Resolver la siguiente ecuación de tercer orden

y′′′ =y′′2

y′(1 + y′)(5.25)

Solución. Aplicando el método de Lie lo primero que tenemos que plantear el es sistema de edps que nos

Page 127: Ecuaciones Diferenciales Mediante Simetrías · vi CONTENTS Lieperomuchísimomássencillodeaplicaryaqueunonodebeaplicarningúnalgoritmoparaobtener las simetrías que inducen dichos

5.1. EJEMPLOS 117

determinarán las simetrías, este sistema de edps es:

ξy = 0,

−ξx + ηy − 2ξy = 0,

ηx = 0,

ξyy = 0,

−10ξyy − 9ξxy + 3ηyy = 0,

4ηyy − 3ξxx + 3ηxy − 14ξxy − 4ξyy = 0,

2ηxy − 4ξxx − 5ξxy − ηyy = 0,

−2ηxx − ηxy − ξxx = 0,

ηxx = 0,

ξyyy = 0,

−3ξxyy − 2ξyyy + ηyyy = 0,

−6ξxyy − 3ξxxy + 3ηxyy + 2ηyyy − ξyyy = 0,

−6ξxxy + 6 + 3ηxyy − ξxxx + 3ηxxy − 3ξxyy + ηyyy = 0,

6ηxxy − 2ξxxx + ηxxx − 3ξxxy + 3ηxyy = 0,

2ηxxx − ξxxx + 3ηxxy = 0,

ηxxx = 0,

cuyas soluciones son:X1 = ∂y, X2 = ∂x, X3 = x∂x + y∂y, (5.26)

comprobando que[X1, X2] = αX1

vemos que al encontrar 3 simetrías, entonces nuestra ode va a poder ser reducida a una ode tipo cuadraturas.

De esta forma las variables canónicas que induce este campo X1 son:

(t = y, u(t) = x) =⇒ (x = u, y = t) (5.27)

en estas nuevas variables la ode original (5.25) se rescribe como:

u′′′ =u′′2 (2u′ + 3)

u′ (u′ + 1)(5.28)

que no parece muy fácil de resolver.

En esta ocasión utilizaremos las variables que canónicas que inducen cada una de las simetrías obtenidas hastallegar a una solución (en este caso podemos), resultando

u′′ =u′2 (2u + 3)

u (u + 1)(5.29)

donde (t = y, u(t) =

1y′

), (5.30)

la siguiente reducción es:

s′ = − s (2r + 3)

r (r + 1)(5.31)

donde (r = u(t), s(r) =

1u′

)(5.32)

Page 128: Ecuaciones Diferenciales Mediante Simetrías · vi CONTENTS Lieperomuchísimomássencillodeaplicaryaqueunonodebeaplicarningúnalgoritmoparaobtener las simetrías que inducen dichos

118 CHAPTER 5. ECUACIONES DE TERCER ORDEN.

por lo tanto

s = e∫

h(i)di , h(i) = − (2i + 3)

i (i + 1)(5.33)

donde (i = r, h(i) =

s′

s

)(5.34)

de esta forma, deshaciendo todos los cambios de variables y resolviendo las odes resultantes llegamos a lasolución de la ode original (5.25) resultando ser:

x = −√

1 − 2C1y − 2C1C2

C1− 1

2ln

(√1 − 2C1y − 2C1C2 − 1

)

C1+

+12

ln(√

1 − 2C1y − 2C1C2 + 1)

C1− 1

2ln (y + C2)

C1+ C3 (5.35)

este es en realidad el camino que deberíamos seguir en cada caso, pero he preferido ser especialmente pesadodesarrollando otras tácticas ya adquiridas.

El camino que siempre hemos seguido sería el de estudiar la ode (5.29) aplicando de nuevo el método de Lie i.e.planteando un nuevo sistema de edps para buscar las simetrías de dicha ecuación, en este caso resulta que dichaecuación admite muchas simetrías entre ellas X4 = (u2 + u)∂u

X4 = (u2 + u)∂u (5.36)

que induce el siguiente c.v. que nos permite obtener una ode de primer orden:

s′ = 2s2 (5.37)

donde (r = t, s(r) =

u′

u (u + 1)

)(5.38)

por lo tanto

s = − 12r − C1

(5.39)

y deshaciendo el c.v.u′

u (u + 1)= − 1

2t − C1=⇒ 1

u= C2

√2t − C1 − 1 (5.40)

de nuevo y por último deshacemos el último c.v. obteniéndose:

y′ = C2√

2y − C1 − 1 (5.41)

y cuya solución es precisamente la ya encontrada i.e. la solución (5.35).

Intentaremos encontrar una solución particular mediante el método de los invariantes. La solución que induceel campo X3 = x∂x + y∂y es por lo tanto:

dx

x=

dy

y=⇒ y = ax (5.42)

donde a ∈ R.

El método pedestre nos lleva a obtener la misma solución después de homogeneizar dimensionalmente la odemediante la introducción de una constante dimensional

y′y′′′ + Ky′2y′′′ = y′′2 (5.43)

donde [K] = y−1x, por lo tantoy K x

y 1 −1 0x 0 1 1

=⇒ y ≈x

K(5.44)

Page 129: Ecuaciones Diferenciales Mediante Simetrías · vi CONTENTS Lieperomuchísimomássencillodeaplicaryaqueunonodebeaplicarningúnalgoritmoparaobtener las simetrías que inducen dichos

5.1. EJEMPLOS 119

como ya sabíamos.

Ejemplo 5.1.4 Resolver la siguiente ecuación de tercer orden

y′′′ =2y′′2

y′+

y′′

x+

y′2

x(5.45)

Solución. Aplicando el método de Lie lo primero que tenemos que plantear el es sistema de edps que nosdeterminarán las simetrías, este sistema de edps es:

x2ξy = 0,

x2ηx = 0,

x2ξyy = 0,

−x2ξyx − x2ηyy − xξy = 0,

ξ − 5x2ηxy − xξx + x2ξxx = 0,

x2ηxx = 0,

x2ξyyy = 0,

2xξy + x2ηyyy − 3x2ξxyy + xξyy = 0,

ξ − xηy − xξx + 3x2ηxyy − 3x2ξxxy + 2xξxy − xηyy = 0,

−2xηx − x2ξxxx + 3x2ηxxy − 2xηxy + xξxx = 0,

−xηxx + x2ηxxx = 0,

cuyas soluciones son:X1 = ∂y, X2 = x∂x. (5.46)

De esta forma las variables canónicas que induce este campo X1 son:

(t = x, u(t) = y) =⇒ (x = t, y = u) (5.47)

en estas nuevas variables la ode original (5.45) se rescribe como:

u′′′ =2u′′2t + u′′u′ + u′3

u′t(5.48)

que no parece muy fácil de resolver.

En esta ocasión utilizaremos las variables que canónicas que inducen cada una de las simetrías obtenidas hastallegar a una solución (en este caso podemos), resultando

u′′ =2u′2

u+

u′

t+

u2

t(5.49)

donde (t = x, u(t) = y′

), (5.50)

aplicando de nuevo el método de Lie i.e. planteando un nuevo sistema de edps para buscar las simetrías dedicha ecuación, en este caso resulta que dicha ecuación admite muchas simetrías entre ellas

X3 = −t∂t + u∂u (5.51)

Page 130: Ecuaciones Diferenciales Mediante Simetrías · vi CONTENTS Lieperomuchísimomássencillodeaplicaryaqueunonodebeaplicarningúnalgoritmoparaobtener las simetrías que inducen dichos

120 CHAPTER 5. ECUACIONES DE TERCER ORDEN.

que induce el siguiente c.v. que nos permite obtener una ode de primer orden (tipo Bernoulli):

s′ = −(r3 − r2) s3

r− 2s

r(5.52)

donde (r = ut, s(r) =

1t (u + u′t)

)(5.53)

por lo tanto

s = ± 1

r√−2r + 1 + r2C1

(5.54)

y deshaciendo el c.v.1

t (u + u′t)= ± 1

ut√−2ut + 1 + u2t2C1

(5.55)

que resulta ser una ode de tipo homogénea, cuya solución es:

ln t − C2 − arctan h

(ut − 1√

−2ut + 1 + u2t2C1

)= 0 (5.56)

de nuevo y por último deshacemos el último c.v. (5.50) obteniéndose:

ln x − C2 − arctan h

(y′x − 1√

−2y′x + 1 + (y′x)2C1

)= 0 (5.57)

y cuya solución es:

y =

2 arctan h

(2xC2−1√4C1C2

2+1

)

√4C1C2

2 + 1+ C3 (5.58)

tal y como queríamos hacer ver.

Page 131: Ecuaciones Diferenciales Mediante Simetrías · vi CONTENTS Lieperomuchísimomássencillodeaplicaryaqueunonodebeaplicarningúnalgoritmoparaobtener las simetrías que inducen dichos

Chapter 6

Aplicaciones en problemas cosmológicos.

6.1 Distintas Aplicaciones

6.1.1 Lie method for the H-equation.

··H + K1H

·H + K2H−1

·H2 + K3H3 = 0

(6.1)

we consider··H = −

(K1H

·H + K2H−1

·H2 + K3H3

)(6.2)

i.e. we have an eq. type··H = f (t, H, H) where

f (t, H, H) = −(

K1H·

H + K2H−1·

H2 + K3H3

)(6.3)

therefore

ft = 0 (6.4)

fH = −(

K1·

H − K2H−2·

H2 + 3K3H2

)(6.5)

fH = −(

K1H + 2K2H−1·

H

)(6.6)

According with the previous theory, a vector field X

X = ξ(t, H)∂t + η(t, H)∂H (6.7)

is a symmetry of (6.2) iff

−ξ ft − η fH + ηtt + (2ηtH − ξtt) H′ + (ηHH − 2ξtH) H′2 − ξHH H′3 + ...

... +(ηH − 2ξt − 3H′ξH

)f −

[ηt + (ηH − ξt) H′ − H′2ξH

]fH′ = 0 (6.8)

121

Page 132: Ecuaciones Diferenciales Mediante Simetrías · vi CONTENTS Lieperomuchísimomássencillodeaplicaryaqueunonodebeaplicarningúnalgoritmoparaobtener las simetrías que inducen dichos

122 CHAPTER 6. APLICACIONES EN PROBLEMAS COSMOLÓGICOS.

simpliflying it is obtained:

−ξ ft = 0

−η fH = η

(K1

·H − K2H−2

·H2 + 3K3H2

)

= ηK1·

H − ηK2H−2·

H2 + η3K3H2

ηtt, (2ηtH − ξtt) H′, (ηHH − 2ξtH) H′2, ξHH H′3

ηH f = −ηH

(K1H

·H + K2H−1

·H2 + K3H3

)

= −ηHK1H·

H − ηHK2H−1·

H2 − ηHK3H3

−2ξt f = 2ξt

(K1H

·H + K2H−1

·H2 + K3H3

)

= 2ξtK1H·

H + 2ξtK2H−1·

H2 + 2ξtK3H3

−3·

HξH f = 3H′ξH

(K1H

·H + K2H−1

·H2 + K3H3

)

= 3ξHK1H·

H2 + 3ξHK2H−1·

H3 + 3ξHK3H3·

H

−[ηt + (ηH − ξt) H′ − H′2ξH

]fH′

−ηt fH′ = ηt

(K1H + 2K2H−1

·H

)

= ηtK1H + ηt2K2H−1H

−ηH

·H fH′ = ηH

(K1HH + 2K2H−1H2

)

= ηHK1HH + ηH2K2H−1H2

ξt H fH′ = −ξt

(K1HH + 2K2H−1H2

)

= −ξtK1HH − ξt2K2H−1H2

H2ξH fH′ = −ξH

(K1HH2 + 2K2H−1H3

)

= −ξHK1HH2 − ξH2K2H−1H3

therefore−η fH + ηtt + (2ηtH − ξtt) H′ + (ηHH − 2ξtH) H′2 − ξHH H′3 + ...

... +(ηH − 2ξt − 3H′ξH

)f −

[ηt + (ηH − ξt) H′ − H′2ξH

]fH′ = 0 (6.9)

Page 133: Ecuaciones Diferenciales Mediante Simetrías · vi CONTENTS Lieperomuchísimomássencillodeaplicaryaqueunonodebeaplicarningúnalgoritmoparaobtener las simetrías que inducen dichos

6.1. DISTINTAS APLICACIONES 123

ηK1H − ηK2H−2H2 + η3K3H2 + ηtt + 2ηtH H − ξttH+

ηHH H2 − 2ξtH H2 − ξHH H3 − ηHK1HH − ηHK2H−1H2 − ηHK3H3+

2ξtK1HH + 2ξtK2H−1H2 + 2ξtK3H3 + 3ξHK1HH2 + 3ξHK2H−1H3 + 3ξHK3H3H+

ηtK1H + ηt2K2H−1H + ηHK1HH + ηH2K2H−1H2 − ξtK1HH − ξt2K2H−1H2

− ξHK1HH2 − ξH2K2H−1H3

separating with respect to powers of H we obtain:

1. For H3

−ξHH H3 + 3ξHK2H−1H3 − ξH2K2H−1H3 = 0

− ξHH + ξHK2H−1 = 0 (6.10)

2. For H2

− ηK2H−2 + ηHH − 2ξtH − ηHK2H−1 + 2ξtK2H−1 + 3ξHK1H + ηH2K2H−1

− ξt2K2H−1 − ξHK1H

− ηK2H−2 + ηHH − 2ξtH + ηHK2H−1 + 2ξHK1H = 0 (6.11)

3. For H

ηK1H + 2ηtH H − ξttH − ηHK1HH + 2ξtK1HH + 3ξHK3H3H

+ ηt2K2H−1H + ηHK1HH − ξtK1HH

ηK1 + 2ηtH − ξtt + ξtK1H + 3ξHK3H3 + 2ηtK2H−1 = 0 (6.12)

4. For H0

3ηK3H2 + ηtt − ηHK3H3 + 2ξtK3H3 + ηtK1H = 0 (6.13)

Therefore we obtain the overdetermined system:

−ξHH + ξHK2H−1 = 0 (6.14)

−ηK2H−2 + ηHH − 2ξtH + ηHK2H−1 + 2ξHK1H = 0 (6.15)

ηK1 + 2ηtH − ξtt + ξtK1H + 3ξHK3H3 + 2ηtK2H−1 = 0 (6.16)

3ηK3H2 + ηtt − ηHK3H3 + 2ξtK3H3 + ηtK1H = 0 (6.17)

Solving (6.29-6.17), we find thatξ(t, H) = −at + b, η(t, H) = aH (6.18)

being a and b numerical constant. For this form of ξ and η eq. (6.2) admits a single symmetry

X = (at − b)∂t − (aH) ∂H , (6.19)

the knowledge of one symmetry X might suggest the form of a particular solution as an invariant of the operatorX, i.e. the solution of

dt

ξ (t, ρ)=

η (t, ρ)(6.20)

This particular solution is known as an invariant solution (generalization of similarity solution). In this case

H =1

at − b, (6.21)

which imply that

f = (at − b)1a

that is to said we again obtain a power law solution.

Page 134: Ecuaciones Diferenciales Mediante Simetrías · vi CONTENTS Lieperomuchísimomássencillodeaplicaryaqueunonodebeaplicarningúnalgoritmoparaobtener las simetrías que inducen dichos

124 CHAPTER 6. APLICACIONES EN PROBLEMAS COSMOLÓGICOS.

6.1.2 Fluidos perfectos con constantes variables

We will use the field equations in the form:

Rij −12

gijR =8πG(t)

c4(t)Tij + Λ(t)gij (6.22)

Where the energy momentum tensor is:

Tij = (ρ + p) uiuj − pgij (6.23)

where p = ωρ in such a way that ω ∈ [0, 1] .

The cosmological equations are now:

2f ′′

f+

( f ′)2

f 2 = −8πG(t)

c(t)2 p + c(t)2Λ(t) (6.24)

3( f ′)2

f 2 =8πG(t)

c(t)2 ρ + c(t)2Λ(t) (6.25)

where f stands for the scale factor. Applying the covariance divergence to the second member of equation (6.22)we get:

Tji;j =

(4c,j

c−

G,j

G

)T

ji −

c4(t)δjiΛ,j

8πG(6.26)

that simplifies to:

ρ′ + 3(ω + 1)ρH︸ ︷︷ ︸T

ji;j

= − Λ′c4

8πG− ρ

G′

G+ 4ρ

c′

c︸ ︷︷ ︸(6.27)

where H stands for the Hubble parameter (H =f ′

f). The last equation may be written in the form:

ρ′ + 3(ω + 1)ρH +Λ′c4

8πG+ ρ

G′

G− 4ρ

c′

c= 0 (6.28)

or the equivalentρ′

ρ+ 3(ω + 1)H +

Λ′c4

8πGρ+

G′

G− 4

c′

c= 0 (6.29)

In this equation we can take a general energy density ρ or use the black body equation of state ρ = aθ4 where

a =π2k4

B

15c3h3 so that equation (6.29) is now:

4θ′

θ− 3

[c′

c+

h′

h

]+ 3(ω + 1)H +

15Λ′c7h3

8π3Gk4Bθ4 +

G′

G− 4

c′

c= 0 (6.30)

Simplifying hypothesis

Hypothesis number one (H1). We introduce

G

c2 = const. ≡ B (6.31)

Page 135: Ecuaciones Diferenciales Mediante Simetrías · vi CONTENTS Lieperomuchísimomássencillodeaplicaryaqueunonodebeaplicarningúnalgoritmoparaobtener las simetrías que inducen dichos

6.1. DISTINTAS APLICACIONES 125

with [B] = LM−1. In this way we obtain directly a term that controls the behavior of the energy density.

ρ =b

Bt2 b ∈ R (6.32)

If we take ρ =b

Bt2 and introduce ρ = aθ4 we arrive at

b

Bt2 = aθ4 =⇒ θ =

(b

Ba

)1/4

t−1/2 (6.33)

Hypothesis number two (H2) We can make a previous hypothesis on the behavior of the Planck´s constant asfollows:

• ch = const. or equivalently h =A

c, where A is a proportionality constant. We are motivated to this relation

because it leaves the radiation constant a as constant with a constant kB = const. (see [?])

• h = Fc where [F] = L1M1. This hypothesis is introduced by some author to explain a possible variation ofthe fine structure constant. We will see that such hypothesis is irreconciliable with our model (see [?]).

Hypothesis number three (H3) As a mathematical possibility (self-similar relation), obtained by means of thesimilarity group acting on the equations, we will assume the following behavior for the cosmological “constant”:

Λ =d

c2(t)t2 (6.34)

where d ∈ R. This hypothesis is a strong one since in some way it forces to the relation f (t) = c(t)t, but this is adesired relation to get the causality principle preserved.

With this hypothesis we now deal with the equations

Lie methods for a perfect fluid model.

The field equations are:

2f ′′

f+

(f ′

f

)2

= −8πG

c2 p + Λc2, (6.35)

3(

f ′

f

)2

=8πG

c2 ρ + Λc2, (6.36)

ρ′ + 3 (ω + 1) ρH = 0, (6.37)

− Λ′c4

8πρG− G′

G+ 4

c′

c= 0, (6.38)

from (6.85) − (6.86) it is obtained

2f ′′

f− 2

(f ′

f

)2

= −8πG

c2 (p + ρ) , (6.39)

it is observed that

2f ′′

f− 2

(f ′

f

)2

= 2 (H)′ (6.40)

then

2 (H)′ = −8πG

c2 (p + Π + ρ) (6.41)

Page 136: Ecuaciones Diferenciales Mediante Simetrías · vi CONTENTS Lieperomuchísimomássencillodeaplicaryaqueunonodebeaplicarningúnalgoritmoparaobtener las simetrías que inducen dichos

126 CHAPTER 6. APLICACIONES EN PROBLEMAS COSMOLÓGICOS.

from (6.87) we can obtain

H = − ρ′

3 ((ω + 1) ρ)(6.42)

therefore (ρ′

ρ

)′= 12π (ω + 1)2 G

c2 ρ (6.43)

making 12π (ω + 1)2 = A then (ρ′

ρ

)′= A

G

c2 ρ (6.44)

expanding we obtain

ρ′′ =ρ′2

ρ+ A

G

c2 ρ2. (6.45)

A vector field XX = ξ(t, ρ)∂t + η(t, ρ)∂ρ (6.46)

is a symmetry of (6.110) iff

−ξ ft − η fρ + ηtt +(

2ηtρ − ξtt

)ρ′ +

(ηρρ − 2ξtρ

)ρ′2 − ξρρρ′3 + ...

... +(

ηρ − 2ξt − 3ρ′ξρ

)f −

[ηt +

(ηρ − ξt

)ρ′ − y′2ξρ

]fy′ = 0 (6.47)

By expanding and separating (6.92) with respect to powers of ρ′ we obtain the overdetermined system:

ξρρ + ρ−1ξρ = 0 (6.48)

ηρρ − 2ξtρ + ρ−2η − ρ−1ηρ = 0 (6.49)

2ηtρ − ξtt − 3AG

c2 ρ2ξρ − 2ρ−1ηt = 0 (6.50)

ηtt − A

(G′

c2 − 2Gc′

c3

)ρ2ξ − 2ηA

G

c2 ρ +(

ηρ − 2ξt

)A

G

c2 ρ2 = 0 (6.51)

Solving (6.103-6.106), we find that

ξ(t, ρ) = −2et + a, η(t, ρ) = (bt + d) ρ (6.52)

subject to the constrainG′

G= 2

c′

c+

bt + d − 4e

2et − a(6.53)

with a, b, e, and d constants. In order to solve (6.113) we can consider the following cases:

1. Case I: b = 0 and d − 4e = 0 that brings us to the solution

G′

G= 2

c′

c=⇒ G

c2 = B = const. (6.54)

The knowledge of one symmetry X might suggest the form of a particular solution as an invariant of theoperator X i.e. the solution of

dt

ξ (t, ρ)=

η (t, ρ)(6.55)

this particular solution is known as an invariant solution (generalization of similarity solution), thereforethe energy density is obtained from:

dt

−2et + a=

4eρ(6.56)

Page 137: Ecuaciones Diferenciales Mediante Simetrías · vi CONTENTS Lieperomuchísimomássencillodeaplicaryaqueunonodebeaplicarningúnalgoritmoparaobtener las simetrías que inducen dichos

6.1. DISTINTAS APLICACIONES 127

ρ =1

(2et − a)2 (6.57)

for simplicity we adoptρ = ρ0t−2 (6.58)

Once we have obtained ρ we can obtain f (the scale factor) from

ρ = Aω f−3(ω+1) =⇒ f = (Aωt)2

3(ω+1) , (6.59)

in this way we find H and from eq. (6.86) it is obtained the behaviour of Λ, being this:

c2Λ = 3H2 − 8πG

c2 ρ (6.60)

therefore

Λ =(

3β2 − 8πBρ0

) 1c2t2 =

l

c2t2 (6.61)

if we replace all these results into eq. (6.88) then we shall obtain the exact behaviour for c, i.e.

−(

1t

+c′

c

)λ =

c′

c(6.62)

being λ = l8πBρ0

, obtaining in this way:

c = c0t−α, (6.63)

with α =(

λ1+λ

).

If we follow each step then it will be obtained the complete solution for each case.

2. Case II, b = e = 0 which imply ξ(t, ρ) = a, η(t, ρ) = dρ and therefore:

G′

G= 2

c′

c− d

a(6.64)

that brings us to:G

c2 = K exp(−αt) (6.65)

where da = α

dt

ξ (t, ρ)=

η (t, ρ)=⇒ dt

a=

dρ=⇒ ρ = ρ0 exp (αt) (6.66)

this expression only has sense if α ∈ R−.

The scale factor f verifies the relationship:

ρ = Aω f−3(ω+1) =⇒ f = K f (exp (αt))−1

3(ω+1) (6.67)

in this way we find that

H = − α

3 (ω + 1)(6.68)

the cosmological “constant” is obtained from:

c2Λ =α2

3 (ω + 1)2 − 8πKρ0 =⇒ c2Λ = l (6.69)

if we replace all these results into eq. (6.88) then we shall obtain the exact behaviour for c, i.e.(

l

8πKρ0+ 2

)c′

c= α (6.70)

finding thatc = K exp(c0t) (6.71)

being c0 = α(l

8πKρ0+2

) , note that α ∈ R− that is to say, c is a decreasing function on time t.

Page 138: Ecuaciones Diferenciales Mediante Simetrías · vi CONTENTS Lieperomuchísimomássencillodeaplicaryaqueunonodebeaplicarningúnalgoritmoparaobtener las simetrías que inducen dichos

128 CHAPTER 6. APLICACIONES EN PROBLEMAS COSMOLÓGICOS.

3. Case III. d−4e2e = α, b

2e = β, −a2e = λ

G′

G− 2

c′

c=

βt + α

t + λ(6.72)

G

c2 = K(t + λ)α−βλ exp(βt) (6.73)

dt

ξ (t, ρ)=

η (t, ρ)=⇒ − dt

t + λ=

(βt + α + 2) ρ=⇒ (6.74)

ρ = ρ0(t + λ)−α+βλ−2 exp(−βt) (6.75)

following the above steps we find that the scale factor verifies the next relationship

ρ = Aω f−3(ω+1) =⇒ f = K f

((t + λ)−α+βλ−2 exp(−βt)

) −13(ω+1) (6.76)

f = K f (t + λ)−13−α+λβ−2

ω+1 e1

3(ω+1)βt (6.77)

rewriting α−λβ+23(ω+1)

= ν and β3(ω+1)

= µ

f = K f (t + λ)ν eµt (6.78)

it is easy find that

H =ν

(t + λ)+ µ (6.79)

therefore

c2Λ = 3(

ν + µ (t + λ)

(t + λ)

)2

− 8πKρ0

(t + λ)2 =⇒ (6.80)

Λ =K1

c2(t + λ)2 +K2

c2(t + λ)+

K3

c2 (6.81)

where K1 =(3ν2 − 8πKρ0

), K2 = 6µν and K3 = 3µ2.

Λ′ = −2K1

(c′

c3(t + λ)2 +1

c2(t + λ)3

)

− K2

(2

c′

c3(t + λ)+

1c2(t + λ)2

)− 2K3

c′

c3 (6.82)

In this way we find that c verifies the next eq.

c′

c

(2K1 + 2 + K2(t + λ) + 2K3(t + λ)2

)+

2K1 + βt + α

(t + λ)+ K2 = 0 (6.83)

being Ki = Ki8πKρ0

, therefore

ln c =∫

− 2K1 + βt + α + K2(t + λ)

(t + λ)(2K1 + 2 + K2(t + λ) + 2K3(t + λ)2

) (6.84)

Page 139: Ecuaciones Diferenciales Mediante Simetrías · vi CONTENTS Lieperomuchísimomássencillodeaplicaryaqueunonodebeaplicarningúnalgoritmoparaobtener las simetrías que inducen dichos

6.1. DISTINTAS APLICACIONES 129

6.1.3 Fluidos viscosos

The field equations are:

2f ′′

f+

(f ′

f

)2

= −8πG

c2 (p + Π) + Λc2 (6.85)

3(

f ′

f

)2

=8πG

c2 ρ + Λc2 (6.86)

ρ′ + 3 (ω + 1) ρH = −3HΠ (6.87)

Λc4

8πG+ ρ

G

G− 4ρ

c

c= 0 (6.88)

Π′ +Π

kγργ−1 = −3ρH − 12

Π

(3H + W

ρ′

ρ

)(6.89)

from (6.85) − (6.86) it is obtained

2f ′′

f− 2

(f ′

f

)2

= −8πG

c2 (p + Π + ρ)

it is observed that

2f ′′

f− 2

(f ′

f

)2

= 2 (H)′

then

2 (H)′ = −8πG

c2 (p + Π + ρ)

from (6.87) we can obtain

H = − ρ′

3 ((ω + 1) ρ + Π)

if we make the assumption Π = κρ with κ ∈ R− (that may be unfounded, a strong simplification), then we

obtain

H = − ρ′

3 (ω + 1 + κ) ρ

but in this new approach we are going to study the eq. (6.89) under the assumption Π = κρ with κ ∈ R−. In

this way we find that

Π′ +Π

kγργ−1 = −3ρH − 12

Π

(3H + W

ρ′

ρ

)

ρ′ + κk−1γ ρ2−γ = −3ρH − 1

2κρ

(3H + W

ρ′

ρ

)

then

H = −δ

(D

ρ′

ρ+ k−1

γ ρ1−γ

)where δ =

6 + 3κ, D =

(1 +

κW

2

)

now we go next to calculate H′,

H′ = −δ

(D

ρ′′

ρ− D

(ρ′

ρ

)2

+ k−1γ (1 − γ) ρ−γρ′

)

therefore

2 (H)′ = −8πG

c2 (p + Π + ρ)

(H)′ = −4π

c2 Gαρ

Page 140: Ecuaciones Diferenciales Mediante Simetrías · vi CONTENTS Lieperomuchísimomássencillodeaplicaryaqueunonodebeaplicarningúnalgoritmoparaobtener las simetrías que inducen dichos

130 CHAPTER 6. APLICACIONES EN PROBLEMAS COSMOLÓGICOS.

where α = (ω + 1 + κ). Simplifying it yields

ρ′′ =ρ′2

ρ− D−1k−1

γ βρβρ′ + AG

c2 ρ2 (6.90)

with (1 − γ) = β and A = 4πδ α.

As we can see this equation is similar to the eq. under study i.e.

y′′ = f (x, y, y′) i.e. f (t, ρ, ρ′) =ρ′2

ρ− k−1

γ βρβρ′ + AG

c2 ρ2

Model 1. Constants constants.

According with the previous theory, a vector field X

X = ξ(t, ρ)∂t + η(t, ρ)∂ρ (6.91)

is a symmetry of (6.90) iff

−ξ ft − η fρ + ηtt +(

2ηtρ − ξtt

)ρ′ +

(ηρρ − 2ξtρ

)ρ′2 − ξρρρ′3 + ...

... +(

ηρ − 2ξt − 3ρ′ξρ

)f −

[ηt +

(ηρ − ξt

)ρ′ − ρ′2ξρ

]fρ′ = 0 (6.92)

Simplifying it:

ξρρ + ρ−1ξρ = 0 (6.93)

ηρ−2 − ηρρ−1 +(

ηρρ − 2ξtρ

)+ 2ξρKρβ = 0 (6.94)

(2ηtρ − ξtt

)+ ξtKρβ − 3ξρ A

G

c2 ρ2 − ηt2ρ−1 + ηKβρβ−1 = 0 (6.95)

−η2AG

c2 ρ + ηtt + ηρ AG

c2 ρ2 − 2ξt AG

c2 ρ2 + ηtKρβ = 0 (6.96)

where K = k−1γ β, note that

f =ρ′2

ρ− k−1

γ βρβρ′ + AGρ2 =ρ′2

ρ− Kρβρ′ + AGρ2

ft = 0

fρ = −ρ′2

ρ2 − k−1γ β2ρβ−1ρ′ + 2A

G

c2 ρ = −ρ′2

ρ2 − Kβρβ−1ρ′ + 2AG

c2 ρ

fρ′ = 2ρ′

ρ− k−1

γ βρβ = 2ρ′

ρ− Kρβ

We take into account the following dimensional considerations:

[ηρρ

]=

η

ρρ,

[ξtρ

]=

ξ

tρ=⇒ ηt = ξρ

and[ηρ AGρ2

]= ηAGρ,

[ξt AGρ2

]=

ξ AGρ2

t

[ηtKρβ

]=

η

t2

Page 141: Ecuaciones Diferenciales Mediante Simetrías · vi CONTENTS Lieperomuchísimomássencillodeaplicaryaqueunonodebeaplicarningúnalgoritmoparaobtener las simetrías que inducen dichos

6.1. DISTINTAS APLICACIONES 131

we can see that

ηAGρ =ξ AGρ2

t⇐⇒ ηt = ξρ

ηAGρ =η

t2 ⇐⇒ AGρt2 = 1

ξAGρ2

t=

η

t2 ⇐⇒ ξAGρ2t = η ⇐⇒ ηt = ξρ

therefore with this constrain ηt = ξρ, we solve eqs. (6.93-6.96), finding

ξ(ρ, t) = −aβt + b

η(ρ, t) = aρ

with the help of eq. (6.96)

−2AaG

c2 ρ2 + AaG

c2 ρ2 + 2aβAG

c2 ρ2 = 0

we find that

− 1 + 2β = 0 ⇐⇒ β =12

(6.97)

where β = (1 − γ) , that is to say γ = 12 .

ξ(ρ, t) = − a

2t + b, η(ρ, t) = aρ (6.98)

and thereforeX =

(− a

2t + b

)∂t + (aρ) ∂ρ (6.99)

As we can see, I would like to emphasize the result (6.97) since I believe that it is very important. In this caseLie method indicates us that there is only one symmetry but iff γ = 1

2 i.e. the crucial valor which makes scaleinvariant the equations etc...

Cases.

1. a 6= 0 For this value of a eq. (6.90) admits a single symmetry

X =( a

2t − b

)∂t − (aρ) ∂ρ

the knowledge of one symmetry X might suggest the form of a particular solution as an invariant of theoperator X i.e. the solution of

dt

ξ (x, y)=

η (x, y)(6.100)

this particular solution is known as an invariant solution (generalization of similarity solution). In thiscase

ρ = ρ0t−2

for simplicity we have take b = 0, therefore the condition Π = κρ are very restrictive etc....

Model 2. G and Λ variables.

According with the previous theory, a vector field X

X = ξ(t, ρ)∂t + η(t, ρ)∂ρ (6.101)

Page 142: Ecuaciones Diferenciales Mediante Simetrías · vi CONTENTS Lieperomuchísimomássencillodeaplicaryaqueunonodebeaplicarningúnalgoritmoparaobtener las simetrías que inducen dichos

132 CHAPTER 6. APLICACIONES EN PROBLEMAS COSMOLÓGICOS.

is a symmetry of (6.90) iff

−ξ ft − η fρ + ηtt +(

2ηtρ − ξtt

)ρ′ +

(ηρρ − 2ξtρ

)ρ′2 − ξρρρ′3 + ...

... +(

ηρ − 2ξt − 3ρ′ξρ

)f −

[ηt +

(ηρ − ξt

)ρ′ − ρ′2ξρ

]fρ′ = 0 (6.102)

Therefore the new results are:

ξρρ + ρ−1ξρ = 0 (6.103)

ηρ−2 − ηρρ−1 +(

ηρρ − 2ξtρ

)+ 2ξρKρβ = 0 (6.104)

(2ηtρ − ξtt

)+ ξtKρβ − 3ξρ A

G

c2 ρ2 − ηt2ρ−1 + ηKβρβ−1 = 0 (6.105)

−ξAG′

c2 ρ2 − η2AG

c2 ρ + ηtt + ηρ AG

c2 ρ2 − 2ξt AG

c2 ρ2 + ηtKρβ = 0 (6.106)

where K = k−1γ β, note that

f =ρ′2

ρ− Kρβρ′ + A

G

c2 ρ2

ft = AG′

c2 ρ2

fρ = −ρ′2

ρ2 − Kβρβ−1ρ′ + 2AG

c2 ρ

fρ′ = 2ρ′

ρ− Kρβ

therefore with this constrain ηt = ξρ, we solve eqs. (6.103-6.106), finding

ξ(ρ, t) = −aβt + b, η(ρ, t) = aρ

withG′

G=

(1 − 2β)a

aβt − b

where β = (1 − γ) .

Cases We have only two simple cases:

1. a = 0 which correspond to G = const.

2. a 6= 0 which correspond to

G = Gβ (βt + b)−2+ 1

β

it is observed that if γ = 12 then G = const. since β = (1 − γ), if γ >

12 then G is a growing function on

time and if γ <12 then G is a decreasing function on time. For this form of G eq. (6.90) admits a single

symmetryX = (aβt − b) ∂t − (aρ) ∂ρ

the knowledge of one symmetry X might suggest the form of a particular solution as an invariant of theoperator X i.e. the solution of

dt

ξ (x, y)=

η (x, y)(6.107)

Page 143: Ecuaciones Diferenciales Mediante Simetrías · vi CONTENTS Lieperomuchísimomássencillodeaplicaryaqueunonodebeaplicarningúnalgoritmoparaobtener las simetrías que inducen dichos

6.1. DISTINTAS APLICACIONES 133

this particular solution is known as an invariant solution (generalization of similarity solution). In thiscase

t− 1

β = ρ

for simplicity we have take b = 0.

With this result we can apply the standard Lie procedure or through pedestrian method to try to obtainthe rest of quantities like ρ. In this way taking into account dimensional considerations from eq.

ρ′′ =ρ′2

ρ− k−1

γ βρβρ′ + AGρ2

it is obtained the next relationship between quantities

k−1γ βρβt ⋍ 1 and AGρt2

⋍ 1

this last relationship is the known relation for the inertia obtained by Sciama. From these relationship weobtain

ρ ≈ A−1 (βt + b)2− 1

β t−2 ≈ A−1 (βt)− 1

β

we can see that this result verifies the relation

k−1γ βρβt ⋍ 1 since k−1

γ βt−1t ⋍ 1

therefore the condition Π = κρ are very restrictive.

Model 3. G, c and Λ variables.

According with the previous theory, a vector field X

X = ξ(t, ρ)∂t + η(t, ρ)∂ρ (6.108)

is a symmetry of (6.90) iff

−ξ ft − η fρ + ηtt +(

2ηtρ − ξtt

)ρ′ +

(ηρρ − 2ξtρ

)ρ′2 − ξρρρ′3 + ...

... +(

ηρ − 2ξt − 3ρ′ξρ

)f −

[ηt +

(ηρ − ξt

)ρ′ − ρ′2ξρ

]fρ′ = 0 (6.109)

ξρρ + ρ−1ξρ = 0, (6.110)

ηρ−2 − ηρρ−1 + ηρρ − 2ξtρ + 2Aξρρ1−s = 0, (6.111)

2ηtρ − ξtt + Aξtρ1−s − 3Bξρ

G

c2 ρ2 − 2ηtρ−1 + A (1 − s) ηρ−s = 0, (6.112)

−Bξ

(G

c2 − 2Gc

c3

)ρ2 − 2Bη

G

c2 ρ + ηtt +(

ηρ − 2ξt

)B

G

c2 ρ2 + Aηtρ1−s = 0, (6.113)

Solving (6.110-6.113), we find thatξ(ρ, t) = −ast + b, η(ρ, t) = aρ (6.114)

with the constraintG

G= 2

c

c+

(1 − 2s)a

b − ast, (6.115)

being a and b numerical constant.

Thus we have found all the possible forms of G and c for which eq. (6.90) admits symmetries. There are twocases with respect to the values of the constant a, a = 0 which correspond to G = const, c = const. and a 6= 0which correspond to

G

c2 = K [ast + b]−2+ 1s , (6.116)

Page 144: Ecuaciones Diferenciales Mediante Simetrías · vi CONTENTS Lieperomuchísimomássencillodeaplicaryaqueunonodebeaplicarningúnalgoritmoparaobtener las simetrías que inducen dichos

134 CHAPTER 6. APLICACIONES EN PROBLEMAS COSMOLÓGICOS.

with K a constant of integration. If s = 12 , which correspond to γ = 1

2 , then G and c remain constants. For thisform of G and c eq. (6.90) admits a single symmetry

X = (ast − b) ∂t − (aρ) ∂ρ. (6.117)

this is not a surprising result since as already have been point out by M. Szydlowki and M. Heller there is onlyone symmetry.

The knowledge of one symmetry X might suggest the form of a particular solution as an invariant of the operatorX, i.e. the solution of

dt

ξ (t, ρ)=

η (t, ρ)(6.118)

This particular solution is known as an invariant solution (generalization of similarity solution). In this case

ρ = ρ0t−1s , (6.119)

where for simplicity we have taken b = 0, and ρ0 is a constant of integration (note that s = (1 − γ) being γ thebulk viscous parameter).

We can apply a pedestrian method to try to obtain the same results. In this way, taking into account dimensionalconsiderations, from the eq. (6.90) we obtain the following relationships between density, time and gravitationalconstant:

k−1γ sρst ⋍ 1, B

G

c2 ρt2⋍ 1. (6.120)

This last relationship is also known as the relation for inertia obtained by Sciama. From these relationship weobtain

ρ ≈ B−1K−1c2 [ast + b]2s−1

s t−2 ≈ B−1K−1c2 [ast]−1s . (6.121)

We can see that this result verifies the relation

k−1γ sρst ⋍ 1. (6.122)

Once we have obtained ρ we can obtain f (the scale factor) from

ρ = Aω,κ f−3(ω+1+κ) =⇒ f =(

Aω,κρ−10 t

) 13(ω+1+κ)s , (6.123)

in this way we find H and from eq. (6.86) it is obtained the behaviour of Λ, being this:

Λ =

(3κ2 − 8πK(as)−2+ 1

s ρ0

)

c2t2 =l

c2t2 (6.124)

being κ =(Aω,κρ−1

0 )3(ω+1+κ)s

if we replace all these results into eq. (6.88) then we shall obtain the exact behaviour for c,i.e.

−(

l

4πKρ0

+1 − 2s

s

)1t

=

(l

4πKρ0

+ 2

)c

c, (6.125)

thereforec = c0t−ε, (6.126)

being ε =

(λ+ (1−2s)

s

)

(λ+2)and λ = l

4πKρ0.

Therefore the conditions Π = κρ together with the equation (6.89) are very restrictive. Hence we have shownthat under these assumptions there are no another possible solutions for the field equations.